92
ĐI HC QUC GIA HÀ NI TRƯNG ĐI HC KHOA HC T NHIÊN KHOA TOÁN - CƠ - TIN HC Nguyn Văn Tun MT S LP BÀI TOÁN V PHƯƠNG TRÌNH HÀM LUN VĂN THC SĨ KHOA HC Chuyên ngành : Phương pháp toán sơ cp Mã s : 60 - 46 - 40 NGƯI HƯNG DN KHOA HC TS. Nguyn Thành Văn Hà Ni - 2011

Đ—I H¯C QU¨C GIA HÀ N¸I TRƯ˝NG Đ—I H¯C KHOA H¯C TÜ NHIÊN KHOA … · 2016-05-09 · Đ—I H¯C QU¨C GIA HÀ N¸I TRƯ˝NG Đ—I H¯C KHOA H¯C TÜ NHIÊN KHOA TOÁN

  • Upload
    others

  • View
    5

  • Download
    0

Embed Size (px)

Citation preview

Page 1: Đ—I H¯C QU¨C GIA HÀ N¸I TRƯ˝NG Đ—I H¯C KHOA H¯C TÜ NHIÊN KHOA … · 2016-05-09 · Đ—I H¯C QU¨C GIA HÀ N¸I TRƯ˝NG Đ—I H¯C KHOA H¯C TÜ NHIÊN KHOA TOÁN

ĐẠI HỌC QUỐC GIA HÀ NỘITRƯỜNG ĐẠI HỌC KHOA HỌC TỰ NHIÊN

KHOA TOÁN - CƠ - TIN HỌC

Nguyễn Văn Tuấn

MỘT SỐ LỚP BÀI TOÁN VỀ PHƯƠNG TRÌNH HÀM

LUẬN VĂN THẠC SĨ KHOA HỌCChuyên ngành : Phương pháp toán sơ cấp

Mã số : 60 - 46 - 40

NGƯỜI HƯỚNG DẪN KHOA HỌCTS. Nguyễn Thành Văn

Hà Nội - 2011

Page 2: Đ—I H¯C QU¨C GIA HÀ N¸I TRƯ˝NG Đ—I H¯C KHOA H¯C TÜ NHIÊN KHOA … · 2016-05-09 · Đ—I H¯C QU¨C GIA HÀ N¸I TRƯ˝NG Đ—I H¯C KHOA H¯C TÜ NHIÊN KHOA TOÁN

Mục lục

Lời nói đầu . . . . . . . . . . . . . . . . . . . . . . . . . . . . . . . . . . . . . . . . . . . . . . . . . . . . . . . . . . . . . . . . iii

Chương 1. Kiến thức chuẩn bị 1

1.1. Hàm số liên tục . . . . . . . . . . . . . . . . . . . . . . . . . . . . . . . . . . . . . . . . . . . . . . . . . . . . . . . . . 1

1.1.1. Định nghĩa về hàm số liên tục . . . . . . . . . . . . . . . . . . . . . . . . . . . . . . . . . . . . . . . . 1

1.1.2. Tính chất của hàm số liên tục . . . . . . . . . . . . . . . . . . . . . . . . . . . . . . . . . . . . . . . . . 2

1.2. Hàm số chẵn, hàm số lẻ . . . . . . . . . . . . . . . . . . . . . . . . . . . . . . . . . . . . . . . . . . . . . . . . . . 3

1.3. Hàm số tuần hoàn và phản tuần hoàn . . . . . . . . . . . . . . . . . . . . . . . . . . . . . . . . . . . . . . 3

1.4. Tính đơn điệu của hàm số . . . . . . . . . . . . . . . . . . . . . . . . . . . . . . . . . . . . . . . . . . . . . . . . 4

1.5. Tính chất ánh xạ của hàm số . . . . . . . . . . . . . . . . . . . . . . . . . . . . . . . . . . . . . . . . . . . . . . 4

Chương 2. Một số phương trình hàm cơ bản 5

2.1. Phương trình hàm Cauchy . . . . . . . . . . . . . . . . . . . . . . . . . . . . . . . . . . . . . . . . . . . . . . . . 5

2.2. Phương trình hàm Jensen . . . . . . . . . . . . . . . . . . . . . . . . . . . . . . . . . . . . . . . . . . . . . . . . 11

2.3. Vận dụng phương trình hàm cơ bản vào giải toán . . . . . . . . . . . . . . . . . . . . . . . . . . 14

Chương 3. Một số phương pháp giải phương trình hàm 31

3.1. Phương pháp sử dụng tính liên tục của hàm số . . . . . . . . . . . . . . . . . . . . . . . . . . . . . 31

3.2. Phương pháp Qui nạp toán học . . . . . . . . . . . . . . . . . . . . . . . . . . . . . . . . . . . . . . . . . . . 43

3.3. Phương pháp sử dụng tính đơn điệu của hàm số . . . . . . . . . . . . . . . . . . . . . . . . . . . . 46

3.4. Phương pháp sử dụng tính chất ánh xạ của hàm số . . . . . . . . . . . . . . . . . . . . . . . . . 58

i

Page 3: Đ—I H¯C QU¨C GIA HÀ N¸I TRƯ˝NG Đ—I H¯C KHOA H¯C TÜ NHIÊN KHOA … · 2016-05-09 · Đ—I H¯C QU¨C GIA HÀ N¸I TRƯ˝NG Đ—I H¯C KHOA H¯C TÜ NHIÊN KHOA TOÁN

3.5. Phương pháp điểm bất động . . . . . . . . . . . . . . . . . . . . . . . . . . . . . . . . . . . . . . . . . . . . . 71

3.6. Phương pháp đưa về dãy số . . . . . . . . . . . . . . . . . . . . . . . . . . . . . . . . . . . . . . . . . . . . . . 79

3.6.1. Cơ sở lý thuyết phương trình sai phân . . . . . . . . . . . . . . . . . . . . . . . . . . . . . . . . . 79

3.6.2. Một số bài toán vận dụng . . . . . . . . . . . . . . . . . . . . . . . . . . . . . . . . . . . . . . . . . . . . 81

Kết luận . . . . . . . . . . . . . . . . . . . . . . . . . . . . . . . . . . . . . . . . . . . . . . . . . . . . . . . . . . . . . . . . . . . 86

Tài liệu tham khảo . . . . . . . . . . . . . . . . . . . . . . . . . . . . . . . . . . . . . . . . . . . . . . . . . . . . . . . . . 87

ii

Page 4: Đ—I H¯C QU¨C GIA HÀ N¸I TRƯ˝NG Đ—I H¯C KHOA H¯C TÜ NHIÊN KHOA … · 2016-05-09 · Đ—I H¯C QU¨C GIA HÀ N¸I TRƯ˝NG Đ—I H¯C KHOA H¯C TÜ NHIÊN KHOA TOÁN

Lời nói đầu

Phương trình hàm là một trong những lĩnh vực hay và khó của toán học sơ cấp. Trongcác kì thi Olympic Toán học Quốc gia, Khu vực và Quốc tế thường xuyên xuất hiện cácbài toán phương trình hàm. Các bài toán này thường là khó, đôi khi là rất khó. Để giải cácbài toán đó, trước tiên ta cần nắm vững các tính chất cơ bản về hàm số, một số phươngtrình hàm cơ bản, các phương pháp giải và có sự vận dụng thích hợp. Với mong muốncó thể tiếp cận được với bài toán phương trình hàm trong các kì thi Olympic Toán, luậnvăn sẽ đi theo hướng trên. Cụ thể, luận văn chia làm ba chương:

Chương 1. Kiến thức chuẩn bị.Trình bày về các định nghĩa tính chất cơ bản của hàm số.

Chương 2. Một số phương trình hàm cơ bản.Trình bày về một số phương trình hàm cơ bản, các dạng liên quan trong một sốtrường hợp riêng lẻ hay mở rộng, trong đó đi sâu về phương trình hàm Cauchy, cùngvận dụng của nó. Trong chương này cũng sẽ có nhiều hơn bài toán thi Olympic đểthấy được tầm quan trọng của các phương trình hàm cơ bản.

Chương 3. Một số phương pháp giải phương trình hàm.Trình bày một số phương pháp giải phương trình hàm thông dụng. Ở mỗi phươngpháp sẽ bắt đầu bằng phương pháp chung, một số lý thuyết hữu ích liên quan, cácbài toán vận dụng, cuối cùng là phần bài tập với gợi ý kèm theo. Trong đó cũng cókhông ít các bài toán khó, các bài toán thi học sinh giỏi giúp chúng ta tìm hiểu sâuhơn, nắm vững hơn từng phương pháp.

Để hoàn thành luận văn, trước hết tôi xin được gửi lời cảm ơn sâu sắc tới TS.Nguyễn Thành Văn đã dành thời gian hướng dẫn, đánh giá, chỉ bảo, tận tình giúp đỡtrong quá trình xây dựng đề tài cũng như hoàn thiện luận văn. Qua đây, tôi cũng xin gửilời cảm ơn chân thành tới các thầy cô, các anh chị học viên cao học toán khóa 2009-2011,Ban giám hiệu, Phòng sau đại học, Khoa Toán - Cơ - Tin học trường đại học Khoa họcTự nhiên Hà Nội đã tạo điều kiện, giúp đỡ trong suốt quá trình hoàn thành khóa học.

iii

Page 5: Đ—I H¯C QU¨C GIA HÀ N¸I TRƯ˝NG Đ—I H¯C KHOA H¯C TÜ NHIÊN KHOA … · 2016-05-09 · Đ—I H¯C QU¨C GIA HÀ N¸I TRƯ˝NG Đ—I H¯C KHOA H¯C TÜ NHIÊN KHOA TOÁN

Tuy đã có nhiều cố gắng nhưng do thời gian và khả năng còn hạn hẹp nên các vấn đềtrình bày trong luận văn còn chưa được trình bày sâu sắc và không thể tránh khỏi nhữngsai sót. Mong nhận được sự góp ý xây dựng của thầy cô cùng các bạn.

Tôi xin chân thành cảm ơn !

Hà nội, ngày 1 tháng 11 năm 2011

Học viên

Nguyễn Văn Tuấn

iv

Page 6: Đ—I H¯C QU¨C GIA HÀ N¸I TRƯ˝NG Đ—I H¯C KHOA H¯C TÜ NHIÊN KHOA … · 2016-05-09 · Đ—I H¯C QU¨C GIA HÀ N¸I TRƯ˝NG Đ—I H¯C KHOA H¯C TÜ NHIÊN KHOA TOÁN

Chương 1

Kiến thức chuẩn bị

Trong chương này, ta chỉ trình bày các định nghĩa, tính chất cơ bản liên quan đến hàmsố phục vụ cho các bài toán được trình bày trong các chương sau. Ta quan tâm đến cáchàm số f (x) với tập xác định D( f )⊆ R và tập giá trị R( f )⊆ R.

1.1 Hàm số liên tục

1.1.1 Định nghĩa về hàm số liên tục

Định nghĩa 1.1. Giả sử hàm số f xác định trên khoảng (a,b)⊂R và x0 ∈ (a,b). Ta nói

f (x) là hàm liên tục tại x0 nếu với mọi dãy số {xn}∞

n=1, xn ∈ (a,b) sao cho limn→+∞

xn = x0

ta đều có limn→+∞

f (xn) = f (x0).

Định nghĩa này tương đương với định nghĩa sau:

Định nghĩa 1.2. Hàm f (x), xác định trên (a;b), được gọi là liên tục tại x0 ∈ (a,b) nếu

limx→x0

f (x) = f (x0).

Điều này có nghĩa ∀ε > 0 tồn tại δ (ε), chỉ phụ thuộc vào ε , sao cho ∀x : |x− x0| < δ

thì | f (x)− f (x0)|< ε.

Hàm số không liên tục tại x0 được gọi là gián đoạn tại điểm x0.

Định nghĩa 1.3. Giả sử hàm số f xác định trên tập hợp J, trong đó J là một khoảng

hoặc hợp của nhiều khoảng hoặc R. Ta nói hàm số f liên tục trên J nếu nó liên tục tại

mọi điểm thuộc tập hợp đó.

Định nghĩa 1.4. Hàm số f xác định trên đoạn [a,b] được gọi là liên tục trên đoạn [a,b]nếu nó liên tục trên khoảng (a,b) và

limx→a+

f (x) = f (a), limx→b−

f (x) = f (b).

1

Page 7: Đ—I H¯C QU¨C GIA HÀ N¸I TRƯ˝NG Đ—I H¯C KHOA H¯C TÜ NHIÊN KHOA … · 2016-05-09 · Đ—I H¯C QU¨C GIA HÀ N¸I TRƯ˝NG Đ—I H¯C KHOA H¯C TÜ NHIÊN KHOA TOÁN

1.1.2 Tính chất của hàm số liên tục

Ở mục trên, ta đã có các cách để xác định một hàm số liên tục. Tuy nhiên việc sử dụngcác định nghĩa đó không phải lúc nào cũng đơn giản. Do vậy, người ta đã chứng minhđược các tính chất rất hữu ích, giúp ta xác định nhanh hàm số liên tục, như sau:

1. Các hàm số sơ cấp cơ bản như: hàm lũy thừa, hàm căn thức, hàm lượng giác, hàmsố mũ, hàm logarit,... là các hàm số liên tục trên miền xác định của nó.

2. Giả sử f (x), g(x) là các hàm số liên tục trên D ∈ R.Khi đó ( f +g)(x) = f (x)+g(x), ( f ◦g)(x) = f [g(x)] là các hàm liên tục trên D.

3. Giả sử g(x) 6= 0, ∀x∈R. Khi đóf (x)g(x)

cũng là hàm liên tục. Trong trường hợp ngược

lại thì nó liên tục trên tập xác định của nó.

Một số tính chất quan trọng khác của hàm số liên tục:

Định lý 1.1. (Định lý về giá trị trung gian của hàm số liên tục)

Giả sử hàm số f liên tục trên đoạn [a,b]. Nếu f (a) 6= f (b) thì với mọi số thực M nằm

giữa f (a) và f (b), tồn tại ít nhất một điểm c ∈ (a,b) sao cho f (c) = M.

Mệnh đề 1.1. Giả sử f (x), g(x) là hai hàm xác định và liên tục trên R. Khi đó nếu

f (x) = g(x), ∀x ∈Q thì ta có f (x)≡ g(x) trên R.

Chứng minh. Với mỗi x ∈ R, ta xét dãy số hữu tỉ sn, n ∈ N thỏa mãn limn→+∞

sn = x.

Do f (r) = g(r), ∀r ∈ Q, nên f (sn) = g(sn), ∀n ∈ N. Lấy giới hạn hai vế khi n→ +∞,

với chú ý f (x), g(x) là các hàm liên tục, ta có

limn→+∞

f (sn) = limn→+∞

g(sn)⇒ f ( limn→+∞

sn) = g( limn→+∞

sn)⇒ f (x) = g(x).

Vậy với x ∈ R bất kì ta có f (x) = g(x). Hay f (x) = g(x), ∀x ∈ R (ĐPCM) .

Nhận xét: Trong mệnh đề trên ta có thể thay giả thiết f (x) = g(x), ∀x ∈ Q bằng giảthiết f (x) = g(x), ∀x ∈ A, trong đó A là tập hợp trù mật trong R bất kì. Với định nghĩavề tập hợp trù mật như sau:

Định nghĩa 1.5. Tập A ∈ R được gọi là tập trù mật trong R nếu và chỉ nếu

∀x,y ∈ R, x < y thì đều tồn tại a ∈ A sao cho x < a < y.

Ví dụ 1.1.

1. Q là tập trù mật trong R.

2. Giả sử 2≤ p ∈ N. Tập A = {mpn |m ∈ Z,n ∈ N} trù mật trong R.

2

Page 8: Đ—I H¯C QU¨C GIA HÀ N¸I TRƯ˝NG Đ—I H¯C KHOA H¯C TÜ NHIÊN KHOA … · 2016-05-09 · Đ—I H¯C QU¨C GIA HÀ N¸I TRƯ˝NG Đ—I H¯C KHOA H¯C TÜ NHIÊN KHOA TOÁN

1.2 Hàm số chẵn, hàm số lẻ

Định nghĩa 1.6. Xét hàm số f (x) với tập xác định D( f ) ⊆ R và tập giả trị R( f ) ⊆ R.

Khi đó:

(i). f (x) được gọi là hàm số chẵn trên M, M ⊂ D( f ) nếu

∀x ∈M⇒−x ∈M và f (−x) = f (x), ∀x ∈M.

(ii). f (x) được gọi là hàm số lẻ trên M, M ⊂ D( f ) nếu

∀x ∈M⇒−x ∈M và f (−x) =− f (x), ∀x ∈M.

1.3 Hàm số tuần hoàn và phản tuần hoàn

Định nghĩa 1.7. Hàm số f (x) được gọi là hàm tuần hoàn (cộng tính) chu kì a, a > 0trên M, M ∈ D( f ) nếu với mọi x ∈M thì ta có x±a ∈M và

f (x+a) = f (x), ∀x ∈M.

Số thực T > 0 nhỏ nhất (nếu có) thỏa mãn f (x +T ) = f (x), ∀x ∈M được gọi là chu kì

cơ sỏ của hàm số tuần hoàn f (x).

Định nghĩa 1.8. Hàm số f (x) được gọi là phản tuần hoàn (cộng tính) chu kì b, b > 0trên M, M ∈ D( f ) nếu với mọi x ∈M thì ta có x±b ∈M và

f (x+b) =− f (x), ∀x ∈M.

Ví dụ 1.2. (IMO 1968) Cho số thực a. Giả sử hàm f : R→ R thỏa mãn

f (x+a) =12

+√

f (x)− [ f (x)]2, ∀x ∈ R.

Chứng minh rằng f (x) là hàm tuần hoàn. Lấy ví dụ hàm f trong trường hợp a = 1.

Giải. Giả sử f là hàm cần tìm. Ta thấy rằng12≤ f (x)≤ 1, ∀x ∈ R.

Đặt f (x)− 12

= g(x), ∀x ∈ R. Khi đó 0≤ g(x)≤ 12, ∀x ∈ R và ta có

g(x+a) =

√14− [g(x)]2, ∀x ∈ R.

Hay là [g(x+a)]2 =14− [g(x)]2. Suy ra

[g(x+2a)]2 =14− [g(x+a)]2 = [g(x)]2⇒ g(x+2a) = g(x), ∀x ∈ R.

Do đó f (x+2a) = f (x), ∀x ∈ R, hay f (x) là hàm tuần hoàn.

Với a = 1 dễ dàng kiểm chứng hàm f (x) =12|sin

π

2x|+ 1

2, ∀x ∈ R thỏa mãn bài toán.

3

Page 9: Đ—I H¯C QU¨C GIA HÀ N¸I TRƯ˝NG Đ—I H¯C KHOA H¯C TÜ NHIÊN KHOA … · 2016-05-09 · Đ—I H¯C QU¨C GIA HÀ N¸I TRƯ˝NG Đ—I H¯C KHOA H¯C TÜ NHIÊN KHOA TOÁN

1.4 Tính đơn điệu của hàm số

Định nghĩa 1.9. Giả sử hàm số f (x) xác định trên I ∈ D( f ), ở đây ta chỉ xét I là một

khoảng, nửa khoảng hay đoạn thực. Khi đó, hàm số f (x) được gọi là không giảm (hoặc

không tăng) trên I ∈D( f ) nếu với mọi a,b ∈ I thì f (a)≥ f (b) ⇐⇒ a≥ b (tương ứng

f (a)≥ f (b) ⇐⇒ a≤ b).

Định nghĩa 1.10. Hàm số f (x) được gọi là đồng biến (đơn điệu tăng) trên I ∈D( f ) nếu

với mọi a,b ∈ I ta có

f (a) > f (b) ⇐⇒ a > b.

Định nghĩa 1.11. Hàm số f (x) được gọi là nghịch biến (đơn điệu giảm) trên I ∈ D( f )nếu với mọi a,b ∈ I ta có

f (a) > f (b) ⇐⇒ a < b.

1.5 Tính chất ánh xạ của hàm số

Giả sử /0 6= X ∈ R. Xét hàm số f : X → R, ta có các định nghĩa sau:

Định nghĩa 1.12. Hàm số f (x) được gọi là đơn ánh trên X nếu với mọi a,b ∈ X thì

f (a) = f (b) ⇐⇒ a = b.

Định nghĩa 1.13. Hàm số f (x) được gọi là toàn ánh từ X vào Y nếu với mọi y ∈ Y thì

tồn tại x ∈ X thỏa mãn f (x) = y.

Định nghĩa 1.14. Hàm số f (x) được gọi là song ánh từ X vào Y nếu nó vừa là đơn ánh

trên X vừa là toàn ánh từ X vào Y .

Định nghĩa 1.15. Giả sử f : X → Y là một song ánh. Khi đó, ta có thể định nghĩa hàm

số f−1 : Y → X như sau: với mỗi y ∈ Y thì f−1(y) = x khi và chỉ khi x là phần tử duy

nhất của X thỏa mãn f (x) = y. Ta gọi f−1 là hàm số ngược của f . Có thể thấy rằng f−1

là song ánh từ Y vào X .

4

Page 10: Đ—I H¯C QU¨C GIA HÀ N¸I TRƯ˝NG Đ—I H¯C KHOA H¯C TÜ NHIÊN KHOA … · 2016-05-09 · Đ—I H¯C QU¨C GIA HÀ N¸I TRƯ˝NG Đ—I H¯C KHOA H¯C TÜ NHIÊN KHOA TOÁN

Chương 2

Một số phương trình hàm cơ bản

2.1 Phương trình hàm Cauchy

Bài toán 2.1.1. (PTH Cauchy) Tìm tất cả các hàm số f (x) liên tục trên R thỏa mãn

f (x+ y) = f (x)+ f (y), ∀x,y ∈ R (2.1)

LỜI GIẢI.Với mọi n∈N∗, từ (2.1) ta suy ra f (x1 +x2 + ...+xn) = f (x1)+ f (x2)+ ...+ f (xn), trongđó x1,x2, ...,xn ∈ R tùy ý. Lấy x1 = x2 = ... = xn = x ta được

f (nx) = n f (x),∀x ∈ R. (i)

Đặc biệt, khi ta lấy x = 1 thì ta có f (n) = n f (1),∀n ∈ N∗. Trong (i), thay x =1n, ta có

f (1) = n f (1n) hay f (

1n) =

1n

f (1),∀n ∈ N∗.

Từ đó, với mọi m,n ∈ N∗ ta có

f (mn

) = m f (1n) =

mn

f (1).

Điều này có nghĩaf (x) = x f (1),∀x ∈Q+. (ii)

Trong (2.1) thay x = y = 0, ta có 2 f (0) = f (0) suy ra f (0) = 0. Khi đó, thay y =−x tacó 0 = f (0) = f (x)+ f (−x) nên f (−x) = − f (x) hay f (x) là hàm lẻ. Do đó từ (ii) dẫnđến f (x) = x f (1), ∀x ∈Q.

Nhưng f (x) và x f (1) là hai hàm liên tục trên R, nên theo mệnh đề 1.1 ta suy raf (x) = x f (1),∀x ∈ R. Đặt f (1) = a, thế thì f (x) = ax, hàm này thỏa mãn bài toán.Vậy nghiệm của bài toán PTH Cauchy là f (x) = ax,∀x ∈ R, với a ∈ R tùy ý.

Nhận xét:

5

Page 11: Đ—I H¯C QU¨C GIA HÀ N¸I TRƯ˝NG Đ—I H¯C KHOA H¯C TÜ NHIÊN KHOA … · 2016-05-09 · Đ—I H¯C QU¨C GIA HÀ N¸I TRƯ˝NG Đ—I H¯C KHOA H¯C TÜ NHIÊN KHOA TOÁN

1. Với điều kiện (2.1), ta chỉ cần giả thiết f (x) liên tục tại một điểm x0 ∈ R cho trước,khi đó f (x) sẽ liên tục trên R.Thật vậy, theo giả thiết thì lim

x→x0f (x) = f (x0). Với mỗi x1 ∈ R ta có

f (x) = f (x− x1 + x0)+ f (x1)− f (x0),∀x ∈ R

Từ đó suy ra limx→x1

f (x) = limx→x1{ f (x− x1 + x0)+ f (x1)− f (x0)}

= limx→x1{ f (x− x1 + x0)}+ f (x1)− f (x0) = f (x0)+ f (x1)− f (x0) = f (x1).

Do x1 ∈ R lấy bất kì nên f liên tục trên R.

2. Từ lời giải ta nhận thấy rằng nếu thiếu giả thiết hàm f (x) liên tục thì hàm f (x) chỉthỏa mãn (2.1) là f (x) = ax,∀x ∈Q, trong đó a tùy ý.

3. Từ bài toán PTH Cauchy ta có thể thấy rằng: hàm f (x) liên tục trên R, thỏa mãn

f (x1 + x2 + ...+ xn) = f (x1)+ f (x2)+ ...+ f (xn), ∀x1,x2, ...,xn ∈ R

vẫn chỉ là hàm f (x) = ax, ∀x ∈ R, với a ∈ R bất kì.

4. Kết quả của bài toán PTH Cauchy sẽ không thay đổi nếu ta thay R bằng [0, +∞)hoặc (−∞, 0] (theo [1]) .

Các hàm f thỏa mãn tính chất (2.1) được gọi là hàm cộng tính, hay thỏa mãn PTHCauchy (theo một số tài liệu). Để có thể xác định hoàn toàn hàm cộng tính f trên R, tacó thể thay giả thiết f liên tục trên R hay chỉ tại một điểm, bằng một trong các giả thiết:f là hàm đơn điệu trên R; f (x)≥ 0,∀x≥ 0, hay f bị chặn trên một đoạn nào đó,...

Vì tính quan trọng của lớp bài toán PTH Cauchy, ta sẽ đi tìm hiểu các bài toán này.

Bài toán 2.1.2. Xác định hàm số f (x) đơn điệu trên R và thỏa mãn phương trình (2.1).

LỜI GIẢI. Ta đã biết f (x) thỏa mãn (2.1) thì f (r) = ar, ∀r ∈ R, với a = f (1) ∈ R tùy ý.Ta sẽ chỉ ra rằng nếu f đơn điệu thì f (x) = ax,∀x ∈R. Ta đi chứng minh cho trường hợpf không giảm, trường hợp f không tăng tương tự.Giả sử f không giảm trên R. Khi đó, a = f (1)≥ f (0) = 0.

Với mỗi x ∈ R bất kì, xét hai dãy số hữu tỉ sn giảm và qn tăng cùng có giới hạn là x. Khiđó f (sn) = asn và f (qn) = aqn, ∀n ∈ N. Ngoài ra, f không giảm trên R, nên

asn ≥ f (sn)≥ f (x)≥ f (qn) = aqn, ∀n ∈ N.

6

Page 12: Đ—I H¯C QU¨C GIA HÀ N¸I TRƯ˝NG Đ—I H¯C KHOA H¯C TÜ NHIÊN KHOA … · 2016-05-09 · Đ—I H¯C QU¨C GIA HÀ N¸I TRƯ˝NG Đ—I H¯C KHOA H¯C TÜ NHIÊN KHOA TOÁN

Lấy giới hạn hai vế khi n→+∞ ta có

limn→+∞

asn ≥ f (x)≥ limn→+∞

aqn⇒ ax≥ f (x)≥ ax.

Vậy f (x) = ax, nhưng x ∈ R bất kì nên f (x) = ax, ∀x ∈ R (ĐPCM) .

Nhận xét: Tuy từ giả thiết f đơn điệu trên R và thỏa mãn (2.1), ta cũng có thể suy raf liên tục tại x = 0, từ đó suy ra f (x) = x f (1),∀x ∈ R. Nhưng cách làm trên khá ngắngọn và rõ ràng độc lập hơn là nếu ta qui về tính liên tục của f . Ngoài ra, đây cũng là kếtquả nền tảng của các bài toán về lớp PTH vừa cộng tính vừa đơn điệu.- Nếu thay giả thiết f đơn điệu bởi f (x) ≥ 0,∀x ≥ 0, kết hợp f thỏa mãn (2.1) thì tasuy ra f là hàm không giảm trên R, do đó f (x) = ax, ∀x ∈ R, với a ≥ 0. Đặc biệt,nếu f (x2n) = [ f (x)]2n, n ∈ N∗ thì ta sẽ suy ra được f (x) ≡ 0 hoặc f (x) = x,∀x ∈ R.

Còn trường hợp f (x) ≤ 0, ∀x ≥ 0 thì ta sẽ suy ra hàm f không tăng trên R, và từ đóf (x) = ax, ∀x ∈ R, với a≤ 0.

Bài toán 2.1.3. Tìm tất cả các hàm f (x) xác định trên R, thỏa mãn (2.1) và bị chặn trên

đoạn [c,d] với c < d bất kì.

LỜI GIẢI. Giả sử f là hàm thỏa mãn bài toán. Do f thỏa mãn (2.1) nên f (x) = ax,∀x∈Qvới a = f (1). Ta chỉ ra điều này cũng đúng khi x ∈ R, nghĩa là f (x) = ax,∀x ∈ R.

Thực vậy, lấy x ∈ R bất kì. Khi đó với mỗi n ∈ N tồn tại rn ∈ Q, phụ thuộc vào n và x,

sao cho nx−d ≤ rn ≤ nx− c, khi đó f (nx− rn) bị chặn do c≤ nx− rn ≤ d. Ta có

| f (nx− rn)|= | f (nx)+ f (−rn)|= |n f (x)−arn|=

= |n( f (x)−ax)+a(nx− rn)| ≥ n| f (x)−ax|− |a(nx− rn)|

Suy ra | f (nx− rn)|+ |a(nx− rn)| ≥ n| f (x)−ax|. Nhưng |a(nx− rn)| ≤max{|ac|, |ad|},và f (nx− rn) bị chặn với mọi n ∈ N. Nên n| f (x)−ax| cũng bị chặn với mọi n ∈ N.

Điều này chỉ xảy ra khi f (x)−ax = 0. Vậy f (x) = ax,∀x ∈ R (ĐPCM) .

Ở trên, ta đã tìm hiểu bài toán PTH Cauchy trong các trường hợp khác nhau củahàm số f (x). Tiếp theo ta sẽ trình bày một số dạng cơ bản khác của PTH Cauchy.

Bài toán 2.1.4. Xác định các hàm số f (x) liên tục trên R thỏa mãn điều kiện

f (x+ y) = f (x) f (y), ∀x,y ∈ R. (2.2)

7

Page 13: Đ—I H¯C QU¨C GIA HÀ N¸I TRƯ˝NG Đ—I H¯C KHOA H¯C TÜ NHIÊN KHOA … · 2016-05-09 · Đ—I H¯C QU¨C GIA HÀ N¸I TRƯ˝NG Đ—I H¯C KHOA H¯C TÜ NHIÊN KHOA TOÁN

LỜI GIẢI. Ta thấy rằng f (x)≡ 0 là nghiệm của bài toán (2.2).Xét trường hợp f (x) không đồng nhất bằng 0. Khi đó tồn tại x0 ∈ R mà f (x0) 6= 0.

Theo (2.2) thì f (x0) = f (x+(x0− x)) = f (x) f (x0− x) 6= 0, ∀x ∈ R.

Suy ra f (x) 6= 0,∀x ∈ R và f (x) = f (x2

+x2) = [ f (

x2)]2 > 0, ∀x ∈ R.

Đặt ln f (x) = g(x). Khi đó g(x) là hàm liên tục trên R và

g(x+ y) = ln f (x+ y) = ln [ f (x) f (y)] = ln f (x)+ ln f (y) = g(x)+g(y),∀x ∈ R.

Theo bài toán PTH Cauchy thì g(x) = bx,b ∈ R tùy ý.Hay f (x) = ebx = ax với a > 0 tùy ý. Thử lại, hàm này thỏa mãn bài toán.

Bài toán 2.1.5. Xác định các hàm số f (x) liên tục trên R\{0} thỏa mãn điều kiện

f (xy) = f (x) f (y), ∀x,y ∈ R\{0} (2.3)

LỜI GIẢI. Thay y = 1 vào phương trình (2.3) ta được f (x)(1− f (1)) = 0,∀x ∈ R.

- Nếu f (1) 6= 1 thì ta có f (x)≡ 0,∀x ∈ R. Nghiệm này thỏa mãn bài toán.- Xét f (1) = 1. Khi đó

f (1) = f (x1x) = f (x) f (

1x),∀x ∈ R\{0}

Vậy f (x) 6= 0,∀x ∈ R\{0}. Do đó f (x2) = f (x) f (x) = [ f (x)]2 > 0,∀x ∈ R\{0}.a) Xét x,y ∈ R+. Đặt x = eu,y = ev và f (et) = g(t). Khi đó g(t) liên tục trên R và

g(u+ v) = g(u)g(v),∀x,y ∈ R

Theo bài toán trên thì g(t) = at , ∀t ∈ R, a > 0 tùy ý, và do đó

f (x) = f (eu) = g(u) = au = alnx = xlna = xα ,∀x ∈ R+, trong đó α = lna.

b) Khi x,y ∈ R− thì xy ∈ R+. Với y = x, thì từ (2.3) và kết quả phần a), ta có

[ f (x)]2 = f (x2) = (x2)α ,∀x ∈ R−,α ∈ R xác định ở trên.

Do f (x) 6= 0,∀x 6= 0 và f (x) là hàm liên tục trên R−, nên

f (x) = |x|α , ∀x ∈ R− hoặc −|x|α , ∀x ∈ R−.

Kết hợp a) và b) và thử lại các kết quả ta có kết luận :Nghiệm của (2.3) là một trong các hàm số sau

1. f (x)≡ 0,∀x ∈ R\{0},

8

Page 14: Đ—I H¯C QU¨C GIA HÀ N¸I TRƯ˝NG Đ—I H¯C KHOA H¯C TÜ NHIÊN KHOA … · 2016-05-09 · Đ—I H¯C QU¨C GIA HÀ N¸I TRƯ˝NG Đ—I H¯C KHOA H¯C TÜ NHIÊN KHOA TOÁN

2. f (x) = |x|α ,∀x ∈ R\{0},α ∈ R tùy ý,

3. f (x) ={|x|α , ∀x ∈ R+,−|x|α , ∀x ∈ R−,α ∈ R tùy ý.

Bài toán 2.1.6. Xác định các hàm số f (x) liên tục trên R\{0} thỏa mãn điều kiện

f (xy) = f (x)+ f (y), ∀x,y ∈ R\{0} (2.4)

LỜI GIẢI.a) Trước hết xét x,y ∈ R+. Đặt x = eu,y = ev, f (et) = g(t). Khi đó (2.4) trở thành

g(u+ v) = g(u)+g(v),∀u,v ∈ R.

Ngoài ra, g(t) liên tục trên R. Nên theo bài toán PTH Cauchy ta có g(t) = at. Do đó

f (x) = a lnx,∀x ∈ R+, a ∈ R tùy ý.

b) Khi x,y ∈ R− thì xy ∈ R+. Với y = x, từ (2.4) và kết quả phần a) ta có

f (x) =12

f (x2) =12

a ln(x2) = a ln |x|,∀x ∈ R−, với a ∈ R xác định ở trên.

Như vậy, với mọi x ∈ R thì f (x) = a ln |x| với a ∈ R tùy ý, hàm này thỏa mãn bài toán.Vậy nghiệm của bài toán là f (x) = a ln |x|,∀x ∈ R\{0}, với a ∈ R tùy ý.

Bài toán 2.1.7. (PTH Pexider) Tìm tất cả các hàm số f (x), g(x), h(x) xác định và liên

tục trên R và thỏa mãn điều kiện

f (x+ y) = g(x)+h(y), ∀x,y ∈ R.

LỜI GIẢI. Thay y = 0 và đặt h(0) = c thì ta có

f (x) = g(x)+ c, ∀x ∈ R.

Còn thay x = 0 và đặt g(0) = b ta có

f (y) = h(y)+b, ∀y ∈ R.

Từ đó thay vào phương trình điều kiện ta thu được

f (x+ y) = f (x)+ f (y)−b− c, ∀x,y ∈ R.

Bằng cách đặt f (z)−b− c = k(z), ∀z ∈ R thì ta có

k(x+ y) = k(x)+ k(y), ∀x,y ∈ R.

9

Page 15: Đ—I H¯C QU¨C GIA HÀ N¸I TRƯ˝NG Đ—I H¯C KHOA H¯C TÜ NHIÊN KHOA … · 2016-05-09 · Đ—I H¯C QU¨C GIA HÀ N¸I TRƯ˝NG Đ—I H¯C KHOA H¯C TÜ NHIÊN KHOA TOÁN

Ngoài ra, ta cũng thấy rằng do f (x) liên tục nên k(x) liên tục. Do vậy, theo bài toán PTHCauchy thì ta có k(x) = ax, a ∈ R tùy ý. Suy ra với mọi x ∈ R thì

f (x) = ax+b+ c, g(x) = ax+b, h(x) = ax+ c, với a,b,c ∈ R tùy ý.

Thử lại, ta thấy rằng các hàm số này thỏa mãn bài toán.

Nhận xét: Ta thấy rằng có thể có thêm những bài toán "dạng PTH Pexider", ứng vớicác dạng cơ bản của PTH Cauchy, tuy nhiên điều kiện của các hàm có thể đòi hỏi nhiềuhơn. Ta lấy ví dụ đơn giản như: Tìm các hàm f (x),g(x),h(x) liên tục trên R thỏa mãn

f (x+ y) = g(x)h(y), ∀x,y ∈ R.

Ta đi xét một vài ví dụ đơn giản, áp dụng khá trực tiếp PTH Cauchy trong cáctrường hợp khác nhau của hàm số f (x).

Ví dụ 2.1. (Olympic SV 2010) Tìm hàm f : R→ R liên tục thỏa mãn f (1) = 2010 và

f (x+ y) = 2010x f (y)+2010y f (x), ∀x,y ∈ R.

LỜI GIẢI. Đặt 2010−x f (x) = g(x). Khi đó, g(x) liên tục trên R và

g(x+ y) = g(x)+g(y),∀x,y ∈ R.

Từ đây dẫn đến g(x) = ax, a ∈ R. Do đó, f (x) = ax.2010x, ∀x ∈ R. Mà f (1) = 2010suy ra a = 1. Nên f (x) = 2010xx, ∀x ∈ R. Thử lại, đây là nghiệm của bài toán.

Ví dụ 2.2. Tìm f : R→ R thỏa mãn các điều kiện sau

(i). f (x+ y) = f (x)+ f (y), ∀x,y ∈ R,

(ii). f (xy) = f (x) f (y), ∀x,y ∈ R.

LỜI GIẢI. Trong (ii) lấy y = x, ta có f (x2) = [ f (x)]2,∀x ∈ R. Do đó f (x) ≥ 0,∀x ≥ 0.

Trong (i) ta xét với y ≥ 0, ta có f (x + y) = f (x) + f (y) ≥ f (x),∀x ∈ R,y ≥ 0. Suy raf (x) đồng biến trên R. Như vậy, f cộng tính và đồng biến nên f (x) = ax,a≥ 0. Nhưngf (x2) = [ f (x)]2 nên a = 0 hoặc a = 1. Suy ra f (x) ≡ 0,∀x ∈ R hoặc f (x) = x,∀x ∈ R.

Thử lại, đây là hai nghiệm của bài toán.Nhận xét: Từ bài toán này nếu ta thay giả thiết (ii) bằng giả thiết f (x2n) = [ f (x)]2n hayf (xn) = xn, 2≤ n ∈ N∗ thì nghiệm bài toán vẫn là f (x)≡ 0,∀x ∈ R hoặc f (x) = x.

Ví dụ 2.3. Xác định tất cả các hàm số f (x) đồng biến trên R+ thỏa mãn điều kiện

f (xy) = f (x)+ f (y), ∀x,y > 0.

10

Page 16: Đ—I H¯C QU¨C GIA HÀ N¸I TRƯ˝NG Đ—I H¯C KHOA H¯C TÜ NHIÊN KHOA … · 2016-05-09 · Đ—I H¯C QU¨C GIA HÀ N¸I TRƯ˝NG Đ—I H¯C KHOA H¯C TÜ NHIÊN KHOA TOÁN

LỜI GIẢI. Với x,y > 0 ta có thể đặt x = eu, y = ev. Và đặt f (et) = g(t),∀t ∈ R. Khi đó,g(t) đồng biến trên R và g(u+v) = g(u)+g(v),∀u,v ∈R. Do đó g(x) = ax, a > 0. Nênf (x) = f (eu) = g(u) = au = a lnx,∀x > 0 với a > 0.

Ví dụ 2.4. Tìm hàm f : R→ R+ đồng biến thỏa mãn f (x+ y) = f (x) f (y),∀x,y ∈ R.

LỜI GIẢI. Vì f (x) > 0,∀x ∈ R nên ta có thể đặt g(x) = ln f (x),∀x ∈ R.

Do f (x) đồng biến nên g(x) cũng đồng biến, ngoài ra từ phương trình điều kiện ta có

ln f (x+ y) = ln f (x)+ ln f (y)⇒ g(x+ y) = g(x)+g(y), ∀x,y ∈ R.

Do đó g(x) = ax,a > 0. Suy ra f (x) = eax = cx, với c = ea > 0 tùy ý.

Ví dụ 2.5. Xác định hàm f : R+→ R thỏa mãn

(i). f (xy) = f (x) f (y), ∀x,y > 0,

(ii). limx→1

f (x) = 1.

LỜI GIẢI. Với mọi x > 0 ta có f (x) = f (√

x√

x) = [ f (√

x)]2 ≥ 0. Nếu tồn tại x0 > 0 màf (x0) = 0 thì

f (x) = f (x0xx0

) = f (x0) f (xx0

) = 0, ∀x > 0.

Điều này mâu thuẫn với (ii). Nên f (x) > 0, ∀x > 0. Từ (i) dễ suy ra f (1) = 1. Do đóf (x) liên tục tại x = 1. Ta chỉ ra f (x) liên tục trên R+. Thật vậy, với bất kì x0 > 0 ta có

limy→1{ f (yx0)− f (x0)}= lim

y→1{ f (x0) f (y)− f (x0)}= f (x0) lim

y→1{ f (y)−1}= 0.

Điều này chỉ ra f (x) liên tục tại x0 > 0 bất kì. Do đó f (x) liên tục trên R+.

Đặt g(x) = ln f (x), thì g(x) liên tục và

g(xy) = ln f (xy) = ln f (x) f (y) = ln f (x)+ ln f (y) = g(x)+g(y),∀x,y > 0.

Từ đây suy ra g(x) = a lnx, ∀x > 0⇒ f (x) = eg(x) = ea lnx = xa, ∀x > 0.

Thử lại, ta đi đến kết luận nghiệm của bài toán là f (x) = xa,∀x > 0, với a ∈ R bất kì.

2.2 Phương trình hàm Jensen

Bài toán 2.2.1. (PTH Jensen) Tìm hàm f (x) xác định và liên tục trên R thỏa mãn

f (x+ y

2) =

f (x)+ f (y)2

, ∀x,y ∈ R. (2.5)

11

Page 17: Đ—I H¯C QU¨C GIA HÀ N¸I TRƯ˝NG Đ—I H¯C KHOA H¯C TÜ NHIÊN KHOA … · 2016-05-09 · Đ—I H¯C QU¨C GIA HÀ N¸I TRƯ˝NG Đ—I H¯C KHOA H¯C TÜ NHIÊN KHOA TOÁN

LỜI GIẢI. Đặt f (x)− f (0) = g(x), ta có g(x) liên tục trên R với g(0) = 0 và

g(x+ y

2) =

g(x)+g(y)2

,∀x,y ∈ R.

Lần lượt cho y = 0 và x = 0, ta suy ra g(x2) =

g(x)2

, g(y2) =

g(y)2

, ∀x,y ∈ R.

Do g(x+ y

2) =

g(x)+g(y)2

,∀x,y ∈ R suy ra g(x+ y) = g(x)+g(y),∀x,y ∈ R.

Mà g(x) là hàm liên tục nên theo bài toán PTH Cauchy, ta có g(x) = ax, a ∈ R.Suy ra f (x) = ax+b. Thử lại, ta đi đến kết luận nghiệm của bài toán PTH Jensen là

f (x) = ax+b, ∀x ∈ R, với a,b ∈ R tùy ý.

Nhận xét: Ta có một hướng mở rộng của bài toán trên như sau: Tìm hàm f (x) liên tục

thỏa mãn f (x1 + x2 + . . .+ xn

n) =

f (x1)+ f (x2)+ . . .+ f (xn)n

, ∀x1,x2, . . . ,xn ∈ R.

Nghiệm của bài toán này vẫn là nghiệm của bài toán PTH Jensen.

Bây giờ, ta sẽ thử thay đổi hệ số của các biến trong bài toán PHT Jensen, và đitìm nghiệm của bài toán khi đó. Cụ thể ta có bài toán sau đây:

Bài toán 2.2.2. Cho a,b ∈ R\{0}. Tìm tất cả các hàm f (x) liên tục trên R thỏa mãn

f (ax+by) = a f (x)+b f (y), ∀x,y ∈ R. (2.6)

LỜI GIẢI. Thay x = y = 0, ta có f (0)[(a+b)−1] = 0. Xét các trường hợp sau:*) Nếu a+b 6= 1 thì f (0) = 0. Khi đó, trong (2.6) lần lượt thay y = 0,x = 0 ta có

f (ax) = a f (x), f (by) = b f (y), ∀x,y ∈ R. (i)

Từ (2.6) và (i) suy ra (2.6) ⇐⇒ f (ax+by) = f (ax)+ f (by), ∀x,y ∈ R.

Từ đây do a,b 6= 0 nên ta có f (x+ y) = f (x)+ f (y), ∀x,y ∈ R. Mặt khác, f (x) liên tụctrên R nên theo bài toán PTH Cauchy ta có f (x) = cx, ∀x ∈ R, với c ∈ R bất kì.

*) Nếu a+b = 1 thì f (0) nhận giá trị tùy ý. Khi đó

(2.6) ⇐⇒ f (ax+by)− f (0) = a[ f (x)− f (0)]+b[ f (y)− f (0)], ∀x,y ∈ R,

hay g(ax+by) = g(ax)+g(by), ∀x,y ∈ R, trong đó g(x) = f (x)− f (0), g(0) = 0.

Khi đó, tương tự phần trên ta có g(x) = cx. Suy ra f (x) = cx+d; c,d ∈ R tùy ý.Kết luận:

- Nếu a+b 6= 1 thì f (x) = cx, ∀x ∈ R, với c ∈ R tùy ý.- Nếu a+b = 1 thì f (x) = cx+d, ∀x ∈ R, với c,d ∈ R tùy ý.

12

Page 18: Đ—I H¯C QU¨C GIA HÀ N¸I TRƯ˝NG Đ—I H¯C KHOA H¯C TÜ NHIÊN KHOA … · 2016-05-09 · Đ—I H¯C QU¨C GIA HÀ N¸I TRƯ˝NG Đ—I H¯C KHOA H¯C TÜ NHIÊN KHOA TOÁN

Nhận xét: Ta hoàn toàn có thể giải tương tự bài toán mở rộng sau:Với n≥ 2, n ∈ N và a1,a2, ...,an ∈ R\{0}. Tìm hàm f : R→ R liên tục thỏa mãn

f (a1x1 +a2x2 + ...+anxn) = a1 f (x1)+a2 f (x2)+ ...+an f (xn), ∀x1,x2, ...,xn ∈ R.

- Từ bài toán trên, ta hoàn toàn giải được bài toán sau: Với a,b 6= 0, tìm hàmf : R+→ R+ liên tục trên R+ thỏa mãn f (xayb) = [ f (x)]a[ f (yb)], ∀x,y > 0.

Một trong những ứng dụng quan trọng nhất của bài toán PTH Jensen, đó là giúpgiải quyết bài toán PTH chuyển đổi giữa các đại lượng trung bình. Bạn đọc quan tâm cóthể dễ dàng tìm hiểu chi tiết, tường tận trong tài liệu [3], nhờ sự trình bày rất rõ ràng vàkhoa học của GS.TSKH Nguyễn Văn Mậu.

Để kết thúc mục này, xin đưa ra một mở rộng, theo hướng tuyến tính, đối với bàitoán PTH Cauchy và PTH Jensen. PTH này thường được gọi là "PTH tuyến tính". Cụthể, ta có bài toán như sau:

Bài toán 2.2.3. Với a,b,c, p,q,r ∈ R, trong đó a,b 6= 0. Tìm hàm số f (x) xác định và

liên tục trên R thỏa mãn

f (ax+by+ c) = p f (x)+q f (y)+ r, ∀x,y ∈ R. (2.7)

LỜI GIẢI. Do a,b 6= 0 nên từ (2.7), bằng phép thế thích hợp, không khó để ta thấy rằng:

f (0) = p f (−ca

)+q f (0)+ r,

f (u) = p f (u− c

a)+q f (0)+ r, ∀u ∈ R,

f (v) = p f (−ca

)+q f (vb)+ r, ∀v ∈ R,

f (u+ v) = p f (u− c

a)+q f (

vb)+ r, ∀u,v ∈ R.

Từ bốn phương trình trên ta thấy rằng f (u+ v)+ f (0) = f (u)+ f (v), ∀u,v ∈ R.

Bằng phép đặt f (x)− f (0) = g(x), ∀x ∈ R, ta có g(x) liên tục trên R và

g(u+ v) = g(u)+g(v), ∀u,v ∈ R.

Theo bài toán PTH Cauchy thì g(x) = sx với s∈R tùy ý. Suy ra f (x) = sx+ t,∀x ∈R,

ở đây t = f (0). Thay lại vào (2.7) ta có

s(ax+by+ c)+ t = (spx+ pt)+(sqy+qt)+ r

13

Page 19: Đ—I H¯C QU¨C GIA HÀ N¸I TRƯ˝NG Đ—I H¯C KHOA H¯C TÜ NHIÊN KHOA … · 2016-05-09 · Đ—I H¯C QU¨C GIA HÀ N¸I TRƯ˝NG Đ—I H¯C KHOA H¯C TÜ NHIÊN KHOA TOÁN

⇐⇒ s(a− p)x+ s(b−q)y+(sc+ t− pt−qt− r) = 0, ∀x,y ∈ R.

*) Xét trường hợp: hoặc p 6= a hoặc q 6= b. Khi đó, suy ra s = 0 và t− pt−qt = r.

Nên trong trường hợp này:- Nếu p+q = 1 và r 6= 0 thì (2.7) vô nghiệm.- Nếu p + q = 1 và r = 0 thì s = 0 và t ∈ R tùy ý. Như vậy f (x) = t, ∀x ∈ R, vớit ∈ R tùy ý. Thử lại, hàm này thỏa mãn bài toán.

*) Xét trường hợp p = a và q = b. Khi đó, hai số s, t phải thỏa mãn

sc+ t−at−bt− r = 0.

- Nếu c = a+b−1 = r = 0 thì s, t ∈R tùy ý. Đây là trường hợp mở rộng bài toán PTHJensen đã xét ở trên. Và f (x) = sx+ t, ∀x ∈ R, với s, t ∈ R tùy ý.- Nếu c = a+b−1 = 0, r 6= 0 thì (2.7) vô nghiệm.- Nếu c 6= 0, a+b = 1 thì t ∈ R tùy ý, s =

rc. Và f (x) =

rc

x+ t, ∀x ∈ R.

- Nếu c 6= 0, a+b 6= 1 thì s ∈ R tùy ý, t =sc− r

a+b−1. Nghiệm của bài toán là

f (x) = sx+sc− r

a+b−1, ∀x ∈ R, với s ∈ R tùy ý.

Đến đây, ta đã giải xong bài toán với tất cả các trường hợp.

Nhận xét: Ta thấy rằng, đối với bài toán tổng quát này thì bài toán PTH Cauchy vẫnđóng vai trò quan trọng nhất. Trường hợp: a = 0 hoặc b = 0 bài toán sẽ chuyển sanghướng khác.

2.3 Vận dụng PHT cơ bản vào giải toán

Trong phần này, ta quan tâm nhiều đến các bài toán vận dụng các kết quả của PTHCauchy trong lớp hàm liên tục, đơn điệu và một số áp dụng các kết quả nhận xét; đồngthời ta cũng xét đến một số bài toán tương tự cùng với mở rộng của nó.

Bài toán 2.3.1. (IMO 1979, Shortlist) Cho hàm f : R→R, thỏa mãn với hai số thực bất

kì x,y ta có f (xy+ x+ y) = f (xy)+ f (x)+ f (y). Chứng minh rằng

f (x+ y) = f (x)+ f (y), ∀x,y ∈ R.

LỜI GIẢI. Cho x = y = 0 ta có f (0) = 0. Cho y = −1 ta suy ra f (−x) = − f (x). Lấyy = 1 ta có f (2x+1) = 2 f (x)+1. Vì vậy

f (2(uv+u+ v)+1) = 2 f (uv+u+ v)+1 = 2 f (uv)+2 f (u)+2 f (v)+1, ∀u,v ∈ R.

14

Page 20: Đ—I H¯C QU¨C GIA HÀ N¸I TRƯ˝NG Đ—I H¯C KHOA H¯C TÜ NHIÊN KHOA … · 2016-05-09 · Đ—I H¯C QU¨C GIA HÀ N¸I TRƯ˝NG Đ—I H¯C KHOA H¯C TÜ NHIÊN KHOA TOÁN

Mặt khác, ta lại có

f (2(uv+u+ v)+1) = f (u(2v+1)+u+(2v+1)) =

= f (u(2v+1))+ f (u)+ f (2v+1) = f (2uv+u)+ f (u)+2 f (v)+1, ∀u,v ∈ R.

Do đó 2 f (uv)+2 f (u)+2 f (v)+1 = f (2uv+u)+ f (u)+2 f (v)+1,∀u,v ∈ R. Suy ra

f (2uv+u) = 2 f (uv)+ f (u), ∀u,v ∈ R.

Ở đây, cho v =−12

ta có 0 = f (0) = 2 f (−u2

)+ f (u), ∀u ∈ R. Do f là hàm lẻ nên

f (u) =−2 f (−u/2) ⇒ f (u) = 2 f (u/2) ⇒ f (2u) = 2 f (u), ∀u ∈ R.

Từ đó suy ra f (2uv+u) = 2 f (uv)+ f (u) = f (2uv)+ f (u), ∀u,v ∈ R. Hay là

f (x+ y) = f (x)+ f (y), ∀x,y ∈ R.

Đến đây, bài toán được chứng minh !Nhận xét: Ta có thể dễ thấy rằng nếu hàm f thỏa mãn f (x+y) = f (x)+ f (y), ∀x,y∈R,

thì nó cũng thỏa mãn f (xy + x + y) = f (xy)+ f (x)+ f (y), ∀x,y ∈ R. Như vậy ta có 2khẳng định sau tương đương: " f là hàm cộng tính" và "hàm f thỏa mãn f (xy+ x+ y) =f (xy)+ f (x)+ f (y), ∀x,y ∈ R."

Bài toán 2.3.2. (THTT - T7/2010) Xác định hàm số liên tục f : R→ R thỏa mãn

f (x+ f (y)) = 2y+ f (x), ∀x,y ∈ R.

LỜI GIẢI. Cho x = 0 ta có f ( f (y)) = 2y+ f (0). Từ đây dễ thấy f là đơn ánh trên R. Khiđó, với x = y = 0 ta có f ( f (0)) = f (0) nên f (0) = 0. Do đó f ( f (y)) = 2y, ∀y ∈ R và2 f (y) = f ( f ( f (y))) = f (2y). Từ đó thay y bởi f (y) trong phương trình điều kiện ta được

f (x+2y) = f (x+ f ( f (y))) = 2 f (y)+ f (x) = f (2y)+ f (x), ∀x,y ∈ R.

Hay là f (x+y) = f (x)+ f (y), ∀x,y∈R. Lại có f liên tục trên R nên f (x) = cx, ∀x∈R.

Kết hợp f ( f (y)) = 2y cho ta c =±√

2. Thử lại, ta đi đến kết luận

f (x) =√

2 x, ∀x ∈ R hoặc f (x) =−√

2 x, ∀x ∈ R.

Nhận xét: Ta có thể thay 2 bởi hằng số k2 ∈ R bất kì. Khi đó, nghiệm của bài toán làf (x) = kx, ∀x∈R hoặc f (x) =−kx, ∀x∈R. Tuy nhiên, nếu thay 2 bởi một hằng số c < 0thì sẽ không tồn tại hàm f . Vì khi đó ta cũng chỉ ra f cộng tính và f (x) = ax, a ∈ Q,

nhưng khi thay vào phương trình điều kiện ta sẽ thấy không tồn tại a. Chúng ta có thểhình dung qua bài toán dưới đây.

15

Page 21: Đ—I H¯C QU¨C GIA HÀ N¸I TRƯ˝NG Đ—I H¯C KHOA H¯C TÜ NHIÊN KHOA … · 2016-05-09 · Đ—I H¯C QU¨C GIA HÀ N¸I TRƯ˝NG Đ—I H¯C KHOA H¯C TÜ NHIÊN KHOA TOÁN

Bài toán 2.3.3. (Autriche - Pologne 1997) Chứng minh rằng không tồn tại hàm f : Z→Zthỏa mãn

f (x+ f (y)) = f (x)− y, ∀x,y ∈ Z.

LỜI GIẢI. Giả sử tồn tại hàm f thỏa mãn bài toán.Cho x = 0 ta có f ( f (y)) = f (0)−y, ∀y ∈ Z, (1) . Từ đây, dễ thấy f là song ánh trên Z.

Thật vậy, trước tiên f là toàn ánh vì ∀y ∈ Z thì f ( f ( f (0)−y)) = y. Giả sử f (y1) = f (y2)thì f (0)−y1 = f ( f (y1)) = f ( f (y2)) = f (0)−y2 suy ra y1 = y2, do đó f là đơn ánh. Vậyf là song ánh trên Z. Từ (1) lấy y = 0 ta có f ( f (0)) = f (0), do f đơn ánh nên f (0) = 0.

Suy ra f ( f (y)) =−y, ∀y ∈ Z. Do f toàn ánh trên Z nên với mọi y ∈ Z tồn tại a ∈ Z mày = f (a), khi đó f (y) = f ( f (a)) =−a. Từ đây, với mọi x,y ∈ Z ta có

f (x+ y) = f (x+ f (a)) = f (x)−a = f (x)+ f (y).

Theo bài toán PTH Cauchy ta suy ra f (x) = cx, ∀x ∈Z. Ta lại có f ( f (x)) =−x, ∀x ∈Znên c2x = −x, ∀x ∈ Z suy ra c2 = −1, vô lý. Vậy giả sử tồn tại f thỏa mãn bài toán làsai. Hay nói cách khác không tồn tại f thỏa mãn bài toán (ĐPCM) .

Bài toán 2.3.4. (VietNam 2006 - Bảng B) Tìm hàm f : R→ R liên tục thỏa mãn

f (x− y) f (y− z) f (z− x)+8 = 0, ∀x,y,z ∈ R.

LỜI GIẢI.Cho x = t,y = 0,z =−t ta có f (t) f (t) f (−2t) =−8 suy ra f (−2t) =

−8[ f (t)]2

< 0, ∀t ∈R.

Nên có thể đặt lnf (x)−2

= g(x), ∀x ∈ R, khi đó f (x) = −2eg(x) và g liên tục trên R. Từ

phương trình điều kiện ta suy ra

g(x− y)+g(y− z)+g(z− x) = 0, ∀x,y,z ∈ R.

Ở đây, cho x = y = z = 0 ta có g(0) = 0. Cho y = z = 0 suy ra g(x) = g(−x), ∀x ∈R. Dođó g(x−y)+g(y− z) =−g(z−x) = g(x− z) = g((x−y)+(y− z)), ∀x,y,z ∈R. Hay là

g(u)+g(v) = g(u+ v), ∀u,v ∈ R.

Ngoài ra g liên tục nên g(x) = ax, ∀x ∈ R. Khi đó f (x) = −2eax = −2(ea)x, ∀x ∈ R,

với a ∈ R tùy ý. Hay f (x) =−2cx, ∀x ∈ R, với c > 0 tùy ý.

Bài toán 2.3.5. (ĐH Vinh 2010) Tìm tất cả các hàm liên tục f : R+→ R+ thỏa mãn

f ( f (xy)− xy)+ x f (y)+ y f (x) = f (xy)+ f (x) f (y), ∀x,y > 0.

16

Page 22: Đ—I H¯C QU¨C GIA HÀ N¸I TRƯ˝NG Đ—I H¯C KHOA H¯C TÜ NHIÊN KHOA … · 2016-05-09 · Đ—I H¯C QU¨C GIA HÀ N¸I TRƯ˝NG Đ—I H¯C KHOA H¯C TÜ NHIÊN KHOA TOÁN

LỜI GIẢI. Chọn y = 1 ta được f ( f (x)− x)+ x f (1)+ f (x) = f (x)+ f (x) f (1), ∀x > 0.Suy ra

f ( f (x)− x) = f (1)[ f (x)− x], ∀x > 0. (i)

Phương trình điều kiện có thể viết lại thành

f (1)[ f (xy)− xy] = f (xy)− xy+[ f (x)− x][ f (y)− y], ∀x,y > 0.

Đặt f (x)− x = g(x), ∀x > 0 ta được f (1)g(xy) = g(xy)+g(x)g(y), ∀x,y > 0 hay là

g(1)g(xy) = g(x)g(y), ∀x,y > 0. (ii)

Do f : R+→ R+ nên từ (i) ta suy ra f (x) > x, ∀x > 0 và như vậy g(x) > 0, ∀x > 0.

Đặt h(x) =g(x)g(1)

. Khi đó h(x) liên tục trên R+ và

h(xy) = h(x)h(y), ∀x,y > 0.

Từ đây, theo dạng cơ bản của PTH Cauchy ta suy ra

h(x) = xa⇒ g(x) = cxa, với c > 0,a tùy ý.

Cuối cùng f (x) = x+g(x) = x+ cxa, ∀x ∈ R+ với c > 0,a ∈ R tùy ý.Nhận xét: Hai bài toán khá căn bản. Điểm mấu chốt là đưa phương trình điều kiện vềdạng cơ bản của PTH Cauchy. Bạn đọc có thể tham khảo thêm trong [3], cuốn Phươngtrình hàm của GS - TSKH. Nguyễn Văn Mậu.

Bài toán 2.3.6. (Italy 1999)

a) Xác định hàm đơn điệu (thực sự) f : R→ R thỏa mãn

f (x+ f (y)) = f (x)+ y, ∀x,y ∈ R. (a)

b) Chứng minh rằng, với 1 < n ∈ N, không tồn tại hàm đơn điệu f : R→ R thỏa mãn

f (x+ f (y)) = f (x)+ yn, ∀x,y ∈ R. (b)

LỜI GIẢI.a) Do f đơn điệu (thực sự) nên f đơn ánh. Lấy x = y = 0 ta có f ( f (0)) = f (0), suy ra

f (0) = 0. Lấy x = 0 ta được f ( f (y)) = y. Từ (a) thay y bởi f (y) ta có f (x + f ( f (y))) =f (x)+ f (y), suy ra f (x+y) = f (x)+ f (y), ∀x,y ∈R. Vậy ta có f là hàm cộng tính và f

đơn điệu, từ đó f (x) = cx, c = f (1),∀x ∈ R.Thay lại vào phương trình (a) ta suy ra c2 = 1 ⇐⇒ c = ±1. Khi đó, dễ thấy hai hàmf (x) = x, ∀x ∈ R và f (x) =−x,∀x ∈ R chính là nghiệm của bài toán.

17

Page 23: Đ—I H¯C QU¨C GIA HÀ N¸I TRƯ˝NG Đ—I H¯C KHOA H¯C TÜ NHIÊN KHOA … · 2016-05-09 · Đ—I H¯C QU¨C GIA HÀ N¸I TRƯ˝NG Đ—I H¯C KHOA H¯C TÜ NHIÊN KHOA TOÁN

b) Tương tự như trên, ta cũng có f đơn ánh, f (0) = 0 và f ( f (y)) = yn,∀y∈R. Bâygiờ, ta xét 2 trường hợp với n:- Nếu n chẵn. Khi đó, f ( f (−1)) = 1 = f ( f (1)), mâu thuẫn với tính đơn ánh của f . Nhưvậy, với n chẵn thì không tồn tại f thỏa mãn bài toán.- Nếu n lẻ thì với mọi y tồn tại z sao cho zn = f (y), suy ra f ( f (z)) = zn = f (y), do tínhđơn ánh của f nên f (z) = y. Từ đó với mọi x,y ta có

f (x)+ f (y) = f (x)+ zn = f (x+ f (z)) = f (x+ y).

Như vậy, ta lại có f cộng tính đồng thời f đơn điệu nên f (x) = cx. Suy ra xn = f ( f (x)) =f (cx) = c2x,∀x ∈R. Thay x = 1,2 ta có c2 = 1, 2c2 = 2n suy ra 2n = 2 ⇐⇒ n = 1, mâuthuẫn với giả thiết. Nên trường hợp n lẻ cũng không có nghiệm. Vậy ta có ĐPCM !

Ta xét một số mở rộng của bài toán Italy 1999 như sau.

Bài toán 2.3.7. Tìm hàm f : R→ R đơn điệu trên R thỏa mãn

f (x2n+1 + f (y)) = y+[ f (x)]2n+1, ∀x,y ∈ R, (*)

ở đây, n là số tự nhiên bất kì.

LỜI GIẢI. Do f đơn điệu nên f đơn ánh. Lấy x = 0 ta có f ( f (y)) = y+[ f (0)]2n+1 (1) ,

nên dễ thấy f cũng là toàn ánh. Vậy f là song ánh. Khi đó tồn tại duy nhất a mà f (a) = 0.

Đặt f (0) = b, khi đó trong (1) thay y = a ta có b = f (0) = a + b2n+1. Trong (*) lấyx = a,y = 0 ta có f (a2n+1 +b) = 0 = f (a), do f đơn ánh nên a = a2n+1 +b. Như vậy, tacó hệ

a = a2n+1 +b và b = a+b2n+1.

Dẫn đến a2n+1 +b2n+1 = 0 suy ra a =−b và 2a = a2n+1, 2b = b2n+1.

Nếu a = 0 hoặc b = 0 hoặc a = b ta dễ dàng suy ra f (0) = 0.

Xét trường hợp khác với các trường hợp trên.Do f là toàn ánh trên R nên tồn tại c sao cho f (c) = a. Khi đó, trong (1) thay y = c ta có

0 = f (a) = f ( f (c)) = c+b2n+1.

Từ (1) lấy y = 0 ta có f (b) = f ( f (0)) = b2n+1. Trong (*) thay x = c, y = b ta được

f (c2n+1 +b2n+1) = b+[ f (c)]2n+1 = b+a2n+1 = a = f (c),

do f đơn ánh nên c2n+1 +b2n+1 = c. Như vậy ta lại có hệ mới

c+b2n+1 = 0 và c2n+1 +b2n+1 = c.

18

Page 24: Đ—I H¯C QU¨C GIA HÀ N¸I TRƯ˝NG Đ—I H¯C KHOA H¯C TÜ NHIÊN KHOA … · 2016-05-09 · Đ—I H¯C QU¨C GIA HÀ N¸I TRƯ˝NG Đ—I H¯C KHOA H¯C TÜ NHIÊN KHOA TOÁN

Suy ra c2n+1 = 2c. Nếu c = 0 thì ta cũng suy ra f (0) = 0.

Xét c 6= 0, khi đó ta lại có 2a = a2n+1, 2b = b2n+1 nên hoặc c = a hoặc c = b. Nếu c = a

thì a = f (a) = 0 khi đó f (0) = 0; còn nếu c = b thì b+b2n+1 = 0 suy ra b = 0 do đóf (0) = 0.

Như vậy, ta trong mọi trường hợp ta luôn có f (0) = 0, từ (1) suy ra f ( f (y)) = y, ∀y∈R.

Trong (*) lấy y = 0 thì f (x2n+1) = [ f (x)]2n+1, ∀x ∈ R. Với mọi x ∈ R thì tồn tại z ∈ Rmà x = z2n+1, khi đó f (x) = f (z2n+1) = [ f (z)]2n+1. Do đó với mọi x,y ∈ R ta có

f (x+ y) = f (z2n+1 + f ( f (y))) = f (y)+ [ f (z)]2n+1 = f (y)+ f (x).

Hay f cộng tính trên R. Mặt khác f đơn điệu nên f (x) = cx, ∀x ∈R. Từ đây, kết hợp vớiphương trình f ( f (y)) = y,∀y ∈ R ta suy ra c =±1. Thử lại, ta đi đến kết luận

f (x) = x, ∀x ∈ R hoặc f (x) =−x, ∀x ∈ R.

Nhận xét: Việc vận dụng tính song ánh của f để tính f (0) = 0 là mấu chốt, việc cònlại đưa về PTH Cauchy trong lớp hàm đơn điệu là khá dễ dàng. Ta có bài toán mở rộngthứ 2 đơn giản hơn như sau.

Bài toán 2.3.8. Với n ∈ N∗. Tìm hàm f : R→ R đơn điệu thỏa mãn

f (x+[ f (y)]2n+1) = y2n+1 + f (x), ∀x,y ∈ R. (**)

LỜI GIẢI. Tương tự, ta cũng có f là song ánh. Đặt f (0) = b và giả sử f (a) = 0. Khi đó,trong (**) thay (x,y) = (0,a) ta có

b = f (0) = a2n+1 +b ⇒ a = 0.

Do đó f (0) = 0. Trong (**) thay x = 0 ta có f ([ f (y)]2n+1) = y2n+1, ∀y ∈ R. Với mọiz ∈ R tồn tại y ∈ R mà z = [ f (y)]2n+1, do f là toàn ánh. Khi đó f (z) = f ([ f (y)]2n+1) =y2n+1. Nên với mọi x,z ∈ R ta có

f (x+ z) = f (x+[ f (y)]2n+1) = y2n+1 + f (x) = f (z)+ f (x).

Nói cách khác, f là hàm cộng tính. Mặt khác f đơn điệu nên f (x) = cx, ∀x ∈R. Kết hợpf ([ f (y)]2n+1) = y2n+1, ∀y ∈ R ta sẽ suy ra c =±1. Thử lại, ta đi đến kết luận

f (x) = x, ∀x ∈ R hoặc f (x) =−x, ∀x ∈ R.

Bài toán 2.3.9. (IMO 1992) Tìm tất cả các hàm f : R→ R thỏa mãn

f (x2 + f (y)) = y+[ f (x)]2, ∀x,y ∈ R. (1)

19

Page 25: Đ—I H¯C QU¨C GIA HÀ N¸I TRƯ˝NG Đ—I H¯C KHOA H¯C TÜ NHIÊN KHOA … · 2016-05-09 · Đ—I H¯C QU¨C GIA HÀ N¸I TRƯ˝NG Đ—I H¯C KHOA H¯C TÜ NHIÊN KHOA TOÁN

LỜI GIẢI. Lấy x = 0 ta có f ( f (y)) = y + [ f (0)]2, từ đây dễ thấy f là song ánh. Đặtf (0) = b và giả sử f (a) = 0. Trong (1) thay (x,y) = (0,a) ta có b = f (0) = a + b2.

Trong (1) thay (x,y) = (a,0) ta có f (a2 + b) = 0 = f (a), do f đơn ánh nên a = a2 + b.

Như vậy ta có hệa = a2 +b và b = a+b2.

Từ đây suy ra a = b = 0. Do đó f (0) = 0 và f ( f (y)) = y, ∀y ∈ R. Bây giờ, trong (1) lấyy = 0 ta có

[ f (x)]2 = f (x2) = f ((−x)2) = [ f (−x)]2, ∀x ∈ R.

Suy ra f (x) = ± f (−x), ∀x ∈ R. Nhưng f đơn ánh nên f (−x) = − f (x), ∀x 6= 0, điềunày cũng đúng khi x = 0. Do đó f là hàm lẻ. Với mọi z≥ 0 tồn tại x sao cho z = x2, khiđó f (z) = f (x2) = [ f (x)]2. Do vậy với mọi z≥ 0, y ∈ R ta có

f (z+ y) = f (x2 + f ( f (y))) = f (y)+ [ f (x)]2 = f (y)+ f (z).

Với z≤ 0 thì −z≥ 0. Theo khẳng định trên và chú ý f là hàm lẻ nên

f (z+ y) =− f (−z− y) =−[ f (−z)+ f (−y)] = f (z)+ f (y), ∀z≤ 0,y ∈ R.

Vậy với mọi y,z ∈ R ta đều có f (z + y) = f (z) + f (y). Mặt khác f (x2) = [ f (x)]2 nênf (x) ≥ 0, ∀x ≥ 0. Dẫn đến f (x) = cx, ∀x ∈ R với c ≥ 0. Kết hợp f ( f (y)) = y, ∀y ∈ Rta suy ra f (x) = x, ∀x ∈ R.

Hoàn toàn tương tự, ta có thể dễ dàng giải được các bài toán mở rộng sau.

Bài toán 2.3.10. Cho ∈ N∗. Tìm tất cả các hàm f : R→ R thỏa mãn

f (x2n + f (y)) = y+[ f (x)]2n, ∀x,y ∈ R.

Thay đổi bài toán IMO 1992, ta có bài toán tương tự, nhưng phức tạp hơn như sau:

Bài toán 2.3.11. Tìm hàm f : R→ R thỏa mãn điều kiện

f (x+[ f (y)]2) = f (x)+ y2, ∀x,y ∈ R. (2)

LỜI GIẢI. Cho x = 0 ta được f ([ f (y)]2) = f (0)+ y2, ∀y ∈ R. Từ đây dễ thấy f là đơnánh trên R+. Trong (2) cho y = 0 ta có f (x +[ f (0)]2) = f (x), (i) . Do f đơn ánh trênR+ nên trong (i), ta xét với x > 0 ta suy ra x+[ f (0)]2 = x, ∀x > 0 dẫn đến f (0) = 0.

Từ đây ta có f ([ f (y)]2) = y2, ∀y ∈ R. Do đó f là toàn ánh từ R+ vào [0,+∞) nên vớimọi z ≥ 0 tồn tại y sao cho z = [ f (y)]2, khi đó f (z) = f ([ f (y)]2) = y2. Nên với mọiz≥ 0,x ∈ R ta có

f (x+ z) = f (x+[ f (y)]2) = f (x)+ y2 = f (x)+ f (z).

20

Page 26: Đ—I H¯C QU¨C GIA HÀ N¸I TRƯ˝NG Đ—I H¯C KHOA H¯C TÜ NHIÊN KHOA … · 2016-05-09 · Đ—I H¯C QU¨C GIA HÀ N¸I TRƯ˝NG Đ—I H¯C KHOA H¯C TÜ NHIÊN KHOA TOÁN

Ta chỉ ra điều này cũng đúng khi z≤ 0. Bằng cách chỉ ra f là hàm lẻ. Thật vậy, ta có

f ([ f (y)]2) = y2 = f ([ f (−y)]2), ∀y ∈ R,

nhưng f đơn ánh trên R+ nên [ f (y)]2 = [ f (−y)]2, suy ra f (−y) =± f (y), ∀y ∈ R.

Giả sử tồn tại a 6= 0 mà f (−a) = f (a), khi đó từ (2) ta suy ra

f (a+[ f (y)]2) = f (a)+ y2 = f (−a)+ y2 = f (−a+[ f (y)]2), ∀y ∈ R.

Do f nhận mọi giá trị trên R+ nên ta tồn tại b mà [ f (b)]2 > |a|. Khi đó

a+[ f (b)]2 > 0 và −a+[ f (b)]2 > 0.

Mặt khác f (a+[ f (b)]2) = f (−a+[ f (b)]2) và f đơn ánh trên R+ nên

a+[ f (b)]2 =−a+[ f (b)]2⇒ a =−a⇒ a = 0.

Điều này mâu thuẫn với giả sử a 6= 0. Do đó f (−y) =− f (y), ∀y ∈ R. Hay f là hàm lẻ.Khi đó với mọi t ≤ 0,x ∈ R ta đặt z =−t ≥ 0. Theo trên ta có

f (x+ t) =− f (−x− t) =− f (−x+ z) =−[ f (−x)+ f (z)] = f (x)− f (z) = f (x)+ f (t).

Như vậy với mọi x,y,∈ R ta có f (x+ y) = f (x)+ f (y). Thêm vào đó f ([ f (y)]2) = y2 vàf toàn ánh trên R+, suy ra f (x)≥ 0, ∀x≥ 0. Từ đó ta có f (x) = x, ∀x ∈R. Dễ thấy đâylà nghiệm của bài toán.

Một cách tương tự, ta có thể giải được bài toán mở rộng sau:

Bài toán 2.3.12. Với n ∈ N∗. Tìm hàm f : R→ R thỏa mãn điều kiện

f (x+[ f (y)]2n) = f (x)+ y2n, ∀x,y ∈ R.

Bài toán 2.3.13. (American Mathematical Monthly) Cho 1 < n ∈N. Xác định tất cả các

hàm f : R→ R thỏa mãn

f (x+ yn) = f (x)+ [ f (y)]n, ∀x,y ∈ R. (*)

LỜI GIẢI. Cho x = y = 0 suy ra f (0) = 0. Chỉ lấy x = 0 thì f (yn) = [ f (y)]n, ∀y ∈ R.

Với mọi a ≥ 0 tồn tại y ∈ R mà a = yn, khi đó f (a) = f (yn) = [ f (y)]n. Do đó với mọia≥ 0,x ∈ R ta có

f (x+a) = f (x+ yn) = f (x)+ [ f (y)]n = f (x)+ f (a).

21

Page 27: Đ—I H¯C QU¨C GIA HÀ N¸I TRƯ˝NG Đ—I H¯C KHOA H¯C TÜ NHIÊN KHOA … · 2016-05-09 · Đ—I H¯C QU¨C GIA HÀ N¸I TRƯ˝NG Đ—I H¯C KHOA H¯C TÜ NHIÊN KHOA TOÁN

Ở trên, nếu lấy x =−a thì ta suy ra f (−a) =− f (a), ∀a≥ 0, điều này cũng có nghĩa f

là hàm lẻ. Vì thế với mọi a≥ 0,x ∈ R ta có

f (x−a) =− f (−x+a) =−[ f (−x)+ f (a)] = f (x)− f (a).

Từ đó dẫn đến f (x + y) = f (x)+ f (y), ∀x,y ∈ R. Theo bài toán PTH Cauchy thì ta cóf (rx) = r f (x), ∀r ∈Q, ∀x ∈ R đặc biệt f (r) = r f (1), ∀x ∈Q.

Với mọi r ∈Q, x ∈ R ta có f ((r + x)n) = [ f (r + x)]n = [ f (r)+ f (x)]n. Mặt khác

f ((r + x)n) = f (n

∑k=0

Cknrkxn−k) =

n

∑k=0

Cknrk f (xn−k),

[ f (r)+ f (x)]n =n

∑k=0

Ckn[ f (r)]k[ f (x)]n−k =

n

∑k=0

Cknrk[ f (1)]k[ f (x)]n−k.

Suy ran

∑k=0

Cnk rk f (xn−k) =

n

∑k=0

Cknrk[ f (1)]k[ f (x)]n−k (i) . Cố định x thì hai vế của

đẳng thức (i) đều là đa thức biến r bậc n. Do (i) đúng với mọi r ∈ Q nên hệ số củark, k = 0,1, ...,n bằng nhau. Đặc biệt, với k = n−2 và k = n−1 ta có

f (x2) = [ f (x)]2[ f (1)]n−2 và f (x) = f (x)[ f (1)]n−1, ∀x ∈ R. (ii)

Trong (ii) lấy x = 1 ta suy ra f (1) = 0 hoặc f (1) = 1, còn với n chẵn thì ta có thêmtrường hợp f (1) =−1. Ta đi xét 3 trường hợp này.

1) Nếu f (1) = 0, từ (ii) ta suy ra f (x)≡ 0, ∀x ∈ R. Hàm này thỏa mãn bài toán.2) Nếu f (1) = 1, thì từ (ii) ta suy ra f (x2) = [ f (x)]2. Từ đó dễ suy ra f (x) = x, ∀x ∈R.

Hàm này thỏa mãn bài toán.3) Với n chẵn và f (1) = −1. Đặt g(x) = − f (x), ∀x ∈ R. Khi đó, hàm g(x) thỏa mãn

điều kiện (*) của bài toán và g(1) = 1. Theo trường hợp trên thì g(x) = x, ∀x ∈ R. Nênf (x) = −x, ∀x ∈ R. Nhưng thử lại với chú ý n chẵn thì ta thấy rằng hàm f (x) = −x

không thỏa mãn (*).Kết luận: bài toán có hai nghiệm là f (x)≡ 0, ∀x ∈ R hoặc f (x) = x, ∀x ∈ R.

Bài toán 2.3.14. (USA 2002) Tìm hàm f : R→ R thỏa mãn

f (x2− y2) = x f (x)− y f (y), ∀x,y ∈ R. (*)

LỜI GIẢI. Trong (*) cho x = y = 0 ta có f (0) = 0, với y = 0 thì f (x2) = x f (x), vớix = 0 thì f (−y2) =−y f (y), với x =−y thì f (−x) =− f (x), ∀x ∈ R. Ta viết lại (*) nhưsau f (x2− y2) = f (x2)− f (y2) hay f (u)− f (v) = f (u− v), ∀u≥ 0,v≥ 0. Từ đây ta cóf (x) = f (x + y− y) = f (x + y)− f (y) hay f (x + y) = f (x)+ f (y), ∀x,y ≥ 0 ∈ R. Kết

22

Page 28: Đ—I H¯C QU¨C GIA HÀ N¸I TRƯ˝NG Đ—I H¯C KHOA H¯C TÜ NHIÊN KHOA … · 2016-05-09 · Đ—I H¯C QU¨C GIA HÀ N¸I TRƯ˝NG Đ—I H¯C KHOA H¯C TÜ NHIÊN KHOA TOÁN

hợp 2 khẳng định trên và f là hàm lẻ suy ra f (rx) = r f (x), ∀r ∈ Q, x ∈ R. Đặc biệtf (r) = r f (1), ∀r ∈Q. Ta đi tính f ([x+1]2) với x≥ 0 theo hai cách. Ta có

f ([x+1]2) = f (x2 +2x+1) = f (x2)+2 f (x)+ f (1) = x f (x)+2 f (x)+ f (1), ∀x≥ 0,

và f ([x+1]2) = (x+1) f (x+1) = (x+1)( f (x)+ f (1)) = x f (x)+ f (x)+x f (1)+ f (1).

Từ đó suy ra f (x) = x f (1), ∀x≥ 0. Nhưng theo trên f là hàm lẻ nên với mọi x≤ 0 ta cóf (x) = − f (−x) = −(−x) f (1) = x f (1). Do đó với mọi x ∈ R ta có f (x) = x f (1). Thửlại hàm này thỏa mãn bài toán. Vậy f (x) = cx, ∀x ∈ R với c ∈ R tùy ý.

Nhận xét: Ta chỉ cần có f (x2−y2) = f (x2)− f (y2) suy ra f (x+y) = f (x)+ f (y), ∀x+y,y≥ 0∈R. Và từ đây f (rx) = r f (x), ∀r∈Q+ do f lẻ nên f (rx) = r f (x), ∀r∈Q, x∈R.

Sau đó từ f (x)+ f (x + 1) = f (2x + 1) = f [(x + 1)2− x2] = (x + 1) f (x + 1)− x f (x) tasẽ suy ra f (x) = x f (1), ∀x ∈ R.

Bài toán 2.3.15. (Canada 2008) Xác định hàm số f : Q→Q thỏa mãn

f (2 f (x)+ f (y)) = 2x+ y, ∀x,y ∈Q. (1)

LỜI GIẢI. Lấy y = x ta có f (3 f (x)) = 3x, ∀x ∈Q. Ở đây, thay x bởi 3 f (x) suy ra

f (9x) = f (3 f (3 f (x))) = 3[3 f (x)] = 9 f (x), ∀x ∈Q.

Với x = 0 ta suy ra f (0) = 9 f (0) do đó f (0) = 0.

Lấy x = 0 trong (1) ta có f ( f (y)) = y, ∀y ∈Q. Khi đó, tác động f lên hai vế (1) suy ra2 f (x)+ f (y) = f (2x + y), ∀x,y ∈Q. Ở đây, lấy y = 0 ta có f (2x) = 2 f (x), ∀x ∈Q. Vìvậy, với mọi x,y ∈Q ta có f (2x+ y) = f (2x)+ f (y). Điều này tương đương với

f (x+ y) = f (x)+ f (y), ∀x,y ∈Q.

Theo bài toán PTH Cauchy suy ra f (x) = x f (1), ∀x ∈ Q. Kết hợp f ( f (y)) = y ta suyra f (x)≡ 0, ∀x ∈ R hoặc f (x) = x, ∀x ∈Q. Thử lại, ta thấy chỉ hàm f (x) = x, ∀x ∈Qthỏa mãn bài toán.

Nhận xét: Từ lời giải của bài toán này, một cách tương tự, ta dễ dàng suy ra kết luậnsau: Giả sử a,b ∈Q và a,b 6= 0, a+b 6= 0,±1 thì bài toán tìm hàm f : Q→Q thỏa mãn

f (a f (x)+b f (y)) = ax+by, ∀x,y ∈Q

có nghiệm duy nhất là f (x) = x, ∀x ∈Q.

23

Page 29: Đ—I H¯C QU¨C GIA HÀ N¸I TRƯ˝NG Đ—I H¯C KHOA H¯C TÜ NHIÊN KHOA … · 2016-05-09 · Đ—I H¯C QU¨C GIA HÀ N¸I TRƯ˝NG Đ—I H¯C KHOA H¯C TÜ NHIÊN KHOA TOÁN

Bài toán 2.3.16. (Indian MO 2005) Tìm hàm f : R→ R thỏa mãn

f (x2 + y f (z)) = x f (x)+ z f (y), ∀x,y,z ∈ R. (*)

LỜI GIẢI. Nhận thấy f (x) ≡ 0, ∀x ∈ R là nghiệm của bài toán. Ta đi tìm nghiệmkhác, giả sử đó là f (x). Dễ thấy f (0) = 0 và tồn tại a 6= 0 mà f (a) 6= 0. Trong (*) lấy(x,y,z) = (0,a,z) ta có f (a f (z)) = z f (a), từ đây dễ thấy f là đơn ánh trên R. Từ (*)thay x = 0,y = z = 1 ta có f ( f (1)) = f (1), do f đơn ánh nên f (1) = 1. Lấy x = 0,y = 1ta có f ( f (z)) = z, trong (*) chỉ lấy z = 1 ta có f (x2 + y) = x f (x)+ f (y), lấy y = 0 thìf (x2) = x f (x). Từ (*) thay x =−1,y = z = 0 ta suy ra f (−1) =−1. Lấy x = 0,z =−1 tasuy ra f (y) =− f (y), ∀y∈R. Từ đó, ta chỉ ra f cộng tính. Suy ra f (rx) = r f (x), ∀r ∈Q;f (x+1) = f (x)+ f (1). Đến đây, ta đinh tính f ([x+1]2) theo hai cách.Ta có f ([x + 1]2) = f (x2 + 2x + 1) = f (x2)+ 2 f (x)+ f (1) = x f (x)+ 2 f (x)+ f (1), vàf ([x+1]2) = (x+1) f (x+1) = (x+1)( f (x)+ f (1)) = x f (x)+ f (x)+ x f (1)+ f (1).Từ hai đẳng thức trên suy ra f (x) = x f (1) = x, ∀x ∈ R.

Nhận xét: Nếu thay đổi (*) thành f (x2 +y f (z)) = x f (x)−z f (y), ∀x,y,z∈R, thì nghiệmcủa bài toán là f (x) =−x, ∀x ∈ R.

Bài toán 2.3.17. Tìm hàm f : R→ R thỏa mãn

f (x+ y)+ f (xy) = f (x) f (y)+ f (x)+ f (y), ∀x,y ∈ R. (1)

LỜI GIẢI. Ta thấy rằng (1) là tổng của hai phương trình

f (x+ y) = f (x)+ f (y) (2)

f (xy) = f (x)+ f (y) (3)

Ta biết rằng hàm f thỏa mãn đồng thời (2) và (3) thì f (x) = 0, ∀x ∈ R hoặc f (x) =x, ∀x ∈ R. Do vậy nếu từ (1) ta chỉ ra (2) và (3) cũng đúng thì bài toán được giải xong.Để tách được các phương trình từ (1) ta quan tâm đến tính chẵn, lẻ của hàm số. Chẳnghạn, nếu f là hàm lẻ thì thay y bởi −y ta có

f (−xy)+ f (x− y) = f (x) f (−y)+ f (x)+ f (−y). (4)

Cộng (1) và (4), chú ý hàm f lẻ ta sẽ có

f (x+ y)+ f (x− y) = 2 f (x).

Từ phương trình này và khẳng định f (0) = 0 (suy ra khi lấy x = y = 0 ở (1)), cho y = x

ta có f (2x) = 2 f (x). Suy ra f (x+y)+ f (x−y) = 2 f (x), điều này dẫn đến (2) đúng, kéo

24

Page 30: Đ—I H¯C QU¨C GIA HÀ N¸I TRƯ˝NG Đ—I H¯C KHOA H¯C TÜ NHIÊN KHOA … · 2016-05-09 · Đ—I H¯C QU¨C GIA HÀ N¸I TRƯ˝NG Đ—I H¯C KHOA H¯C TÜ NHIÊN KHOA TOÁN

theo (3) cũng đúng.

Quay trở lại bài toán. Ta quan tâm đến tính chẵn, lẻ của hàm số. Để làm điều này tađi tính f (1), f (−1). Trong (1) thay y = 1 ta có

f (x)+ f (x+1) = f (1) f (x)+ f (x)+ f (1)⇒ f (x+1) = f (1) f (x)+ f (1). (5)

Nếu ta đặt f (1) = a thì theo (5) ta dễ dàng tính được

f (2) = a2 +a, f (3) = a3 +a2 +a, f (4) = a4 +a3 +a2 +a.

Nếu tiếp tục quá trình này, ta sẽ tính được f (n), n ∈ N. Nhưng công việc của ta là tínhf (1). Chú ý rằng ở (1) lấy x = y = 2 ta có 2 f (4) = [ f (2)]2 + 2 f (2). Thay các giá trị ởtrên, ta suy ra

a4 +a3 +a2 +a = (a2 +a)2 +2(a2 +a)⇒ a4 = a2.

Suy ra a = 0,±1. Ta đi xét các trường hợp này.*) Nếu a = f (1) = 0. Thì từ (5) ta suy ra f (x) = 0, ∀x ∈ R. *) Nếu a = f (1) =−1. Từ(5) ta suy ra f (x + 1) =− f (x)−1. Ở đây lấy x =−1 ta có f (−1) =−1. Trong (1) lấyy =−1 ta có

f (x−1)+ f (−x) = f (−1) f (x)+ f (x)+ f (−1). (6)

Do f(-1)=1 nên f (x−1)+ f (−x) =−1, ∀x ∈R. Ở đây thay x bởi −x ta có f (−x−1)+f (x) =−1. Theo trên f (x)+ f (x+1) =−1 do đó f (x+1) = f (−x−1), ∀x ∈R. Hay f

là hàm chẵn. Trừ (1) cho (4), chú ý f là hàm chẵn, ta có f (x+y)− f (x−y) = 0, ∀x,y∈R.

Điều này dẫn đến f là hàm hằng, mâu thuẫn với f (0) = 0 và f (1) =−1. Vậy trường hợpf (1) =−1 không xảy ra.*) Nếu a = f (1) = 1 thì từ (5) ta suy ra f (x+1)− f (x) = 1. Lấy x =−1 thì f (−1) =−1.

Theo (6) ta có f (x−1)+ f (−x) =−1, ở đây thay x bởi−x ta có f (−x−1)+ f (x) =−1.

Mặt khác f (x + 1)− f (x) = 1 nên suy ra f (x + 1) + f (−x− 1) = 0, ∀x ∈ R hay f

là hàm lẻ. Theo lập luận ban đầu thì (2) và (3) đúng. Do vậy trong trường hợp nàyf (x) = x, ∀x ∈ R.

Kết luận: nghiệm của bài toán là f (x) = 0, ∀x ∈ R hoặc f (x) = x, ∀x ∈ R.

Bài toán 2.3.18. (Indian MO 2003) Tìm tất cả các hàm f : R→ R thỏa mãn

f (x+ y) = f (x) f (y)− f (xy)+1, ∀x ∈ R.

LỜI GIẢI. Bàì toán này thực sự có thể giải thông qua bài toán trên.Thật vậy, bằng cách đặt f (x)− 1 = g(x), ∀x ∈ R thì phương trình điều kiện cho trở

25

Page 31: Đ—I H¯C QU¨C GIA HÀ N¸I TRƯ˝NG Đ—I H¯C KHOA H¯C TÜ NHIÊN KHOA … · 2016-05-09 · Đ—I H¯C QU¨C GIA HÀ N¸I TRƯ˝NG Đ—I H¯C KHOA H¯C TÜ NHIÊN KHOA TOÁN

thànhg(x+ y)+1 = [g(x)+1][g(y)+1]− [g(xy)+1]+1, ∀x,y ∈ R

⇐⇒ g(x+ y)+g(xy) = g(x)g(y)+g(x)+g(y), ∀x,y ∈ R.

Nhận xét: Chúng ta hoàn toàn có thể giải độc lập, bằng cách đi tính giá trị của f (1).

Bài toán 2.3.19. (Romania RMC 2008) Tìm hàm số f : R→ R thỏa mãn

f (x+ y

3) =

f (x)+ f (y)2

, ∀x,y ∈ R.

LỜI GIẢI. Đặt f (x)− f (0) = g(x), ∀x ∈ R. Khi đó, từ điều kiện ta có g(0) = 0 và

g(x+ y

3) =

g(x)+g(y)2

, ∀x,y ∈ R.

Lấy y = 0 ta có g(x3) =

g(x)2

, ∀x ∈ R. Từ đó

g(x+ y

3) = g(

x3)+g(

y3), ∀x,y ∈ R.

Điều này tương đương với g(x + y) = g(x)+g(y), ∀x,y ∈ R. Lấy y = x suy ra g(2x) =2g(x), ∀x ∈ R. Ta có

g(x) = g(2x+ x

3) =

g(2x)+g(x)2

⇒ g(2x) = g(x), ∀x ∈ R.

Mà g(2x) = 2g(x) nên g(x) = 0, ∀x ∈R. Do đó f (x) = f (0), ∀x ∈R. Điều này có nghĩaf (x) = const trên R. Đây là nghiệm của bài toán.Nhận xét: Ta có thể mở rộng bài toán như sau: Với p 6= q ∈ N∗, tìm hàm số f : R→ Rthỏa mãn

f (x+ y

p) =

f (x)+ f (y)q

, ∀x,y ∈ R.

Bài toán 2.3.20. (Romania RMC 2006) Cho r,s ∈Q. Tìm hàm f : Q→Q thỏa mãn

f (x+ f (y)) = f (x+ r)+ y+ s, ∀x,y ∈Q. (*)

LỜI GIẢI. Đặt g(x) = f (x)− r− s, ∀x ∈Q. Khi đó

g(x+g(y)) = g(x+ f (y)− r− s) = f ((x− r− s)+ f (y))− r− s =

= f (x− s)+ y− r = g(x− s)+ y+ s, ∀x,y ∈Q.

26

Page 32: Đ—I H¯C QU¨C GIA HÀ N¸I TRƯ˝NG Đ—I H¯C KHOA H¯C TÜ NHIÊN KHOA … · 2016-05-09 · Đ—I H¯C QU¨C GIA HÀ N¸I TRƯ˝NG Đ—I H¯C KHOA H¯C TÜ NHIÊN KHOA TOÁN

Và g2(x + g(y)) = g(y + s + g(x− s)) = x + g(y), ∀x,y ∈ Q hay g2 = id đối với cácphần tử dạng x+g(y). Cố định y = y0 ∈Q thì tập

{x+g(y0) : x ∈Q}= Q,

vì vậy g2 = id trên Q. Do đó ta có

g(x+ y) = g(x+g(g(y))) = g(x− s)+g(y)+ s, ∀x,y ∈Q.

Mặt khác g(x+y) = g(y−s)+g(x)+s. Do đó g(x)−g(x−s) = g(y)−g(y−s), ∀x,y∈Q. Nên g(x)−g(x− s) = c (const). Điều này dẫn đến

g(x+ y) = g(x)+g(y)+ s− c, ∀x,y ∈Q.

Lấy x = y = 0 ta có g(0) = −(s− c), do vậy g(x + y) = g(x)+ g(y)− g(0). Từ đây dễsuy ra g(x) = ax + g(0), ∀x ∈ Q với a,g(0) = −(s− c) ∈ Q là các hằng số. Nhưngg2 = id trên Q, nên ta có a(ax + b)+ b = x, ∀x ∈ Q suy ra hoặc a = 1, b = 0 hoặca =−1, b =−2s. Như vậy ta có g(x) = x hoặc g(x) =−x−2s, ∀x ∈Q. Dẫn đến

f (x) = x+ r + s, ∀x ∈Q và f (x) =−x+ r− s, ∀x ∈Q.

Thử lại, ta thấy đây là hai nghiệm của bài toán.

MỘT SỐ BÀI TẬP VẬN DỤNG

Bài tập 2.3.1. Tìm hàm f : R→ R liên tục thỏa mãn

f (x+ y) = f (x)+ f (y)+ xy(x+ y), ∀x,y ∈ R.

Gợi ý. Đặt f (x)− x3

3= g(x), ∀x ∈ R.

Bài tập 2.3.2. (Balkan 2000) Tìm hàm f : R→ R thỏa mãn

f (x f (x)+ f (y)) = y+[ f (x)]2, ∀x,y ∈ R.

Gợi ý. Chỉ ra f (0) = 0, f ( f (y)) = y. Sau đó chứng minh f cộng tính và [ f (x)]2 = x2.

Từ đó, xét các trường hợp f (1) = 1 hoặc f (1) =−1. Ứng với các trường hợp này là cácnghiệm f (x) = x, ∀x ∈ R và f (x) =−x, ∀x ∈ R.

Bài tập 2.3.3. (Belarus 1997) Tìm hàm g : R→ R thỏa mãn

g(x+ y)+g(x)g(y) = g(xy)+g(x)+g(y), ∀x,y ∈ R.

27

Page 33: Đ—I H¯C QU¨C GIA HÀ N¸I TRƯ˝NG Đ—I H¯C KHOA H¯C TÜ NHIÊN KHOA … · 2016-05-09 · Đ—I H¯C QU¨C GIA HÀ N¸I TRƯ˝NG Đ—I H¯C KHOA H¯C TÜ NHIÊN KHOA TOÁN

Gợi ý. Ta đi tìm nghiệm f khác hai nghiệm tầm thường f (x) = 0 và f (x) = 2.

Chi ra g(x+ y) = g(x)+g(y), ∀x,y ∈Q và f (x) là hàm lẻ. Từ đó chứng minhg(x2) = [g(x)]2. Đến đây, bài toán đã quan thuộc và có nghiệm f (x) = x, ∀x ∈ R.

Bài tập 2.3.4. (VietNam 1999) Giả sử hàm f : [0,1]→ R liên tục thỏa mãn

(i). f (0) = f (1) = 0,

(ii). Với mọi x,y ∈ [0,1] ta có 2 f (x)+ f (y) = 3 f (2x+ y

3).

Chứng minh rằng f (x) = 0,∀x ∈ [0,1].

Gợi ý. - Thay (x,y) = (0,1) ta suy ra f (1/3) = 0.

- Thay (x,y) = (1,0) ta suy ra f (2/3) = 0.- Thay (x,y) = (0,1/3) ta suy ra f (1/9) = 0.- Thay (x,y) = (1/3,0) ta suy ra f (2/9) = 0.- Thay (x,y) = (2/3,0) ta suy ra f (4/9) = 0.- Thay (x,y) = (1/3,1) ta suy ra f (5/9) = 0.- Thay (x,y) = (1,1/3) ta suy ra f (7/9) = 0.Tương tự như trên, sử dụng (ii), bằng quy nạp ta có thể chỉ ra f ( a

3n ) = 0, a,n ∈ N. Màtập hợp gồm tất cả các số hữu tỉ có dạng a

3n , a,n ∈ N trù mật trong đoạn [0,1] nênf (x) = 0,∀x ∈ [0,1].

Bài tập 2.3.5. Tìm tất cả các hàm f (x) xác định trên R thỏa mãn

f [(x+1) f (y)] = y[ f (x)+1], ∀x,y ∈ R.

Gợi ý. Chỉ ra f (0) = 0, f (−1) = −1, f (1) = 1. Từ đó chỉ ra f nhân tính và f cộngtính. Đến đây bài toán đã quen thuộc, và nghiệm là f (x) = x, ∀x ∈ R.

Bài tập 2.3.6. (THTT) Tìm hàm f : (0,+∞)→ R có đạo hàm tại x = 1 và

f (xy) =√

x f (y)+√

y f (x), ∀x,y > 0.

Gợi ý. Đặt g(x) =f (x)√

x, ∀x > 0.

Bài tập 2.3.7. (IMO 1989, Shortlist) Xác định tất cả các số thực a sao cho tồn tại hàm

f : [0,1]→ R liên tục thỏa mãn f (0) = 0, f (1) = 1 và

f (x+ y

2) = (1−a) f (x)+a f (y), ∀ 0≤ x≤ y≤ 1.

Gợi ý. Chỉ ra f (1/2) = a, f (1/4) = a2, f (3/4) = 2a−a2, f (1/2) = 3a2−2a3. Do đóta có 3a2−2a3 = a suy ra a ∈ {0,1/2,1}. Rồi đi xét từng trường hợp này. Đi đến giá trịcần tìm của a là a = 1/2.

28

Page 34: Đ—I H¯C QU¨C GIA HÀ N¸I TRƯ˝NG Đ—I H¯C KHOA H¯C TÜ NHIÊN KHOA … · 2016-05-09 · Đ—I H¯C QU¨C GIA HÀ N¸I TRƯ˝NG Đ—I H¯C KHOA H¯C TÜ NHIÊN KHOA TOÁN

Bài tập 2.3.8. (IMO 2002) Tìm tất cả các hàm f : R→ R thỏa mãn

( f (x)+ f (z))( f (y)+ f (t)) = f (xy− zt)+ f (xt + yz), ∀x,y,z, t ∈ R. (*)

Bài tập 2.3.9. (IMO 2005, Shortlist) Tìm hàm f : R→ R thỏa mãn

f (x+ y)+ f (x) f (y) = f (xy)+2xy+1, ∀x,y ∈ R.

Đáp số. Nghiệm cần tìm là: f (x) = 2x−1, f (x) =−x−1 hoặc f (x) = x2−1, ∀x∈R.

Bài tập 2.3.10. (IMO 2004, Shortlist) Tìm f : R→ R thỏa mãn

f (x2 + y2 +2 f (xy)) = [ f (x+ y)]2, ∀x,y ∈ R.

Bài tập 2.3.11. Tìm hàm f : R→ R liên tục thỏa mãn

( f (x)+ f (y)) f (x+ y

2) = 2 f (x) f (y), ∀x,y ∈ R.

Gợi ý. Nghiệm tầm thường f (x)≡ 0, ∀x∈R. Tìm nghiệm khác, khi đó f (x) 6= 0, ∀x∈R. Đặt g(x) = 1/ f (x). Khi đó ta sẽ đưa về dạng PTH Jensen.

Bài tập 2.3.12. (Japan Math Olympiad Final 2008) Timg f : R→ R thỏa mãn

f (x+ y) f ( f (x)− y) = x f (x)− y f (y), ∀x,y ∈ R.

Bài tập 2.3.13. Tìm hàm f : R→ R thỏa mãn

(i). f (x+ y) = f (x)+ f (y), ∀x,y ∈ R,

(ii). f (x) f (1x) = 1, ∀x 6= 0,

Gợi ý. Chỉ ra f liên tục tại 0.

Bài tập 2.3.14. (BMO 2003, Shortlist) Tìm tất cả giá trị có thể có của f (20042003

), trong

đó f : Q→ [0,+∞) là hàm thỏa mãn các tính chất

(i). f (xy) = f (x) f (y), ∀x,y ∈Q,

(ii). Với mọi x ∈Q nếu f (x)≤ 1 thì ta suy ra f (x+1)≤ 1,

(iii). f (20032002

) = 2.

Gợi ý. Đáp số f (20042003

) =14.

Bài tập 2.3.15. (THTT) Tìm hàm f : R→ R thỏa mãn

f (x+ y) = x2 f (1x)+ y2 f (

1y), ∀x,y 6= 0.

29

Page 35: Đ—I H¯C QU¨C GIA HÀ N¸I TRƯ˝NG Đ—I H¯C KHOA H¯C TÜ NHIÊN KHOA … · 2016-05-09 · Đ—I H¯C QU¨C GIA HÀ N¸I TRƯ˝NG Đ—I H¯C KHOA H¯C TÜ NHIÊN KHOA TOÁN

Bài tập 2.3.16. (IMO 1996, Shortlist) Giả sử 0 < a < 1 và f là hàm liên tục trên [0,1]thỏa mãn f (0) = 0, f (1) = 1 và

f (x+ y

2) = (1−a) f (x)+a f (y), ∀x,y ∈ [0,1].

Xác định giá trị của f (1/7).

Bài tập 2.3.17. (THTT - T10/2004) Tìm tất cả các số thực a > 0 sao cho tồn tại số thực

k > 0 và hàm số f : R→ R thỏa mãn

f (x)+ f (y)2

≥ f (x+ y

2)+ k|x− y|a, ∀x,y ∈ R.

Bài tập 2.3.18. (IMO 2003, Shortlist) Tìm hàm f : R+→ R+ thỏa mãn

(i). f (xyz)+ f (x)+ f (y)+ f (z) = f (√

xy)+ f (√

yz)+ f (√

zx), ∀x,y,z > 0(ii). f (x) < f (y), ∀1≤ x < y.

Bài tập 2.3.19. (China TST 2011) Cho só nguyên n≥ 2. Tìm hàm f : R→R thỏa mãn

f (x− f (y)) = f (x+ yn)+ f ( f (y)+ yn), ∀x,y ∈ R.

Bài tập 2.3.20. (APMO 2011) Tìm hàm f : R→ R thỏa mãn hai điều kiện sau

(i). Tồn tại số thực M mà f (x) < M, ∀x ∈ R,

(ii). f (x f (y))+ y f (x) = x f (y)+ f (xy), ∀x,y ∈ R.

Bài tập 2.3.21. (Romania TST 1997) Tìm tất cả các hàm số f : R→ [0,+∞) thỏa mãn

f (x2 + y2) = f (x2− y2)+ f (2xy), ∀x,y ∈ R.

Gợi ý. Chỉ ra f (0) = 0, f (x) là hàm chẵn. Ta xác định f (x) với x > 0.

Với a,b > 0 tồn tại x,y sao cho x2− y2 = a, 2xy = b. Ta có

f (a)+ f (b) = f (√

a2 +b2), ∀a,b > 0.

Đặt g(t) = f (√

t), ∀t > 0. Suy ra g(a)+g(b) = g(a+b), ∀a,b > 0 và g(t)≥ 0, ∀t ≥ 0.

Do đó g(t) = kt,k ≥ 0. Từ đó f (x) = kx2.

30

Page 36: Đ—I H¯C QU¨C GIA HÀ N¸I TRƯ˝NG Đ—I H¯C KHOA H¯C TÜ NHIÊN KHOA … · 2016-05-09 · Đ—I H¯C QU¨C GIA HÀ N¸I TRƯ˝NG Đ—I H¯C KHOA H¯C TÜ NHIÊN KHOA TOÁN

Chương 3

Một số phương pháp giải phươngtrình hàm

3.1 Phương pháp sử dụng tính liên tục của hàm số

Đối với phương pháp này, ta thường sử dụng một cách trực tiếp định nghĩa của hàmliên tục, tức f (x) liên tục tại x0 khi và chỉ khi mọi dãy xn,n ∈ N∗, có giới hạn là x0, thìf (xn) có giới hạn là f (x0) khi n→+∞. Một số kiến thức quan trọng nữa cần quan tâm,đó là: định lý giá trị trung gian, f (x) liên tục và đơn ánh thì đơn điệu, hay f (x) liên tụctrên một đoạn thì đạt được GTLN, GTNN trên đoạn đó.

Bài toán 3.1.1. Cho trước số thực a 6= ±1. Xác định tất cả các hàm số f : R→ R liên

tục thỏa mãn f (x) = f (ax), ∀x ∈ R.

LỜI GIẢI. Ta xét hai trường hợp sau:- Nếu |a|< 1. Khi đó ta có

f (x) = f (ax) = f (a2x) = ... = f (anx), ∀n ∈ N∗, x ∈ R. (1)

Do |a|< 1 nên |a|n→ 0 khi n→+∞, suy ra với mỗi x ∈R ta có |a|nx→ 0 khi n→+∞.Mặt khác, f là hàm liên tục nên từ (1) ta có

f (x) = limn→+∞

f (anx) = f ( limn→+∞

anx) = f (0), ∀x ∈ R.

Nghĩa là f (x)≡ c với c là hằng số thực bất kì.- Nếu |a| > 1. Từ f (x) = f (ax),∀x ∈ R suy ra f (x) = f (

xa),∀x ∈ R. Đặt 1/a = b thì

|b|= |1/a|< 1 và f (x) = f (bx),∀x ∈R. Theo trường hợp trên ta suy ra f (x)≡ c trên R.Vậy nghiệm của bài toán là f (x)≡ c,∀x ∈ R, với c ∈ R bất kì.

31

Page 37: Đ—I H¯C QU¨C GIA HÀ N¸I TRƯ˝NG Đ—I H¯C KHOA H¯C TÜ NHIÊN KHOA … · 2016-05-09 · Đ—I H¯C QU¨C GIA HÀ N¸I TRƯ˝NG Đ—I H¯C KHOA H¯C TÜ NHIÊN KHOA TOÁN

Bài toán 3.1.2. Tìm hàm f : R→ R liên tục thỏa mãn

f (x)+ f (23

x) =35

x, ∀x ∈ R.

LỜI GIẢI. Đặt f (x)− 925

x = g(x), do f liên tục nên g cũng liên tục. Từ phương trình điều

kiện ta có g(x)+g(23

x) = 0⇒ g(x) =−g(23

x) = g(49

x),∀x ∈ R.

Sử dụng kết quả của bài toán ban đầu, ta có g(x)≡ c, với c ∈ R. Do đó f (x) =925

x+ c,với c ∈ R, là nghiệm của bài toán.Nhận xét: Ta có thể mở rộng bài toán, với hướng giải không khác, như sau: Tìm f liêntục trên R thỏa mãn f (ax)+ f (bx) = h(x), trong đó a,b ∈R và h(x) biểu diễn được dướidạng h(x) = k(ax)+ k(bx), với k là hàm liên tục trên R bất kì.

Bài toán 3.1.3. Tìm hàm f xác định và liên tục trên R thỏa mãn

f (4x)+ f (9x) = 2 f (6x), ∀x ∈ R.

LỜI GIẢI. Từ phương trình điều kiện ta suy ra f (23

x)+ f (32

x) = 2 f (x), ∀x ∈ R.

Đặt f (x)− f (32

x) = g(x). Do hàm f liên tục nên g là hàm liên tục, ngoài ra

g(x) = g(23

x),∀x ∈ R. Dẫn đến g là hàm hằng, nhưng g(0) = 0 nên g(x)≡ 0 trên R.

Suy ra f (x) = f (32

x),∀x∈R mà hàm f liên tục nên f (x) = c, c∈R tùy ý. Đây là nghiệmcủa bài toán.Nhận xét: Ta có thể mở rộng bài toán như sau: Với a 6= ±b, tìm f liên tục thỏa mãnf (a2x)+ f (b2x) = 2 f (abx), ∀x ∈ R.

Bài toán 3.1.4. Tìm hàm liên tục f : R→ R thỏa mãn

3 f (3x) = f (x)+ x, ∀x ∈ R.

LỜI GIẢI. Đặt f (x)− x8

= g(x). Khi đó g(x) liên tục và 3g(3x) = g(x),∀x ∈ R. Suy ra

với mọi x ∈ R ta có g(x) =13

g(x3) = ... = (

13)ng(

x3n ).

Lấy giới hạn hai vế khi n→+∞ ta được g(x) = 0. Do đó f (x) =x8,∀x ∈ R.

Nhận xét: Ta có bài toán tổng quát:Cho a,b,c ∈ R thỏa mãn |a|> 1, |b|> 1. Tìm hàm f (x) liên tục trên R thỏa mãnf (x)+a f (bx) = cx,∀x ∈ R. Nghiệm của bài toán này là f (x) =

cx1+ab

.

Bài toán 3.1.5. (THTT T4/2010) Tìm hàm f liên tục trên R thỏa mãn

f (2010x− f (y)) = f (2009x)− f (y)+ x, ∀x,y ∈ R.

32

Page 38: Đ—I H¯C QU¨C GIA HÀ N¸I TRƯ˝NG Đ—I H¯C KHOA H¯C TÜ NHIÊN KHOA … · 2016-05-09 · Đ—I H¯C QU¨C GIA HÀ N¸I TRƯ˝NG Đ—I H¯C KHOA H¯C TÜ NHIÊN KHOA TOÁN

LỜI GIẢI. Với (x,y) = (0,0) ta có f (− f (0)) = 0. Với y =− f (0), sử dụng f (− f (0)) = 0,ta suy ra f (2010x) = f (2009x)− x, ∀x ∈ R.

Đặt f (x)−x = g(x), suy ra g(2010x) = g(2009x),∀x∈R. Hay g(x) = g(20092010

x), ∀x∈R.

Từ đây dễ dàng suy ra g(x) = c,∀x ∈ R, với c là hằng số.Do đó f (x) = x+ c,∀x ∈ R. Thử lại, đây là nghiệm của bài toán.

Bài toán 3.1.6. Tìm hàm f (x) liên tục trên [0,1] thỏa mãn

f (x f (x)) = f (x), ∀x ∈ [0,1].

LỜI GIẢI. Với mọi a ∈ [0,1], đặt b = f (a).Khi đó f (x) xác định tại x = ab với f (ab) = f (a), nên

f (a) = f (ab) = f (ab f (ab)) = f (ab f (a)) = f (ab2),

như vậy f (x) cũng xác định tại x = ab2, f (ab2) = f (a).Bằng quy nạp ta có thể thấy f (x) xác định với x = abn,n ∈ N∗ theo công thức

b = f (a) = f (abn−1) = f (abn−1 f (abn−1))

= f (abn−1 f (a)) = f (abn−1b) = f (abn).

Ta thấy b ∈ [0,1], vì nếu b < 0 thì ab < 0, nếu b > 1 thì khi chọn 1 > a > 0 ta có abn > 1với n ∈ N đủ lớn, mâu thuẫn với khẳng định f xác định tại abn,∀n ∈ N∗ ở trên.Nếu tồn tại a ∈ [0,1] mà 0 < f (a) = b < 1 thì do f liên tục nên

b = f (a) = f (abn) = limn→+∞

f (abn) = f ( limn→+∞

abn) = f (0).

Như vậy, trên [0,1] hàm f nhận không quá 3 giá trị là 0,1, f (0). Nhưng do f là hàm liêntục nên f = const trên [0,1]. Đây là nghiệm của bài toán.

Bài toán 3.1.7. Cho a ∈ R,a 6=±1. Tìm hàm f liên tục trên R+ thỏa mãn

f (xa) = f (x),∀x ∈ R+.

LỜI GIẢI. Với a = 0 thì bài toán có nghiệm f (x)≡ c, với c ∈ R tùy ý.Xét bài toán khi a 6= 0. Ta chỉ ra nghiệm của bài toán là f là hàm hằng.Ta có f (x) = f (xa),∀x ∈ R+ nên f (x) = f (x

1a ),∀x ∈ R+. Do đó, ta chỉ cần quan tâm

trường hợp |a|< 1.Ta có f (x) = f (xa) = f (xa2

) = ... = f (xan),∀n∈N∗. Do |a|< 1 nên an→ 0 khi n→+∞,

suy ra với mọi x ∈ R+ thì xan → 1 khi n→+∞. Kết hợp giả thiết f là hàm liên tục ta có

f (x) = limn→+∞

f (xan) = f ( lim

n→+∞xan

) = f (1),∀x ∈ R+.

Do đó f (x) là hàm hằng trên R+ (ĐPCM).Vậy nghiệm của bài toán là f (x)≡ c,∀x ∈ R+, với c ∈ R tùy ý.

33

Page 39: Đ—I H¯C QU¨C GIA HÀ N¸I TRƯ˝NG Đ—I H¯C KHOA H¯C TÜ NHIÊN KHOA … · 2016-05-09 · Đ—I H¯C QU¨C GIA HÀ N¸I TRƯ˝NG Đ—I H¯C KHOA H¯C TÜ NHIÊN KHOA TOÁN

Bài toán 3.1.8. Tìm tất cả các hàm số xác định và liên tục trên R thỏa mãn

f (x2)+ f (x) = x2 + x, ∀x ∈ R.

LỜI GIẢI. Dễ thấy rằng f (0) = 0. Đặt f (x)−x = g(x). Khi đó g xác định và liên tục trênR thỏa mãn g(x2)+g(x) = 0,∀x ∈ R, đặc biệt g(0) = 0. Ta có g(x2)+g(x) = 0,∀x ∈ Rnên g(x) = g(−x),∀x ∈R. Như vậy g là hàm chẵn, ta chỉ cần đi xác định hàm g trên R+.Ta có g(x2)+ g(x) = 0,∀x ∈ R nên g(x) = −g(x2) = g(x4). Đến đây, áp dụng bài toántrên, ta suy ra g là hàm hằng trên R+. Nhưng g(0) = 0 nên g(x) = 0,∀x ∈ R+, ngoài rag là hàm chẵn nên g(x) = 0,∀x ∈ R. Do đó f (x) = g(x)+ x = x,∀x ∈ R.

Bài toán 3.1.9. (VietNam 2001 - Bảng A) Cho hàm g(x) =2x

1+ x2 . Hãy tìm tất cả các

hàm f (x) xác định và liên tục trên khoảng (−1,1) thỏa mãn

(1− x2) f (g(x)) = (1+ x2)2 f (x), ∀x ∈ (−1,1).

LỜI GIẢI. Viết lại điều kiện của bài toán

(1− x2)2

(1+ x2)2 f (g(x)) = (1− x2) f (x),∀x ∈ (−1,1).

Đặt ϕ(x) = (1− x2) f (x), x ∈ (−1,1). Khi đó ϕ(x) liên tục trên (−1,1) thỏa mãnϕ(g(x)) = ϕ(x),∀x ∈ (−1,1) (∗).Dễ thấy u(x) =

1− x1+ x

là một song ánh từ (0,+∞) vào (−1,1).

Vì thế có thể viết lại (*) như sau

ϕ(g(1− x1+ x

)) = ϕ(1− x1+ x

) ⇐⇒ ϕ(1− x2

1+ x2 ) = ϕ(1− x1+ x

),∀x ∈ (0,+∞).

Xét hàm số h(x) = ϕ(1− x1+ x

),∀x ∈ (0,+∞). Khi đó hàm số h(x) liên tục thỏa mãn

h(x2) = h(x), ∀x > 0. Đây là bài toán quen thuộc và có nghiệm h(x) = const trên R+.

Nên ϕ(x) = const trên (−1,1). Vì vậy f (x) =c

1− x2 , với c ∈ R tùy ý.

Thử lại thấy rằng đây là nghiệm của bài toán.

Bài toán 3.1.10. Tìm f (x) liên tục trên R thỏa mãn f (x2) f (x) = 1,∀x ∈ R.

LỜI GIẢI. Từ giả thiết suy ra f (x) 6= 0,∀x ∈ R và f (0) =±1, f (1)±1.

Ta có f (x2) f (x) = 1 = f (x2) f (−x),∀x ∈ R suy ra f (x) = f (−x),∀x ∈ R. Do đó ta chỉcần xét hàm f trên R+.

Với mọi x ∈ R+ ta có f (x) =1

f (x2)= f (x4).

Như vậy f (x) = f (x4),∀x > 0. Đây là bài toán quen thuộc, có nghiệm là

34

Page 40: Đ—I H¯C QU¨C GIA HÀ N¸I TRƯ˝NG Đ—I H¯C KHOA H¯C TÜ NHIÊN KHOA … · 2016-05-09 · Đ—I H¯C QU¨C GIA HÀ N¸I TRƯ˝NG Đ—I H¯C KHOA H¯C TÜ NHIÊN KHOA TOÁN

f (x) = f (1) =±1,∀x > 0. Do f là hàm chẵn nên f (x) =±1,∀x 6= 0.

Vậy với mọi x ∈ R thì f (x) = 1 hoặc f (x) = −1. Nhưng f là hàm liên tục trên R nênf (x) = 1,∀x ∈ R hoặc f (x) =−1,∀x ∈ R.Nhận xét: Nếu ta chỉ giả thiết f liên tục trên R\{0} thỏa mãn f (x2) f (x) = 1, với mọisố thực x 6= 0, thì nghiệm của trường hợp này là f (x) = c,∀x 6= 0, với c =±1.

Bài toán 3.1.11. Tìm tất cả các hàm f xác định và liên tục trên R\{0} thỏa mãn

( f (x2)− x2)( f (x)− x) =1x3 , ∀x 6= 0. (1)

LỜI GIẢI. Ta có (1) ⇐⇒ (x2 f (x2)− x4)(x f (x)− x2) = 1. Đặt x f (x)− x2 = g(x) thì g

là hàm liên tục trên R \ {0}. Hơn nữa g(x2)g(x) = 1,∀x 6= 0. Sử dụng nhận xét của bàitoán trên, ta suy ra g(x) = c,∀x 6= 0,c =±1. Do đó x f (x)− x2 = c,∀x 6= 0.

Suy ra f (x) =cx

+ x,c =±1,∀x 6= 0. Thử lại thỏa mãn bài toán.

Vậy hàm cần tìm là f (x) =cx

+ x,c =±1 trên R\{0}.

Bài toán 3.1.12. (Bulgaria 1997) Tìm hàm số f (x) liên tục trên R thỏa mãn

f (x) = f (x2 +14), ∀x ∈ R.

LỜI GIẢI. Dễ thấy rằng f (x) là hàm số chẵn. Vì thế ta chỉ cần xác định f (x) với x ≥ 0.

Ta xét 2 trường hợp sau:- Với mỗi 0≤ x0 ≤ 1/2. Xác định dãy xn, n ∈ N, bởi xn+1 = x2

n +1/4. Bằng quy nạp tachứng minh được 0≤ xn ≤ 1/2,∀n ∈ N. Hơn nữa, ∀n ∈ N ta có

xn+1− xn = x2n− xn +1/4 = (xn−1/2)2 ≥ 0.

Suy ra dãy xn là dãy không giảm và bị chặn trên bởi 1/2. Do đó tồn tại a≤ 1/2 sao cholim

n→+∞xn = a, với a là nghiệm của phương trình a2−a+1/4 = 0 ⇐⇒ a = 1/2.

Ta có f (x0) = f (xn),∀n ∈ N. Mặt khác, do hàm f liên tục nên

f (x0) = limn→+∞

f (xn) = f ( limn→+∞

xn) = f (12).

Vậy f (x) = f (1/2), ∀x ∈ [0,1/2].- Với x0 > 1/2. Xét dãy xn, n∈N như sau xn+1 =

√xn−1/4 hay xn = x2

n+1 +1/4. Tươngtự như trên, ta chứng minh được xn là dãy giảm bị chặn dưới bởi 1/2, lim

n→+∞xn = 1/2. Và

ta cóf (x0) = lim

n→+∞f (xn) = f ( lim

n→+∞xn) = f (

12).

Nên với mọi x > 1/2 thì f (x) = f (1/2).Vậy với mọi x ∈ R thì f (x) = f (1/2), hay f (x) = c,∀x ∈ R, với c là hằng số thực tùy ý.

35

Page 41: Đ—I H¯C QU¨C GIA HÀ N¸I TRƯ˝NG Đ—I H¯C KHOA H¯C TÜ NHIÊN KHOA … · 2016-05-09 · Đ—I H¯C QU¨C GIA HÀ N¸I TRƯ˝NG Đ—I H¯C KHOA H¯C TÜ NHIÊN KHOA TOÁN

Đây là nghiệm của bài toán.Nhận xét: Về bản chất, bài toán này cũng không khác nhiều so với những bài toán đãxét ở trên, nhưng ý tưởng sử dụng tính liên tục của hàm số được giấu kín hơn. Ta hoàntoàn có thể thay hàm x2 +1/4 bởi hàm g(x) sao cho: ta có thể chia R thành các khoảngthích hợp, mà với x0 trên mỗi khoảng này thì dãy xn định nghĩa bởi xn+1 = g(nn) hoặcxn = g(xn+1) hội tụ đến cùng một giá trị. Khi đó, do tính liên tục của hàm f ta suy ra f

là hàm hằng trên mỗi khoảng này. Bài toán dạng này mang dấu ấn của giới hạn dãy số.Với nhận xét trên đây ta có thể giải một số bài toán tương tự sau dễ dàng hơn.

Bài toán 3.1.13. (VietNam TST 2007) Tìm hàm f liên tục trên R thỏa mãn

f (x) = f (x2 +13

x+19), ∀x ∈ R.

LỜI GIẢI. Trước tiên, ta thấy rằng với mọi x≤ 0 thì x2 +13

x+19

> 0. Do đó, ta sẽ đi xác

định f trên R+ trước.Ta sẽ chứng minh f (x) là hàm hằng trên R+.

- Với 0 < x0 ≤ 1/3 bất kì. Ta xét dãy xn+1 = x2n +

13

x+19. Ta có

xn+1− xn = x2n−

23

xn +19

= (xn−13)2 ≥ 0,∀n ∈ N,

nên xn là dãy tăng. Mặt khác

xn+1−13

= xn−13

xn−29

= (xn−13)(xn +

23),∀n ∈ N,

và 0 < x0 ≤ 1/3. Bằng quy nạp thấy rằng 0 < xn ≤ 1/3,∀n ∈ N. Do đó, dãy xn là dãytăng và bị chặn trên bởi 1/3 nên có giới hạn, giới hạn này là nghiệm dương và≤ 1/3 củaphương trình x = x2 + x/3+1/9, do đó limxn = 1/3 khi n→+∞. Sử dụng tính liên tụccủa f ta có

f (x0) = limn→+∞

f (xn) = f ( limn→+∞

xn) = f (1/3).

Do đó, f = f (1/3) trên (0,1/3].- Với x0 > 1/3. Ta xét hàm f (x) = x2 + x/3 + 1/9 trên khoảng x > 1/3, thấy rằng hàmnày nghịch biến, nên đơn ánh trên khoảng x > 1/3. Do đó, ta có thể định nghĩa dãy

xn,n ∈ N bởi xn = x2n+1 +

xn+1

3+

19. Tương tự như trên, ta cũng dễ dàng chứng minh

được xn là dãy giảm và bị chặn dưới bởi 1/3, nên dãy tồn tại limxn, giới hạn này cũng lànghiệm ≥ 1/3 của phương trình x2 +x/3+1/9 = x, do đó limxn = 1/3. Từ đây sử dụngtính liên tục của f ta cũng suy ra f (x0) = f (1/3) với mỗi x0 > 1/3, hay f (x) = f (1/3)trên khoảng x > 1/3.

36

Page 42: Đ—I H¯C QU¨C GIA HÀ N¸I TRƯ˝NG Đ—I H¯C KHOA H¯C TÜ NHIÊN KHOA … · 2016-05-09 · Đ—I H¯C QU¨C GIA HÀ N¸I TRƯ˝NG Đ—I H¯C KHOA H¯C TÜ NHIÊN KHOA TOÁN

Như vậy f (x) = f (1/3),∀x > 0. Ta thấy rằng với mọi x≤ 0 thì tồn tại x2 +x/3+1/9 > 0để f (x) = f (x2 +x/3+1/9) = f (1/3), suy ra f (x) = f (1/3), ∀x ∈R. Hay nghiệm củabài toán là f (x) = const trên R.

Bài toán 3.1.14. Tìm tất cả các hàm số liên tục f : R+→ R+ thỏa mãn

f (x) = f (x+1x+2

), ∀x > 0.

LỜI GIẢI. Tương tự bài toán trên, ta sẽ chứng minh f là hàm hằng trên R+.

Đặt hàm g(x) =x+1x+2

, x > 0. Ta có g′(x) =1

(x+2)2 <14,∀x > 0.

Với mỗi giá trị x0 > 0, ta xây dựng dãy xn,n ∈ N như sau xn+1 = g(xn) =xn +1xn +2

.

Gọi a > 0 là nghiệm dương của phương trình x = g(x), hay a =−1+

√5

2.

Theo định lý Lagrange ta có |xn+1− a| = |g(xn)− g(a)| = |g′(cn)(xn− a)|,∀n ∈ N, vớicn là giá trị nào đó nằm giữa xn và a, rõ ràng cn > 0,∀n ∈ N.

Do g′(x) <14,∀x > 0 nên |xn+1−a|< 1

4|xn−a|< ... < (

14)n+1|x0−a|.

Do đó limn→+∞

|xn−a|= 0, suy ra limn→+∞

xn = 0 với mọi giá trị x0 > 0.

Ta thấy rằng f (x0) = f (xn),∀n ∈ N, nên sử dụng tính liên tục của hàm số f (x) ta có

f (x0) = limn→+∞

f (xn) = f ( limn→+∞

xn) = f (a).

Do đó f là hàm hằng trên R+. Đây là nghiệm của bài toán.

Nhận xét: Ở bài toán này, nếu không dùng định lý Lagrange để xét tính hội tụ của dãyxn+1 = g(xn), thì ta hoàn toàn có thể làm tương tự bài toán trước, bằng cách xét 2 trường

hợp x0 <−1+

√5

2và x0 ≥

−1+√

52

.

Bài toán 3.1.15. (Croatia 1996) Cho t ∈ (0,1). Tìm hàm f liên tục trên R thỏa mãn

f (x)−2 f (tx)+ f (t2x) = x2, ∀x ∈ R.

LỜI GIẢI. Giả sử f là hàm thỏa mãn bài toán.Đặt f (x)− f (tx) = g(x), thế thì g liên tục, g(0) = 0 và g(x)−g(tx) = x2,∀x ∈ R.

Lần lượt thay x bởi tx, t2x, ..., tn−1x ta suy ra

g(tx)−g(t2x) = t2x2

... g(tn−1)−g(tnx) = t2(n+1)x2.

Cộng n đẳng thức trên ta suy ra với n ∈ N∗ thì

g(x)−g(tnx) = x2(1+ t2 + ...+ t2(n+1)) = x2 1− t2n

1− t2 .

37

Page 43: Đ—I H¯C QU¨C GIA HÀ N¸I TRƯ˝NG Đ—I H¯C KHOA H¯C TÜ NHIÊN KHOA … · 2016-05-09 · Đ—I H¯C QU¨C GIA HÀ N¸I TRƯ˝NG Đ—I H¯C KHOA H¯C TÜ NHIÊN KHOA TOÁN

Sử dụng tính liên tục của g với chú ý 0 < t < 1, trong đẳng thức trên cho n→+∞ ta có

g(x)−g(0) =x2

1− t2 , do g(0) = 0 nên g(x) =x2

1− t2 .

Do đó f (x)− f (tx) =x2

1− t2 . Tương tự như đối với hàm g, ta cũng suy ra

f (x)− f (tnx) =x2

1− t2 (1+ t2 + ...+ t2(n−1)) = x2 1− t2n

(1− t2)2 .

Cho n→+∞ ta có f (x)− f (0) =x2

(1− t2)2 . Hay f (x) =x2

(1− t2)2 +c,∀x ∈R, với c ∈R

tùy ý. Thử lại, đây là nghiệm của bài toán.

Để kết thúc mục này ta đi xét đến các bài toán sử dụng đến định lý "giá trị trunggian" của hàm số.

Bài toán 3.1.16. Chứng minh rằng với mọi hàm f : [0,1]→ [0,1] liên tục thì phương

trình f (x) = x có nghiệm trên [0,1].

LỜI GIẢI. Đặt g(x) = f (x)− x thì g(x) là hàm số liên tục trên [0,1].Ta có g(0) = f (0)≥ 0 và g(1) = f (1)−1≤ 0, kết hợp g là liên tục trên [0,1] nên tồn tạix ∈ [0,1] sao cho g(x) = 0 hay f (x) = x (ĐPCM).

Bài toán 3.1.17. Cho hai hàm số f ,g liên tục trên I = [0,1] thỏa mãn

(i). f (g(x)) = g( f (x)), ∀x ∈ I,

(ii). f là hàm không giảm trên I.

Chứng minh rằng tồn tại a ∈ I sao cho f (a) = g(a) = a.

LỜI GIẢI. Theo bài toán trên thì phương trình g(x) = x có nghiệm trên I. Ta gọi nghiệmnày là x0. Khi đó, nếu f (x0) = x0 thì bài toán được chứng minh.Ta xét trường hợp f (x0) 6= x0. Ta xây dựng dãy số xn xác định bởi xn+1 = f (xn),n∈N (1).Bằng quy nạp, ta chứng minh được rằng g(xn) = xn,∀n ∈ N (2).Ta đi chứng minh dãy xn hội tụ đến a ∈ I, khi đó từ (1) lấy giới hạn hai vế ta suy raf (a) = a, tương tự thì từ (2) ta cũng có g(a) = a. Như vậy f (a) = g(a) = a.Ta có f : I→ I nên xn là dãy bị chặn, chặn trên bởi 1 và chặn dưới bởi 0. Ngoài ra, f làhàm không giảm nên dễ thấy xn là dãy không giảm nếu x0 ≤ x1, hoặc xn là dãy khôngtăng nếu x0 > x1. Do đó dãy xn hội tụ đến a ∈ I. Theo trên ta có ĐPCM.Nhận xét. Bài toán không thay đổi nếu ta thay I = [0,1] bởi đoạn [a,b] với a < b bất kì

Bài toán 3.1.18. Cho f liên tục trên R và thỏa mãn hai điều kiện

(i). f (2012) = 2011,

(ii). f (x) f 4(x) = 1, ∀x ∈ R, ở đây f n(x) = f ( f ...( f (x))), n lần f .

Tính giá trị của f (2011).

38

Page 44: Đ—I H¯C QU¨C GIA HÀ N¸I TRƯ˝NG Đ—I H¯C KHOA H¯C TÜ NHIÊN KHOA … · 2016-05-09 · Đ—I H¯C QU¨C GIA HÀ N¸I TRƯ˝NG Đ—I H¯C KHOA H¯C TÜ NHIÊN KHOA TOÁN

LỜI GIẢI. Gọi D là miền giá trị của f trên R. Từ (ii) ta có x f 3(x) = 1,∀x ∈ D (1).

Mặt khác từ (i) ta có 2011 ∈ D. Trong (ii) lấy x = 2012 ta có f 4(2012) =1

2011. Suy ra

12011

∈ D. Do f liên tục nên I = [1

2011,2011]⊆ D (2).

Từ (1) suy ra f đơn ánh trên D, mà f liên tục nên f đơn điệu trên D. Ta thấy rằngf không thể đơn điệu tăng trên D, vì nếu không f 3 cũng tăng trên D, mâu thuẫn với

f 3(x) =1x,∀x ∈ D. Do vậy f đơn điệu giảm trên D. Nói riêng, f đơn điệu giảm trên I.

Ta chứng minh f (x) =1x,∀x ∈ I.

Thật vậy, giả sử tồn tại x0 ∈ D mà f (x0) >1x0

, do f giảm nên

f 2(x0) < f (1x0

) (3)

⇒ 1x0

= f 3(x0) > f 2(1x0

)⇒ f (1x0

) < f 3(1x0

) = x0. (4)

Từ (3) và (4) ta có x0 > f 2(x0)⇒ f (x0) < f 3(x0) =1x0

, mâu thuẫn với giả sử trên.

Vậy không tồn tại x0 ∈ I mà f (x0) >1x0

.

Tương tự, ta cũng có khẳng định không tồn tại x0 ∈ I mà f (x0) <1x0

.

Vậy f (x) =1x, ∀x ∈ I. Từ (2) ta suy ra f (2011) =

12011

.

Bài toán 3.1.19. Tìm hàm số f : R→ R liên tục thỏa mãn f (0) = 0 và

f (x) = x+ f (x− f (x)), ∀x ∈ R.

LỜI GIẢI. Đặt x− f (x) = g(x) thì f (g(x)) =−g(x). Ta có g liên tục trên R, và

g(0) = 0, g(g(x)) = g(x)− f (g(x)) = 2g(x), ∀x ∈ R.

Từ đó, nếu g(x) > 0 tại x = x0 nào đó thì g(x) có thể nhận giá trị dương lớn tùy ý, cònnếu g(x) có giá trị âm thì nó nhận giá trị âm nhỏ tùy ý.Vì g(x) liên tục nên nếu g(x) nhận giá trị a và b thì nó nhận giá trị tùy ý giữa a và b. Nênta xét 4 trường hợp sau đây :

*) g(x) = 0,∀x ∈ R, thì f (x) = x là nghiệm của bài toán.*) g(x)≥ 0,∀x ∈ R. Khi đó ∀x≥ 0 tồn tại y để g(y) = x nên

f (x) = f (g(y)) =−g(y) =−x.

Với x < 0 thì do g(x) ≥ 0 ta phải có f (x) ≤ x. Như vậy, với x ≥ 0 thì f (x) = −x,với x < 0 thì f (x) ≤ x. Ta chỉ ra hàm này thỏa mãn bài toán. Thật vậy: Nếu x ≥ 0

39

Page 45: Đ—I H¯C QU¨C GIA HÀ N¸I TRƯ˝NG Đ—I H¯C KHOA H¯C TÜ NHIÊN KHOA … · 2016-05-09 · Đ—I H¯C QU¨C GIA HÀ N¸I TRƯ˝NG Đ—I H¯C KHOA H¯C TÜ NHIÊN KHOA TOÁN

thì f (x− f (x)) + x− f (x) = f (2x) + 2x = 0. Còn nếu x < 0, do x− f (x) ≥ 0 nênf (x− f (x))+ x− f (x) = f (x)− x+ x− f (x) = 0.

*) g(x)≤ 0,∀x ∈R. Tương tự, ta suy ra hàm cần tìm là f liên tục, f (x) =−x,∀x≤ 0,

và f (x)≥ x,∀x > 0.

*) g(x) nhận giá trị tùy ý trên R. Khi đó, ∀x ∈ R tồn tại y ∈ R sao cho g(y) = x.

Và ta có f (x) = f (g(y)) =−g(y) =−x,∀x ∈ R.

Vậy bài toán có 4 nghiệm, được xác định như trên.

MỘT SỐ BÀI TẬP VẬN DỤNG:Bài tập 3.1.1. Tìm hàm f xác định và liên tục trên R+ thỏa mãn

f (x3)− x2 f (x) =1x3 − x, ∀x > 0.

Gợi ý. Đặtf (x)

x− 1

x2 = g(x) thì g liên tục trên R+, đồng thời g(x3) = g(x),∀x > 0.

Từ đó g(x) = g(1) = c, với c ∈ R bất kì. Suy ra f (x) = cx+1x, ∀x > 0.

Bài tập 3.1.2. Tìm hàm f xác định và liên tục trên R+ thỏa mãn

( f (x3)− x6)( f (x2)− x4) = x5, ∀x > 0.

Gợi ý. Đặt g(x) =f (x)

x− x,∀x > 0.

Bài tập 3.1.3. Tìm tất cả các hàm f : R+→ R+ liên tục thỏa mãn

f (x) = f (x(x2 +3.20112)

3x2 +20112 ), ∀x > 0.

Gợi ý. Với x > 0 xét dãy x0 = x,xn+1 =xn(x2

n +3.20112)3x2

n +20112 .

Giải phương trình x =x(x2 +3.20112)

3x2 +20112 với x > 0 ta được nghiệm a = 2011.

- Nếu 0 < x < 2011 thì chứng minh dãy xn tăng và bị chặn trên bởi 2011. Dẫn đến xn hộitụ đến 2011.

- Nếu x ≥ 2011 thì chứng minh dãy xn không giảm và bị chặn dưới bởi 2011. Từ đó xn

hội tụ đến 2011.

Từ đó, sử dụng tính chất hội tụ của f (x) ta suy ra nghiệm của bài toán là f (x)≡ 0,∀x > 0.

Bài tập 3.1.4. Tìm tất cả các hàm số liên tục thỏa mãn

f (x) = f (1− cosx),∀x ∈ R.

40

Page 46: Đ—I H¯C QU¨C GIA HÀ N¸I TRƯ˝NG Đ—I H¯C KHOA H¯C TÜ NHIÊN KHOA … · 2016-05-09 · Đ—I H¯C QU¨C GIA HÀ N¸I TRƯ˝NG Đ—I H¯C KHOA H¯C TÜ NHIÊN KHOA TOÁN

Gợi ý. Xét dãy xn+1 = 1− cosxn, n ∈ N với x0 tùy ý. Ta chứng minh xn hội tụ đến 0.

Từ đó sử dụng tính liên tục của hàm số suy ra f (x) = const.

Bài tập 3.1.5. (Putnam 1996) Cho số thực c ≥ 0. Hãy xác định tất cả các hàm số liên

tục f : R→ R thỏa mãn f (x) = f (x2 + c), ∀x ∈ R.

Gợi ý. Bạn đọc có thể xem lời giải cụ thể trong [6], Exemple 3.3 trang 60.

Bài tập 3.1.6. Tìm hàm f liên tục trên R\{0} thỏa mãn f (x) f (1x) = 1, ∀x 6= 0.

Gợi ý. Chứng minh mọi nghiệm của bài toán đều có dạng f (x) =

√g(x)g(1

x), với g(x) là

hàm liên tục tùy ý thỏa mãn g(x) > 0,∀x 6= 0.

Bài tập 3.1.7. (Mock Summer Camp 2008) Tìm f : R→ R liên tục thỏa mãn

f (x+ y) = f (x)+ f (y)+ f (x) f (y), ∀x,y ∈ R.

Bài tập 3.1.8. Giả sử f : R→R liên tục và f (x+y) f (x−y) = [ f (x)]2,∀x,y ∈R. Chứng

minh rằng: hoặc f (x)≡ 0 hoặc phương trình f (x) = 0 không có nghiệm.

Gợi ý. Cho x = y = ta có f (2x) f (0) = [ f (x)]2. Nếu f (0) = 0 thì ta sẽ suy ra f (x)≡ 0.

Nếu f (0) 6= 0, ta giả sử phản chứng tồn tại x sao cho f (x) = 0. Khi đó suy ra f (x/2) = 0,bằng quy nạp ta có f (x/2n) = 0,∀n ∈ N. Từ đó sử dụng tính liên tục suy ra f (0) = 0,

mâu thuẫn. Vậy giả sử trên là sai. Hay f (x) = 0 không có nghiệm.

Bài tập 3.1.9. Cho hàm số f (x) liên tục trên đoạn [0,1] thỏa mãn f (0) = f (1). Chứng

minh rằng phương trình f (x) = f (x+1

2011) có nghiệm x ∈ [0,1].

Gợi ý. Thay 2011 bởi 2≤ k ∈ N. Xét hàm g(x) = f (x+1k)− f (x).

Bài tập 3.1.10. Chứng minh rằng không thể tồn tại hàm số liên tục f : R→ R có tính

chất: f (x) hữu tỉ khi và chỉ khi f (x+1) vô tỉ.

Gợi ý. Xét các hàm số g(x) = f (x+1)− f (x) và h(x) = f (x+1)+ f (x). Chúng không

thể cùng là hàm hằng, vì nếu không f (x) =h(x)−g(x)

2là hàm hằng, mâu thuẫn. Giả sử,

chẳng hạn, h(x) không là hàm hằng, nghĩa là tồn tại x1 6= x2 sao cho h(x1) < h(x2). Khiđó, tồn tại số hữu tỉ r ∈ [h(x1),h(x2)], vì h(x) liên tục nên tồn tại x0 thỏa mãn h(x0) = r.

Như vậy f (x0 + 1)+ f (x0) = r, mâu thuẫn vì f (x0 + 1), f (x0) không cùng là số hữu tỉhoặc vô tỉ. Vậy không tồn tại hàm thỏa mãn bài toán.

41

Page 47: Đ—I H¯C QU¨C GIA HÀ N¸I TRƯ˝NG Đ—I H¯C KHOA H¯C TÜ NHIÊN KHOA … · 2016-05-09 · Đ—I H¯C QU¨C GIA HÀ N¸I TRƯ˝NG Đ—I H¯C KHOA H¯C TÜ NHIÊN KHOA TOÁN

Bài tập 3.1.11. Tìm hàm f : [0,1]→ R liên tục thỏa mãn

f (x) =12[ f (

x2)+ f (

1+ x2

)], ∀x ∈ [0,1].

Gợi ý. f (0) = f (1/2) = f (1). Do f liên tục nên đạt GTLN, GTNN trên [0,1]. Tachứng minh f (x) = f (0),∀x ∈ (0,1). Bằng cách chỉ ra f (0) = f (1) cùng là giá trị lớnnhất và giá trị nhỏ nhất của f (x) trên [0,1].

Bài tập 3.1.12. Chứng minh rằng không tồn tại hàm f xác định và liên tục trên R thỏa

mãn f ( f (x)) =−x, ∀x ∈ R.

Gợi ý. Sử dụng tính chất f liên tục và đơn ánh thì đơn điệu.

Bài tập 3.1.13. (Olympic SV 2003) Tìm tất cả các hàm f (x) xác định và liên tục trên Rthỏa mãn f (x+2002)( f (x)+

√2003) =−2004, ∀x ∈ R.

Gợi ý. Giả sử tồn tại hàm f thỏa mãn bài toán. Khi đó f (x) 6= 0 và f (x) 6= −√

2003trên R. Vì f liên tục nên chỉ xảy ra 3 trường hợp với với miền giá trị D của f như sau:- Nếu D⊂ (−∞,−

√2003) thì f (x+2002)( f (x)+

√2003) > 0 >−2004.

- Nếu D⊂ (−√

2003,0) thì | f (x+2002)|<√

2003, | f (x)+√

2003|<√

2003 nên| f (x+2002)( f (x)+

√2003)|< 2003 < 2004,∀x ∈ R.

- Nếu D⊂ (0,+∞) thì f (x+2008)( f (x)+√

2003) > 0 >−2004,∀x ∈R. Như vậy cả 3trường hợp đều mâu thuẫn với điều kiện. Kết luận, bài toán không có nghiệm !

Bài tập 3.1.14. Cho f (x),g(x) là các hàm số xác định và liên tục trên R thỏa mãn

f (g(x)) = g( f (x)). Chứng minh rằng: nếu phương trình f (x) = g(x) vô nghiệm thì

phương trình f ( f (x)) = g(g(x)) cũng vô nghiệm.

Bài tập 3.1.15. Giả sử hàm số f (x) liên tục trên R, nhận giá trị khác dấu. Chứng minh

rằng tồn tại một cấp số cộng a < b < c sao cho f (a)+ f (b)+ f (c) = 0.

Gợi ý. Có x0 sao cho f (x0) > 0. Do f liên tục trong một lân cận của điểm này vàf (x0) > 0 nên ta tìm được một cấp số cộng a0,b0,c0 sao cho f (a0)+ f (b0)+ f (c0) > 0.Tương tụ, ta tìm được một cấp số cộng a1,b1,c1 sao cho f (a1) + f (b1) + f (c1) < 0.

Với tham số t ∈ [0,1] ta xét cấp số cộng a(t) như sau s(t) = a0(1− t)+ a1t, tương tựvới b(t),c(t). Xét hàm số F(t) = f (a(t)) + f (b(t)) + f (c(t)) là một hàm số liên tục,F(0) < 0,F(1) > 0. Do đó, tồn tại t0 ∈ [0,1] sao cho F(t0) = 0. Khi đó, cấp số cộng cầntìm là a(t0),b(t0),c(t0).

42

Page 48: Đ—I H¯C QU¨C GIA HÀ N¸I TRƯ˝NG Đ—I H¯C KHOA H¯C TÜ NHIÊN KHOA … · 2016-05-09 · Đ—I H¯C QU¨C GIA HÀ N¸I TRƯ˝NG Đ—I H¯C KHOA H¯C TÜ NHIÊN KHOA TOÁN

3.2 Phương pháp qui nạp toán học

Nguyên lý qui nạp toán học: Ta xét khẳng định S(k) mà tính đúng đắn phụ thuộcvào k ∈ N. Giả sử S(k) đúng với giá trị ban đầu k = n0 ∈ N và nếu S(k) đúng với k = n

thì S(k) cũng đúng với k = n + 1. Khi đó ta sẽ có khẳng định S(k) đúng với mọi k màn0 ≤ k ∈ N.

Một dạng khá đặc biệt của nguyên lý qui nạp toán học thường được gọi với cáitên "Qui nạp kiểu Cauchy" phát biểu như sau: Giả sử S(k) đúng với giá trị ban đầuk = n0 ∈N ta sẽ suy ra S(k) đúng với k = fi(n0)≥ n0, ∀i∈N trong đó ta có fi(n0)→+∞

khi i→ +∞. Và từ S(k) đúng với k = fi(n0) ta suy ra S(k) cũng đúng với mọi k màn0 ≤ k ≤ fi(n0). Khi đó S(k) đúng với mọi k ≥ n0.

Ta có thể thấy rằng PP Qui nạp toán học sẽ thường áp dụng cho các bài toán PTHtrên tập số tự nhiên. Tuy nhiên, nó cũng có ích trong các bài toán PTH khi hàm số xácđịnh trên Z, Q, R, với ý tưởng qui nạp từng bước N→ Z→ Q→ R. Từ bước N→ Zta thường quan tâm đến tính chẵn lẻ của hàm số. Riêng bước từ Q→ R ta cần thêm dữkiện, chẳng hạn: hàm đang xét liên tục, tăng thực sự hay đơn điệu,... (Phần này chúng tasẽ xét nhiều hơn trong phần PTH Cauchy hay PP sử dụng tính đơn điệu của hàm số).

Ta đi xét một vài ví dụ để nắm vững phương pháp cơ bản và hữu ính này.

Bài toán 3.2.1. (VietNam 1985) Tìm f : Z→ Z thỏa mãn f (0) 6= 0, f (1) = 5/2 và

f (n) f (m) = f (n+m)+ f (n−m), ∀n,m ∈ Z.

LỜI GIẢI. Lấy n = m = 0 ta suy ra [ f (0)]2 = 2 f (0), do f (0) 6= 0 nên f (0) = 2. Lấym = 1 ta có

f (n) f (1) = f (n+1)+ f (n−1), ∀n ∈ Z.

Ta thấy rằng f xác định hoàn toàn khi biết f (0) = 2 và f (1) = 5/2. Ta sẽ chứng minhqui nạp f (n) = 2n + 2−n, ∀n ∈ Z, (∗). Ta chỉ cần chứng minh đúng với n ∈ N vì nếulấy n = 0 ta có f là hàm chẵn. Ta có (*) đúng với n = 0, n = 1. Giả sử (*) đúng đếnn = k, 1≤ k ∈ N ta chứng minh (*) cũng đúng với n = k +1. Thật vậy

f (k +1) =52(2k +2−k)− (2k−1 +21−k) = 2k+1 +2−k−1.

Do đó theo nguyên lý qui nạp toán học ta có f (n) = n ∈ N. Và do đó nó cũng đúng vớin ∈ Z vì f và hàm 2n +2−n đều là hàm chẵn. Kết luận f (n) = 2n +2−n, ∀n ∈ Z.

Bài toán 3.2.2. Tìm hàm f : N∗→ N∗ thỏa mãn f (2) = 2, f (m) < f (n), ∀m < n và

f (nm) = f (n) f (m) với mọi n,m ∈ N thỏa mãn (m,n)=1.

43

Page 49: Đ—I H¯C QU¨C GIA HÀ N¸I TRƯ˝NG Đ—I H¯C KHOA H¯C TÜ NHIÊN KHOA … · 2016-05-09 · Đ—I H¯C QU¨C GIA HÀ N¸I TRƯ˝NG Đ—I H¯C KHOA H¯C TÜ NHIÊN KHOA TOÁN

LỜI GIẢI. Ta có f (3) f (5) = f (15) < f (18) = f (2) f (9) < f (2) f (10) = f (2) f (2) f (5) =4 f (5)⇒ f (3) < 4. Do đó f (3) = 3. Như vậy ta có f (2) = 2, f (3) = 3. Ta xét dãy an

như sau: a0 = 3, an+1 = an(an− 1), ∀n ∈ N. Khi đó bằng quy nạp chúng ta dễ thấyrằng: f (an) = an và lim

n→+∞an = +∞. Kết hợp giả thiết f tăng trên N ta suy ra nghiệm

của bài toán là f (n) = n, ∀n ∈ N∗.

Bài toán 3.2.3. (Putnam 1963) Tìm tất cả các hàm f : N→ N tăng thực sự thỏa mãn

các điều kiện f (2) = 2 và f (mn) = f (m) f (n), ∀m,n ∈ N.

LỜI GIẢI. Do f tăng thực sự nên 0≤ f (0) < f (1) < f (2) = 2. Do đó f (0) = 0, f (1) = 1.

Giả sử f (3) = 3 + k,k ∈ N, suy ra f (6) = f (2) f (3) = 6 + 2k nên f (5) ≤ 5 + 2k. Ta cóf (10) = f (2) f (5)≤ 10+4k, suy ra f (9)≤ 9+4k, f (18)≤ 18+8k nên f (15)≤ 15+8k.Mặt khác, f (5)≥ 5+k nên f (15) = f (5) f (3)≥ (5+k)(3+k). Vì vậy (5+k)(3+k)≤15 + 2k, từ đó k = 0, f (3) = 3. Bằng quy nạp chứng minh f (2n + 1) = 2n + 1,∀n ∈ Nsuy ra 2n +1 = f (2n +1) < ... < f (2n + k) < ... < f (2n +1) = 2n+1 +1, với mọi n ∈ Nvà k = 1,2, ...,2n. Từ 2n +1 đến 2n+1 +1 có đúng 2n số tự nhiên nên f (2n +k) = 2n +k.Kết luận f (n) = n,∀n ∈ N.

Bài toán 3.2.4. (IMO 1977) Xác định tất cả các hàm f : N→ N thỏa mãn

f (n+1) > f ( f (n)), ∀n ∈ N.

LỜI GIẢI. Bằng phép lặp khá đặc biệt tương tự qui nạp, ta chỉ ra f (k)≥ k, ∀k ∈ N.

Ta có f (n+1) > f ( f (n))≥ 0⇒ f (n+1)≥ 1, ∀n∈N. Nên khi đặt g(n) = f (n+1)−1thì g : N→ N. Và ta có

g(n)+1 = f (n+1) > f ( f (n)) = f (g(n−1)+1) = g(g(n−1))+1, ∀n ∈ N∗.

Như vậy ta lại có g(n + 1) > g(g(n)), ∀n ∈ N. Do đó g(n + 1) ≥ 1 suy ra f (n + 2) =g(n+1)+1≥ 2. Lập lại quá trình trên, tức lại xây dựng hàm h(n) = g(n+1)−1, ... tasuy ra f (n+ k)≥ k, ∀k ∈ N. Lấy n = 0 thì ta có f (k)≥ k, ∀k ∈ N. Theo điều kiện banđầu ta suy ra f (k +1) > f ( f (k))≥ f (k). Điều này dẫn đến f là hàm tăng trên N. Mặtkhác f (k+1) > f ( f (n)) nên k+1 > f (k). Như vậy k+1 > f (k)≥ k, ∀k ∈N. Dẫn đếnnghiệm của bài toán là f (k) = k, ∀k ∈ N.

Bài toán 3.2.5. (THTT - T6/ Số 302) Xét hàm f : N∗→ N∗ thỏa mãn

(i). f (n+1) = f (n)−1 hoặc 4 f (n)−1, ∀n ∈ N∗.(ii). Với mọi m ∈ N∗ thì tồn tại duy nhất n ∈ N∗ mà f (n) = m.

Tính giá trị f (2002)

44

Page 50: Đ—I H¯C QU¨C GIA HÀ N¸I TRƯ˝NG Đ—I H¯C KHOA H¯C TÜ NHIÊN KHOA … · 2016-05-09 · Đ—I H¯C QU¨C GIA HÀ N¸I TRƯ˝NG Đ—I H¯C KHOA H¯C TÜ NHIÊN KHOA TOÁN

LỜI GIẢI. Xét dãy f (1), f (2), f (3), ... Đây là dãy các số nguyên dương đôi một phânbiệt. Ta có các nhận xét sau:

1) Nếu f (n+1) > f (n) thì với mọi k ≥ n+1 ta đều có f (k) > f (n).2) Vì mỗi số nguyên dương xuất hiện đúng một lần trong dãy nên f (n+1) = f (n)−1

nếu f (n)−1 /∈ { f (1), f (2), ..., f (n)}. Trong trường hợp còn lại thì f (n+1) = 4 f (n)−1.

3) Nếu f (1) 6= 1 thì tồn tại a 6= 1 ∈ N∗ để f (a) = 1. Khi đó f (a + 1) = 3, từ đây vànhận xét 1 ta sẽ suy ra a = 2. Do đó f (1) = f (a−1) = 2. Như vậy f (1) ∈ {1, 2}.

Bây giờ, ta đi xét hai trường hợp sau:*) Nếu f (1) = 1. Ta chứng minh f (2n + k) = 2n+1− (k + 1) với 0 ≤ k ≤ 2n− 1.

Ta chứng minh qui nạp theo s = 2+k. Dễ thấy s = 1,2,3 đúng. Giả sử đúng tới s.

Nếu s = 2n + k với 0 ≤ k ≤ 2n − 2 thì f (s) = 2n+1 − (k + 1). Theo nhận xét 2 thìf (s + 1) = f (s)− 1 = 2n+1− (k + 2). Còn nếu s = 2n + k, k = 2n− 1 thì f (s) = 2n.

Ta có f (2n−1) = 2n−1 = f (s)−1 đã xuất hiện trong dãy { f (1), f (2), ..., f (s)} nên theonhận xét 2 thì f (s+1) = 4 f (s)−1 = 2n+2−1. Như vậy ta đã hoàn tất chứng minh khẳngđịnh trên.

*) Nếu f (1) = 2 thì chứng minh tương tụ ta có f (4n + k) = 3.4n − (k + 1) nếuo≤ k ≤ 2.4n−1 và f (4n + k) = 6.4n− (k +1) nếu 2.4k ≤ k ≤ 3.4n− (k +1).Từ đó ta có f (2002) = 1069 hoặc f (2002) = 2093.

MỘT SỐ BÀI TẬP VẬN DỤNG:

Bài tập 3.2.1. Cho hàm số f (x) xác định trên N thỏa mãn f (0) = 0, f (1) = 1 và

f (n+1) = 3 f (n)−2 f (n−1), ∀n ∈ N∗.

Chứng minh rằng f (n) = 2n−1.

Bài tập 3.2.2. Tìm f : R+→ R+ liên tục thỏa mãn các tính chất sau:

(i) f (2x) = 2 f (x),(ii) f ( f (x)) = x f (x), ∀x ∈ R+,

(iii) f (n) ∈ N∗ với n ∈ N∗

Gợi ý. Chỉ ra f đơn ánh, kết hợp f liên tục, f (2) = 2 f (1) = 2 ta suy ra f tăng thực sự.Quy nạp f (2n) = 2n, ∀n ∈ N∗.Từ đó, 2n = f (2n) < f (2n+1) < ... < f (2n+2n) = 2n+1, do từ 2n đến 2n+1 có đúng 2n+1số tự nhiên, suy ra f (2n +k) = 2n +k, ∀k = 0,1, ...,2n. Dẫn đến f (n) = n, ∀n∈N∗. Quy

nạp f (n2k ) =

f (n)2k =

n2k , suy ra f (x) = x trên Q+. Do f liên tục nên f (x) = x trên R+.

45

Page 51: Đ—I H¯C QU¨C GIA HÀ N¸I TRƯ˝NG Đ—I H¯C KHOA H¯C TÜ NHIÊN KHOA … · 2016-05-09 · Đ—I H¯C QU¨C GIA HÀ N¸I TRƯ˝NG Đ—I H¯C KHOA H¯C TÜ NHIÊN KHOA TOÁN

Bài tập 3.2.3. Xác định hàm f : [0,1]→ [0,1] thỏa mãn

f (2x− f (x)) = x, ∀x ∈ [0,1].

Gợi ý. Theo cách xác định của hàm f thì ta có 0 ≤ 2x− f (x) ≤ 1, ∀x ∈ [0,1]. Đặtg(x) = 2x− f (x) thì g : [0,1]→ [0,1]. Ta có f (g(x)) = x và g(g(x)) = 2g(x)− f (g(x)) =2g(x)−x. Từ đó bằng quy nạp ta chỉ ra gn(x) = n(g(x)−x)+x, ∀x ∈ [0,1]. Từ đó ta suyra g(x) = x, ∀x ∈ [0,1]. Vì nếu tồn tại 0 ≤ x0 ≤ 1 mà g(x0) 6= x0 thì gn(x0)→ +∞ khin→+∞, mâu thuẫn. Thế nên f (x) = x, ∀x ∈ [0,1].

Bài tập 3.2.4. Tìm tất cả các hàm số f liên tục trên R thỏa mãn f (1) = 1 và

f (√

x2 + y2) = f (x)+ f (y), ∀x,y ∈ R.

Gợi ý. Chỉ ra f (0) = 0, f (x) là hàm chẵn. Chứng minh bằng qui nạp

f (√

x21 + x2

2 + ...+ x2n) = f (x1)+ f (x2)+ ...+ f (xn), ∀x1,x2, ...,xn ∈ R.

Từ đó chỉ ra f (

√kn) =

kn, ∀k,n ∈ N∗. Hay f (r) = r2, ∀r ∈ Q+. Kết hợp f (x) chẵn và

là hàm liên tục ta suy ra f (x) = x2, ∀x ∈ R.

Bài tập 3.2.5. (China TST 2007) Tìm tất cả các hàm f : Q+→Q+ thỏa mãn

f (x)+ f (y)+2xy f (xy) =f (xy)

f (x)+ f (y), ∀x,y ∈Q+.

Gợi ý. Chứng minh bằng qui nạp các khẳng định sau:

f (n) =1n2 , ∀n ∈ N∗ và f (nx) =

f (x)n2 , ∀n ∈ N∗, x ∈Q+.

Từ đó chỉ ra f (x) =1x2 , ∀x ∈Q+.

3.3 Phương pháp sử dụng tính đơn điệu của hàm số.

Mệnh đề 3.1. Chứng minh rằng:

(i) Nếu f (x),g(x) cùng đơn điệu tăng, hoặc cùng đơn điệu giảm trên R thì f (g(x)), g( f (x))cũng tăng trên R. Từ đó suy ra: Nếu f là hàm tăng trên R thì f n(x) cũng là hàm tăng

trên R, với mọi n ∈ N∗.(ii) Nếu f (x) tăng trên R, g(x) giảm trên R thì f (g(x)),g( f (x)) đơn điệu giảm trên R.

Từ đó suy ra: Nếu f (x) đơn điệu giảm trên R thì f 2n(x) đơn điệu tăng, còn f 2n+1(x) đơn

điệu giảm trên R.

46

Page 52: Đ—I H¯C QU¨C GIA HÀ N¸I TRƯ˝NG Đ—I H¯C KHOA H¯C TÜ NHIÊN KHOA … · 2016-05-09 · Đ—I H¯C QU¨C GIA HÀ N¸I TRƯ˝NG Đ—I H¯C KHOA H¯C TÜ NHIÊN KHOA TOÁN

LỜI GIẢI. (i). Ta chứng minh cho trường hợp khó thấy hơn. Trường hợp còn lại đượcchứng minh tương tự.Giả sử f (x),g(x) là các hàm nghịch biến. Khi đó, với mọi x > y ta có

x > y ⇐⇒ g(x) < g(y) ⇐⇒ f (g(x)) > f (g(y)),

hay f (g(x)) là hàm tăng trên R. Tương tự, g( f (x)) cũng tăng trên R.

- Bây giờ giả sử f (x) là hàm tăng. Áp dụng kết quả trên với g(x)≡ f (x), thì f 2(x) là hàmtăng. Lấy g(x) ≡ f 2(x) thì f 3(x) = f ( f 2(x)) cũng là hàm tăng. Bằng quy nạp dễ dàngchỉ ra f n(x) đơn điệu tăng trên R.

(ii). Giả sử f (x) tăng, còn g(x) giảm. Với mọi x > y ta có

x > y ⇐⇒ g(x) < g(y) ⇐⇒ f (g(x)) < f (g(y)),

do đó f (g(x)) đơn điệu giảm trên R. Tương tự, g( f (x)) cũng đơn điệu giảm trên R.- Nếu f (x) đơn điệu giảm, thì sử dụng kết quả (i) với g(x) ≡ f (x) ta suy ra f 2(x) đơnđiệu tăng, do đó f 2n(x) đơn điệu tăng với mọi n ∈ N, cũng theo hệ quả của (i). Bây giờ,áp dụng (ii) với g(x) = f 2n(x) là hàm tăng, ta suy ra f 2n+1(x) = g( f (x)) đơn điệu giảmvới mọi n ∈ N.

Nhận xét: Ở trên, nếu ta thay các giả thiết "đơn điệu tăng" bởi tính "không giảm", "đơnđiệu giảm" bởi tính "không tăng" thì kết quả không thay đổi. Từ đây ta thấy rằng: "với

mọi n ∈ N∗ thì f 2n+1(x) cùng tính đơn điệu với f (x), nhưng f 2n(x) cùng tính đơn điệu

với f (x) khi và chỉ khi f (x) đơn điệu tăng". Đây là một kết quả cơ bản giúp ta phản xạtốt hơn khi làm việc với hàm đơn điệu.

Bài toán 3.3.1. Cho trước k ∈ N∗. Tìm hàm f : R→ R trong hai trường hợp sau:

(i). f không giảm thỏa mãn f k(x) = x,∀x ∈ R, với k ≥ 2.

(ii). f đơn điệu thỏa mãn f 2k+1(x) = x,∀x ∈ R.

LỜI GIẢI. (i). Nhận thấy rằng f (x) = x, ∀x ∈ R là nghiệm của bài toán.Ta đi chứng minh nghiệm này là duy nhất. Thật vậy, giả sử tồn tại a∈R sao cho f (a) 6= a,ta xét 2 trường hợp sau:- Nếu f (a) > a. Do f không giảm nên f ( f (a)) ≥ f (a) > a suy ra f 3(a) = f ( f 2(a)) ≥f (a) > a. Sử dụng quy nạp, dễ dàng thấy f k(a)≥ a, mâu thuẫn với f k(a) = a.- Nếu f (a) < a thì f ( f (a)) ≤ f (a) < a. Bằng quy nạp ta cũng suy ra f k(a) < a, mâuthuẫn với f k(a) = a.

Vậy giả sử trên là sai. Hay f (x) = x, ∀x ∈ R.(ii). Giả sử f đơn điệu giảm thì theo kết quả của mệnh đề trên, ta suy ra f 2k+1 cũng đơnđiệu giảm, mâu thuẫn vì f 2k+1(x) = x là hàm tăng. Vậy giả sử trên là sai, có nghĩa f làhàm không giảm. Đến đây, áp dụng (i) ta suy ra nghiệm của bài toán là f (x) = x,∀x∈R.

47

Page 53: Đ—I H¯C QU¨C GIA HÀ N¸I TRƯ˝NG Đ—I H¯C KHOA H¯C TÜ NHIÊN KHOA … · 2016-05-09 · Đ—I H¯C QU¨C GIA HÀ N¸I TRƯ˝NG Đ—I H¯C KHOA H¯C TÜ NHIÊN KHOA TOÁN

Nhận xét: Bài toán này là một kết quả rất có ích với nhiều bài PTH. Chúng ta sẽ thấyđược điều đó ở các bài toán tiếp dưới đây.

Mệnh đề 3.2. Cho g(x) là hàm liên tục trên R. Chứng minh rằng nếu f : R→R là hàm

đơn điệu thỏa mãn f (x) = g(x), ∀x ∈Q thì f (x) = g(x), ∀x ∈ R.

Chứng minh.Ta chứng minh cho trường hợp f là hàm không giảm. Trường hợp f là hàm không tăngđược suy ra từ trường hợp f không giảm bằng việc xét hàm − f (x).Với mỗi x ∈ R, lấy hai dãy hữu tỉ sn giảm và rn tăng sao cho

limn→+∞

sn = limn→+∞

rn = x⇒ limn→+∞

g(sn) = limn→+∞

g(rn) = g(x). (1)

Do f không giảm nên g(sn) = f (sn) > f (x) > f (rn) = g(rn), chuyển qua giới hạn khin→+∞, kết hợp (1) ta có g(x)≥ f (x)≥ g(x)⇒ f (x) = g(x).Vì x ∈ R lấy tùy ý nên f (x) = g(x),∀x ∈ R (ĐPCM) !Nhận xét: Kết quả mệnh đề không đổi nếu đổi giả thiết f đơn điệu bởi f liên tục, haythay R bởi một khoảng hay đoạn bất kì. Nếu lấy g(x) = ax thì f (x) = g(x),∀x ∈Q đồngnghĩa f cộng tính. Khi đó, đây là kết quả có nhiều ứng dụng quan trọng với lớp PTH vừađơn điệu vừa cộng tính, mà chúng ta đã xét đến trong phần PTH cơ bản. Dưới đây là mộtvài ví dụ vận dụng mệnh đề này.

Bài toán 3.3.2. (IMO 2002) Tìm tất cả các hàm f : R→ R thỏa mãn

( f (x)+ f (z))( f (y)+ f (t)) = f (xy− zt)+ f (xt + yz), ∀x,y,z, t ∈ R. (*)

LỜI GIẢI. Nhận thấy (*) có các nghiệm: f (x)≡ 0, f (x)≡ 12, f (x) = x2. Ta sẽ chỉ ra đây

là tất cả các nghiệm của bài toán.Giả sử f thỏa mãn (*). Lấy x = y = z = 0 ta có 2 f (0) = 2 f (0)( f (0)+ f (t)). Đặc biệt ta có2 f (0) = 4 f (0), vì vậy f (0) = 0 hoặc f (0) = 1/2. Nếu f (0) = 1/2 thì ta có f (0)+ f (t) =1 nên f (t)≡ 1/2 trên R.Xét trường hợp f (0) = 0. Khi đó, trong (*) lấy z = t = 0 ta có

f (xy) = f (x) f (y), ∀x,y ∈ R.

Như vậy f là hàm nhân tính. Đặc biệt, f (1) = [ f (1)]2 nên f (1) = 0 hoặc f (1) = 1. Nếuf (1) = 0 thì f (x) = f (x) f (1) = 0, ∀x ∈ R.Giả sử f (1) = 1. Trong (*) lấy x = 0 và y = t = 1 ta có f (−z)+ f (z) = 2 f (z). Vì vậyf (z) = f (−z), ∀z ∈R, mặt khác f (x2) = [ f (x)]2 nên f là hàm chẵn và f (y)≥ 0,∀y ∈R.Trong trường hợp cuối này ta chỉ ra rằng f (x) = x2, ∀x ∈ R. Thật vậy, lấy y = z = t = 1trong (*) thì

f (x−1)+ f (x+1) = 2( f (x)+1), ∀x ∈ R. (1)

48

Page 54: Đ—I H¯C QU¨C GIA HÀ N¸I TRƯ˝NG Đ—I H¯C KHOA H¯C TÜ NHIÊN KHOA … · 2016-05-09 · Đ—I H¯C QU¨C GIA HÀ N¸I TRƯ˝NG Đ—I H¯C KHOA H¯C TÜ NHIÊN KHOA TOÁN

Đến đây, ta thấy rằng (1) cũng đúng với x ∈ N. Với f (0) = 0, f (1) = 1 bằng quy nạp tacó thể dễ dàng chứng minh được f (n) = n2, ∀n ∈ N (ta cũng có thể dùng phương phápquy về dãy số để chỉ ra điều này). Nhưng do f là hàm chẵn nên f (x) = x2, ∀x ∈Z. Thêmvào đó, f là hàm nhân tính nên dễ dàng thấy f (x) = x2, ∀x ∈Q.

Ta thấy f không giảm trên R+. Thật vậy, ở (*) lấy t = x,z = y ta có

f (x2 + y2) = ( f (x)+ f (y))2.

Điều này chỉ ra rằng f (x2 + y2) ≥ [ f (x)]2 + [ f (y)]2, do f (x) ≥ 0,∀x ∈ R. Vì vậy nếuu≥ v≥ 0 thì f (u)≥ f (v), hay f không giảm trên R+.Đến đây sử dụng mệnh đề trên ta suy ra f (x) = x2,∀x ∈ R+. Hoặc vì trên R+ hàm x2 cóhàm ngược

√x, nên ta có thể làm đơn giản hơn như sau:

Giả sử tồn tại x > 0 mà 0≤ f (x) 6= x2. Ta xét 2 trường hợp:- Nếu 0≤ f (x) < x2, lấy a hữu tỉ thỏa mãn x > a >

√f (x), thì f (a) = a2 > f (x). Nhưng

do f không giảm trên R+ nên f (a)≤ f (x), mâu thuẫn.- Nếu f (x) > x2, tương tự ta cũng suy ra mâu thuẫn.Vì vậy f (x) = x2, ∀x ∈ R+, kết hợp f là hàm chẵn nên f (x) = x2, ∀x ∈ R. Đến đây tađã giải xong bài toán với kết quả như trên.Nhận xét: Bài toán xuất phát từ đồng nhất thức Lagrange:

(x2 + z2)(y2 + z2) = (xy− zt)2 +(xt + yz)2.

Áp dụng mệnh đề trên: khi biết hàm f trên tập Q nếu biết thêm một số điều kiện nữa,chẳng hạn: f liên tục, đơn điệu,... thì ta có thể xác định được f trên R. Ở bài toán trên,việc thấy f tăng trên R+ là khá quan trọng. Trong bài toán trên nếu xuất phát từ (*) khicho t = 1 thì sẽ được bài toán mới kín hơn, nhưng có thể được giải một cách tương tự,chỉ khác khi chứng minh f không giảm trên R+. Cụ thể như sau.

Bài toán 3.3.3. Tìm tất cả các hàm f : R→ R thỏa mãn

( f (x)+ f (z))( f (y)+1) = f (xy− z)+ f (x+ yz), ∀x,y,z ∈ R. (**)

LỜI GIẢI. Bài toán này sẽ có hai nghiệm f (x)≡ 0 hoặc f (x) = x2, ∀x ∈ R.Trong (**) cho x = z = y = 0 ta có 2 f (0)( f (0) + 1) = 2 f (0) suy ra f (0) = 0. Khichỉ lấy z = 0 ta có f (x)( f (y)+ 1) = f (xy)+ f (x) suy ra f (xy) = f (x) f (y) hay f . Lấyx = y = 1,z = 0 trong (**) ta có f (1)( f (1)+ 1) = 2 f (1) nên f (1) = 0 hoặc f (1) = 1.Nếu f (1) = 0 thì f (x) = f (x) f (1) = 0,∀x ∈ R, đây là một nghiệm của bài toán.Ta xét trường hợp f (1) = 1. Trong (**) lấy x = y = 0 suy ra f (z) = f (−z) hay f là hàmchẵn. Lấy y = z = 1 ta có 2 f (x) = f (x− 1)+ f (x + 1),∀x ∈ R. Tương tự như bài toántrên ta cũng suy ra f (x) = x2 với mọi x hữu tỉ.

49

Page 55: Đ—I H¯C QU¨C GIA HÀ N¸I TRƯ˝NG Đ—I H¯C KHOA H¯C TÜ NHIÊN KHOA … · 2016-05-09 · Đ—I H¯C QU¨C GIA HÀ N¸I TRƯ˝NG Đ—I H¯C KHOA H¯C TÜ NHIÊN KHOA TOÁN

Tiếp theo, ta cũng đi chứng minh f không giảm trên R+. Lấy x = 1,y = z trong (**) tacó [ f (y)+1]2 = f (y2 +1), ∀y ∈ R.

Ta có do f nhân tính nên f (x) ≥ 0,∀x ≥ 0, suy ra f (y2 + 1) = [ f (y)]2 + 2 f (y) + 1 ≥1,∀y ≥ 0. Vậy f (x) ≥ 1,∀x ≥ 1. Khi đó với mọi x ≥ y > 0, ta có f (x) = f (y

xy) =

f (y) f (xy) ≥ f (y). Vì vậy f không giảm trên R+. Một cách hoàn toàn tương tự như

bài toán trên ta suy ra f (x) = x2,∀x ∈ R. Bài toán kết thúc ở đây.

Mệnh đề 3.3. Chứng minh rằng nếu f : R→ R vừa là hàm đơn ánh, vừa là hàm liên

tục, thì f đơn điệu.

LỜI GIẢI. Vì f là đơn ánh, ta chứng minh nếu tồn tại x < y sao cho f (x) < f (y) thì f

đồng biến (tương tự, trường hợp nếu với mọi x < y mà f (x) > f (y) thì f nghịch biến).Giả sử f không đồng biến, tức là sẽ có ba trường hợp sau có thể xảy ra, tồn tại z sao cho

(1) z < x < y và f (z) > f (x), f (x) < f (y);

(2) x < y < z và f (z) < f (y), f (x) < f (y);

(3) x < z < y và [ f (z)− f (x)] [ f (z)− f (y)] > 0.

Dễ thấy, ta chỉ cần chứng minh (1) sai. Chọn M sao cho f (x) < M < min{ f (y), f (z)} .Theo tính chất của hàm liên tục, tồn tại a sao cho z < a < x và f (a) = M, đồng thời tồntại b sao cho x < b < y và f (b) = M. Suy ra f (a) = f (b), suy ra a = b do f đơn ánh,nhưng điều này không thế xảy ra vì a < x < b. Vậy ta có điều giả sử là sai, tóm lại f làhàm đơn điệu.Nhận xét: Kết quả của mệnh đề không đổi nếu ta thay R bởi các khoảng(a,b), (a,b], [a,b), [a,b] với a < b tùy ý.

Bài toán 3.3.4. Cho trước n ∈ N∗. Hàm f : [0,1]→ [0,1] liên tục thỏa mãn

f (0) = 0, f (1) = 1, f n(x) = x, ∀x ∈ [0,1].Chứng minh rằng f (x) = x trên đoạn [0,1].

LỜI GIẢI. Nếu f (x) = f (y) thì f n(x) = f n(y) suy ra x = y. Do đó, f đơn ánh. Lại cóf liên tục trên [0,1]. Áp dụng kết quả mệnh đề trên ta suy ra f đơn điệu trên [0,1], màf (0) = 0, f (1) = 1 nên f tăng trên [0,1]. Như vậy ta có f là hàm tăng, f n(x) = x, ∀x ∈[0,1]. Đây là bài toán quen thuộc và kết quả là f (x) = x, ∀x ∈ [0,1] (ĐPCM) .

Bài toán 3.3.5. Tìm hàm f : R→ R liên tục và thỏa mãn

f (x f (y)) = y f (x), ∀x,y ∈ R. (1)

50

Page 56: Đ—I H¯C QU¨C GIA HÀ N¸I TRƯ˝NG Đ—I H¯C KHOA H¯C TÜ NHIÊN KHOA … · 2016-05-09 · Đ—I H¯C QU¨C GIA HÀ N¸I TRƯ˝NG Đ—I H¯C KHOA H¯C TÜ NHIÊN KHOA TOÁN

LỜI GIẢI. Ta thấy rằng f (x)≡ 0, ∀x ∈ R là một nghiệm của bài toán. Ta đi tìm nghiệmkhác của bài toán. Giả sử tồn tại nghiệm f (x) như thế. Khi đó, tồn tại a sao cho f (a) 6= 0.Trong (1) lấy x = a ta có f (a f (y)) = y f (a), ∀y ∈ R.Hiển nhiên, ta thấy rằng a 6= 0. Từ đó dễ dàng suy ra f là đơn ánh.Trong (1) lấy x = y = 0 thì f (0) = 0. Lấy y = 1 ta có f (x f (1)) = f (x), ∀x ∈ R. Do f

đơn ánh nên x f (1) = x, ∀x ∈ R suy ra f (1) = 1.Bây giờ, do f liên tục và f đơn ánh nên f đơn điệu. Nhưng f (0) = 0 và f (1) = 1 nên f

đơn điệu tăng. Lấy x = 1 trong (1) ta có f ( f (y)) = y, ∀y ∈ R. Như vậy, f tăng thỏa mãnf ( f (x)) = x, ∀x ∈ R, dẫn đến f (x) = x.

Kết luận: f (x)≡ 0 hoặc f (x) = x, ∀x ∈ R là hàm cần tìm.

Bài toán 3.3.6. Cho h(x) là hàm số đơn ánh trên R và n ∈ N∗. Chứng minh rằng: nếu

tồn tại x < y mà h(x) > h(y) thì không tồn tại hàm f (x) liên tục trên R thỏa mãn

f 2n(x) = h(x), ∀x ∈ R.

LỜI GIẢI. Bài toán này được suy ra khá trực tiếp từ mệnh đề 3.1 và mệnh đề 3.3, với chúý: nếu h(x) đơn ánh và f k(x) = h(x) thì ta suy ra f (x) đơn ánh.

Chúng ta tiếp tục xét một số bài toán, mà ở đó tính đơn điệu là điểm mấu chốt.

Bài toán 3.3.7. (IMO 2005, Shortlist) Xác định tất cả các hàm số f : R+→ R+ thỏa

mãn điều kiện f (x) f (y) = 2 f (x+ y f (x)), ∀x ∈ R+.

LỜI GIẢI. Ta đi chứng minh f không giảm. Với mỗi y > 0 cố định, đặt ϕ(x) = x +y f (x), x > 0. Hàm này là đơn ánh, vì nếu ϕ(x1) = ϕ(x2) thì f (x1) f (y) = f (ϕ(x1)) =f (ϕ(x2)) = f (x2) f (y), nên f (x1) = f (x2), do đó từ định nghĩa của ϕ(x) ta suy x1 = x2.

Bây giờ nếu có x1 > x2 > 0 và f (x1) < f (x2) ta đặt y =x1− x2

f (x2)− f (x1)> 0. Khi đó

x1 + y f (x1) = x2 + y f (x2) hay ϕ(x1) = ϕ(x2), mâu thuẫn do ϕ đơn ánh. Do đó f khônggiảm trên R+. Từ đó, f (x) f (y) = 2 f (x+y f (x))≥ 2 f (x),∀x,y > 0 suy ra f (y≥ 2),∀y >

0. Vì thếf (x+ y f (x)) = f (x) f (y) = f (y+ x f (y))≥ f (2x), ∀x,y > 0 (i)

do y+ x f (y)≥ y+2x > 2x.Trong (i) khi y thay đổi đủ nhỏ thì x + y f (x) nhận toàn bộ giá trị trong khoảng (x,2x],nhưng f không giảm trên R+ nên f là hằng số trên khoảng (x,2x] với mỗi x > 0. Nhưngkhi f là hàm hằng trên khoảng (x,2x] đối với mọi x > 0 thì cũng có nghĩa f là hàm hằngtrên R+. Bây giờ sử dụng phương trình của bài toán ta suy ra f (x)≡ 2 trên R+.Kết luận hàm cần tìm là f (x)≡ 2, ∀x ∈ R+.

51

Page 57: Đ—I H¯C QU¨C GIA HÀ N¸I TRƯ˝NG Đ—I H¯C KHOA H¯C TÜ NHIÊN KHOA … · 2016-05-09 · Đ—I H¯C QU¨C GIA HÀ N¸I TRƯ˝NG Đ—I H¯C KHOA H¯C TÜ NHIÊN KHOA TOÁN

Bài toán 3.3.8. Tìm hàm f : R+→ R+ không tăng và thỏa mãn

f ( f (x)+ y) = x f (1+ xy), ∀x,y > 0.

LỜI GIẢI. Nhận thấy f (x) =1x

là nghiệm của bài toán. Ta đi chứng minh điều này.Trước tiên, ta chứng minh f (1) = 1. Giả sử f (1) 6= 1, Thay x = 1, ta có f ( f (1)+ y) =f (1 + y),∀y > 0 nên f (y + f (1)− 1) = f (y),∀y > 1. Điều này có nghĩa f là hàm tuầnhoàn (với chu kì | f (1)− 1| 6= 0) trên khoảng (1,+∞). Do f không tăng nên f là hàmhằng trên khoảng x > 1, nhưng từ phương trình ban đầu dễ thấy điều này là không thể.Vì thế f (1) = 1.

Ta chứng minh f (x) =1x

với x > 1. Thật vậy, với mọi x > 1 thay y = 1− 1x, ta có

f ( f (x)− 1x

+1) = x f (x). (1)

- Nếu f (x) >1x

thì x f (x) > 1 và f ( f (x)− 1x

+ 1) ≤ f (1) = 1, do f không tăng, mâu

thuẫn.- Nếu f (x) <

1x

< 1 thì x f (x) < 1 f ( f (x)− 1x

+1)≥ f (1) = 1, mâu thuẫn.

Vậy f (x) =1x, ∀x > 1.

Với 0 < x < 1, thay y =1x

ta có f ( f (x)+1x) = x f (2) =

x2, vì f (x)+

1x

> 1 nên f (x)+1x

=2x

, tức là f (x) =1x,∀ 0 < x < 1. Điều này có nghĩa f (x) =

1x,∀x ∈ R+ (ĐPCM) !

Tiếp theo, ta sẽ xét đến tính đơn điệu đối với hàm ngược của một hàm số đơn điệu.Đồng thời xét một số bài toán PTH liên quan.

Ở đây, ta chỉ xét hàm f : R→R là hàm đơn điệu (thực sự), khi đó f cũng là một đơnánh. Đồng thời yêu cầu hàm f này phải nhận giá trị trên toàn R, hay f toàn ánh trên R,nên f sẽ là song ánh và tồn tại hàm ngược của f . Ta kí hiệu hàm này là f−1. Có thể thấyrằng f−1 cũng là song ánh trên R và có hàm ngược chính là f .

Trường hợp miền xác định của f chỉ là miền D ⊆ R, ta chỉ yêu cầu f xác định, đơnđiệu trên D và miền giá trị của f là D.

Ta bắt đầu bằng một kết quả quan trọng về tính đơn điệu của hàm ngược f−1 của f ,thể hiện qua mệnh đề dưới đây.

Mệnh đề 3.4. Với mọi hàm f đơn điệu sao cho f có hàm ngược f−1, thì tính đơn điệu

của f và f−1 là như nhau. Tức là: nếu f tăng thực sự thì f−1 cũng tăng thực sự, còn nếu

f giảm thực sự thì f−1 cũng giảm thực sự.

52

Page 58: Đ—I H¯C QU¨C GIA HÀ N¸I TRƯ˝NG Đ—I H¯C KHOA H¯C TÜ NHIÊN KHOA … · 2016-05-09 · Đ—I H¯C QU¨C GIA HÀ N¸I TRƯ˝NG Đ—I H¯C KHOA H¯C TÜ NHIÊN KHOA TOÁN

Chứng minh:Ta chứng minh cho trường hợp f tăng thực sự (trường hợp sau tương tự).Giả sử f tăng thực sự, nghĩa là f (a) > f (b) ⇐⇒ a > b, ∀a,b ∈ R. Nên ∀a,b ∈ R ta cóf−1(a) > f−1(b) ⇐⇒ f ( f−1(a)) > f ( f−1(b)) ⇐⇒ a > b, do f ( f−1(x)) = x,∀x ∈R.Như vậy f−1(a) > f−1(b) ⇐⇒ a > b,∀a,b ∈ R, điều này có nghĩa f−1 là hàm tăngthực sự (ĐPCM).Nhận xét: Từ đây, do f−1 là hàm ngược của f thì f cũng là hàm ngược của f−1, ta cókết luận f tăng thực sự khi và chỉ khi f−1 tăng thực sự.

Bài toán 3.3.9. (APMO 1989) Tìm hàm f : R→R tăng thực sự thỏa mãn f (x)+g(x) =2x, ∀x ∈ R, trong đó g là hàm ngược của f .

LỜI GIẢI. Ta có thể nhận thấy rằng hàm f (x) = x + c, với c là hằng số tùy ý, là nghiệmcủa bài toán. Ta đi chứng minh bài toán có nghiệm duy nhất xác định như trên.Với mỗi số thực a đặt Sa = {x ∈ R | f (x) = x+a}. Mỗi số thực x0 đều thuộc một tập Sa

nào đó, vì nếu đặt f (x0)−x0 = a thì hiển nhiên x0 ∈ Sa. Do vậy, tồn tại số thực c sao choSc 6= /0.Với mọi a ∈ R ta chứng minh x0 ∈ Sa ⇐⇒ x0 + ka ∈ Sa.Nhờ phép quy nạp, ta chỉ cần chứng minh x0 ∈ Sa ⇐⇒ x0 +a ∈ Sa.Thật vậy, ta có

x0 ∈ Sa ⇐⇒ f (x0) = x0 +a ⇐⇒ g(x0 +a) = x0

⇐⇒ f (x0 +a) = x0 +2a ⇐⇒ x0 +a ∈ Sa.

Bây giờ, sử dụng kết quả trên, ta đi chứng minh rằng nếu Sa 6= /0 thì Sb = /0 với mọi a 6= b.- Xét b < a. Lấy x0 ∈ Sa, với mọi y ∈ R luôn tồn tại k ∈ Z sao cho

x0 + k(a−b)≤ y < x0 +(k +1)(a−b)

⇐⇒ x0 + ka≤ y+ kb < (x0 + ka)+(a−b). (1)

Do f tăng thực sự nên

f (y+ kb)≥ f (x0 + ka) = x0 +(k +1)a. (2)

Nếu y+ kb ∈ Sa thì ta có f (y+ kb) = y+(k +1)b, kết hợp (2) ta có

y+(k +1)b≥ x0 +(k +1)a ⇐⇒ y≥ x0 +(k +1)(a−b),

điều nàu mâu thuẫn với (1).Vậy y+ kb /∈ Sb, do đó y /∈ Sb,∀y ∈ R, nên Sb = /0.

53

Page 59: Đ—I H¯C QU¨C GIA HÀ N¸I TRƯ˝NG Đ—I H¯C KHOA H¯C TÜ NHIÊN KHOA … · 2016-05-09 · Đ—I H¯C QU¨C GIA HÀ N¸I TRƯ˝NG Đ—I H¯C KHOA H¯C TÜ NHIÊN KHOA TOÁN

- Xét b > a. Nếu Sb 6= /0 thì theo trên Sa = /0, mâu thuẫn.Như vậy, tồn tại duy nhất c ∈ R mà Sc 6= /0. Nhưng mỗi số thực đều nằm trong một tậpSa nào đó, vì vậy mọi số thực đều nằm trong tập Sc trên, tức là f (x) = x+ c, ∀x ∈ R.Vậy hàm cần tìm là f (x) = x+ c, với c là hằng số thực bất kì.Nhận xét: Bài toán có thể phát biểu dưới dạng khác: Tìm hàm tăng thực sự f : R→ Rthỏa mãn f (2x− f (x)) = x,∀x∈R. Dễ thấy là 2 cách phát biểu này là tương đương. Trongbài toán APMO 1989 thực chất ta chỉ cần điều kiện f đơn điệu, vì từ f (x)+ f−1(x) = 2x

ta sẽ suy ra f không thể nghịch biến. Để tìm hiểu tại sao, ta xét bài toán dưới đây.

Bài toán 3.3.10. (KHTN 2009) Tìm hàm f : R→ R liên tục thỏa mãn

(i). f đơn ánh,

(ii). f (2x− f (x)) = x,

(iii). Tồn tại x0 sao cho f (x0) = x0.

LỜI GIẢI. Từ (ii) ta có f toàn ánh hơn nữa f đơn ánh nên f là song ánh. Do đó hàm f cóhàm ngược g. Từ (ii) ta suy ra f (x)+g(x) = 2x, ∀x ∈ R.Ta sẽ về bài toán APMO 1989 bằng cách chứng minh f tăng thực sự.Thật vậy, do f liên tục và f đơn ánh nên f đơn điệu, tức f tăng thực sự hoặc f giảm thựcsự. Giả sử f nghịch biến, khi đó hàm ngược g của hàm f cũng nghịch biến.- Nếu tồn tại a ∈ R mà f (a) < a thì g(a) < g( f (a)) = a. Khi đó f (a)+g(a) < 2a, mâuthuẫn với (ii).- Nếu tồn tại a mà f (a) > a thì g(a) > g( f (a)) = a, nên f (a)+ g(a) > 2a, mâu thuẫnvới (ii).Do đó với mọi số thực a thì f (a) = a, điều này mâu thuẫn với giả sử f nghịch biến. Nhưvậy, giả sử f nghịch biến là sai.Vậy f tăng thực sự. Đến đây kết hợp (ii) hay f (x)+g(x) = 2x,∀x ∈ R, sử dụng kết quảbài toán APMO 1989, ta suy ra f (x) = x + c với c là hằng số thực nào đó. Nhưng theo(iii) tồn tại x0 sao cho f (x0) = x0 nên ta có c = 0, hay f (x) = x,∀x ∈ R. Đây cũng chínhlà kết quả của bài toán.

Bài toán 3.3.11. (Crux Mathematicorum 2003) Xác định hàm số f : R→ R thỏa mãn

f (x3 + x)≤ x≤ [ f (x)]3 + f (x), ∀x ∈ R.

LỜI GIẢI. Xét hàm g(x) = x3 + x, ta thấy rằng hàm này tăng thực sự trên R và nhận giátrị trên toàn R. Vì vậy, g là song ánh trên R, với hàm ngược là g−1 cũng là song ánh vàtăng thực sự trên R.Giả sử hàm f thỏa mãn bài toán. Từ điều kiện của bài toán ta có

f (g(x))≤ x≤ g( f (x)), ∀x ∈ R.

54

Page 60: Đ—I H¯C QU¨C GIA HÀ N¸I TRƯ˝NG Đ—I H¯C KHOA H¯C TÜ NHIÊN KHOA … · 2016-05-09 · Đ—I H¯C QU¨C GIA HÀ N¸I TRƯ˝NG Đ—I H¯C KHOA H¯C TÜ NHIÊN KHOA TOÁN

Từ vế đầu tiên, thay x bởi g−1(x) ta được f (x)≤ g−1(x),∀x ∈ R. Còn với vế thứ 2, ta tácđộng g−1 lên cả hai vế, chú ý g−1 là hàm tăng ta sẽ có g−1(x) ≤ f (x),∀x ∈ R. Do đóf (x) = g−1(x),∀x ∈ R. Đây là nghiệm của bài toán.Nhận xét: Ta có thể thay g(x) = x3 + x bởi hàm g(x) đơn điệu tăng mà miền giá trị làtoàn R bất kì, ta sẽ được bài toán tương tự. Nếu xuất phát từ g(x) là toàn ánh giảm thì taphải đổi điều kiện bài toán thành f (g(x))≥ x≥ g( f (x)), ∀x ∈R. Tương tự, nghiệm củabài toán mới này vẫn là f (x) = g−1(x),∀x ∈ R.

Để kết thúc mục này, ta đi xét một bài toán khá đặc sắc về hàm f có tính chấtgần giống với hàm đơn điệu: f (x +a)≥ f (x)+ c (∗), với mọi x thực nằm trong khoảngD ⊆ R nào đó; a,c là các hằng số thực (ở đây (*) có thể đúng với nhiều giá trị của a).Ta thấy rằng trong trường hợp c = 0, nếu (*) đúng với mọi a≥ 0 thì f đồng biến trên D,còn nếu (*) đúng với mọi a≤ 0 thì f nghịch biến trên D.

Bài toán 3.3.12. (Bulgaria 1998) Chứng minh rằng không tồn tại hàm f : R+ → R+

thỏa mãn [ f (x)]2 ≥ f (x+ y)( f (x)+ y), ∀x,y > 0.

LỜI GIẢI. Ý tưởng của bài toán là chứng minh f (y) < 0 với giá trị thực y > 0 nào đó,điều này dẫn đến mâu thuẫn. Từ đó ta có ĐPCM.Để có được điều này ta đi chứng minh với mọi x ∈ R+ ta có f (x)− f (x + 1) ≥ c > 0, c

là hằng số nào đó. Khi đó với x cố định ta có f (x)− f (x+m)≥ mc, với m đủ lớn suy raf (x+m) < 0. Công việc của chúng ta là chỉ ra tồn tại c như thế.Giả sử tồn tại hàm f thỏa mãn bài toán. Từ giả thiết ta suy ra

f (x)− f (x+ y)≥ f (x)yf (x)+ y

, ∀x,y > 0.

Với mỗi x do f (x +1) > 0 nên tồn tại n đủ lớn sao cho f (x +1)n≥ 1. Chú ý rằng với k

mà 0≤ k ≤ n−1 ta có

f (x+kn)− f (x+

k +1n

)≥f (x+ k

n)1n

f (x+ kn)+ 1

n

≥ 12n

,

Cộng tất cả n bất đẳng thức cùng chiều này, ta có f (x)− f (x+1)≥ 12.

Điều này có nghĩa tồn tại c = 1/2. Theo lập luận ở trên thì ta có ĐPCM .

MỘT SỐ BÀI TẬP VẬN DỤNG:

55

Page 61: Đ—I H¯C QU¨C GIA HÀ N¸I TRƯ˝NG Đ—I H¯C KHOA H¯C TÜ NHIÊN KHOA … · 2016-05-09 · Đ—I H¯C QU¨C GIA HÀ N¸I TRƯ˝NG Đ—I H¯C KHOA H¯C TÜ NHIÊN KHOA TOÁN

Bài tập 3.3.1. Tìm tất cả các hàm đơn điệu f : R→ R thỏa mãn

f ( f (x)+ y) = f (x+ y)+1, ∀x,y ∈ R.

Gợi ý. Từ điều kiện ta có f ( f (x)+ y) = f ( f (y)+ x),∀x,y ∈ R.Từ đó f (x) = x+ c với c là hằng số. Thử lại ta suy ra f (x) = x+1,∀x ∈ R.

Bài tập 3.3.2. Tìm hàm f : R→ R sao cho với mọi số thực x ta có

f (20x5 +2011x3 +11x)≤ x≤ 20[ f (x)]5 +2011[ f (x)]3 +11[ f (x)].

Bài tập 3.3.3. (Olympic SV 2000) Cho đa thức f (x) thỏa mãn f (x)− x và f (x)− x3 là

những hàm tăng trên R. Chứng minh rằng f (x)−√

32

x2 cũng là hàm tăng.

Gợi ý. Dùng đạo hàm và bất đẳng thức AM-GM.

Bài tập 3.3.4. Tìm tất cả các hàm tăng f : R+→ R thỏa mãn

f (x+1) = f (x)+2−x, ∀x > 0.

Gợi ý. Giả sử tồn tại f . Đặt f (x)− 21−x = g(x) thì do f là hàm tăng trên R+ nên g

cũng là hàm tăng trên R+. Ngoài ra g(x+1) = g(x), hay g là hàm tuần hoàn, mâu thuẫnvới khẳng định hàm g tăng. Vậy không có f thỏa mãn bài toán.

Bài tập 3.3.5. Tìm hàm f : [1,+∞)→ [1,+∞) thỏa mãn

f (x f (y)) = y f (x), ∀x,y≥ 1. (1)

Gợi ý. Chỉ ra f đơn ánh, f (1) = 1 và f ( f (y)) = y. Khi đó, trong (1) thay y bởi f (y)ta có f (xy) = f (x f ( f (y))) = f (y) f (x) ≥ f (x),∀x,y ≥ 1. Suy ra f đồng biến, kết hợpf ( f (y)) = y ta có f (x) = x, ∀x≥ 1.

Bài tập 3.3.6. Tìm hàm số đơn điệu f : [0,+∞)→ R thỏa mãn

[ f (x)+ f (y)]2 = f (x2− y2)+2 f (xy), ∀x,y≥ 0.

Gợi ý. Cho (x,y) = (0,0) suy ra f (0) = 0 hoặc f (0) = 1/2.

*) Nếu f (0) = 1/2. Thay (x,y) = (1,0) suy ra f (1) = −1/2 hoặc f (1) = 1/2. Nếuf (1) =−1/2, cho (x,y) = (1,1) suy ra f (2) = 1/2. Khi đó f (0) > f (1) và f (1) < f (2),mâu thuẫn với giả thiết f đơn điệu. Vậy f (1) = 1/2, xét dãy xn+1 = 2x2

n,x0 = 1, thay(x,y) = (xn,xn) ta suy ra f (xn) = 1/2. Do f (x) đơn điệu nên f (x) = 1/2,∀x≥ 0.

*) Xét f (0) = 0. Cho (x,y) = (x,0) suy ra [ f (x)]2 = f (x2). Cho (x,y) = (x,x) thì

56

Page 62: Đ—I H¯C QU¨C GIA HÀ N¸I TRƯ˝NG Đ—I H¯C KHOA H¯C TÜ NHIÊN KHOA … · 2016-05-09 · Đ—I H¯C QU¨C GIA HÀ N¸I TRƯ˝NG Đ—I H¯C KHOA H¯C TÜ NHIÊN KHOA TOÁN

4[ f (x)]2 = f (2x2), suy ra 4 f (x) = f (2x),∀x ≥ 0. Thay (x,y) = (u + v,u− v) ta suyra [ f (u + v) + f (u− v)]2 = 4[ f (u) + f (v)]2. Suy ra f (u + v) + f (u− v) = 2[ f (u) +f (v)],∀u ≥ v ≥ 0. Từ đó chứng minh f (x) = f (1)x2, ∀x ∈ Q+. Sử dụng mệnh đề3.2, ta suy ra: nếu f (1) = 0 thì f (x) = 0,∀x ≥ 0, còn nếu f (1) 6= 0 thì f (1) = 1 vàf (x) = x2,∀x≥ 0. Bài toán xuất phát từ đẳng thức hình bình hành.

Bài tập 3.3.7. Tìm tất cả các hàm đồng biến f : R→ R thỏa mãn

f (x)+ f−1(y) = x+ y, ∀x,y ∈ R.

Gợi ý. Thay y = x, ta có f (x)+ f−1(x) = 2x. Đến đây đưa về bài toán APMO 1989.

Bài tập 3.3.8. (KHTN 2010) Tìm hàm f : R+→ R+ thỏa mãn

f (x3 + y) = [ f (x)]3 +f (xy)f (x)

, ∀x,y > 0.

Gợi ý. Chỉ ra f (x3 + 1) = [ f (x)]3 + 1 và f (y + 1) = [ f (1)]3 +f (y)f (1)

. Từ đây đi tính

f (1) = 1. Do đó f (x+1) = f (x)+1 và f (x3) = [ f (x)]3. Từ đây chỉ ra f (r) = r, ∀r ∈R.

Chứng minh f là hàm tăng trên R+, và do đó f (x) = x, ∀x ∈ R+.

Bài tập 3.3.9. (Iran 1997) Cho f : R→R là hàm đơn điệu giảm, thỏa mãn với mọi x∈Rta có

f (x+ y)+ f ( f (x)+ f (y)) = f ( f (x+ f (y)))+ f (y+ f (x)), ∀x,y ∈ R.

Chứng minh rằng f ( f (x)) = x, ∀x ∈ R.

Gợi ý. Thay y = x ta có

f (2x)+ f (2 f (x)) = f (2 f (x+ f (x))). (1)

Trong (1) thay x bởi f (x) ta có

f (2 f (x))+ f (2 f ( f (x))) = f (2 f ( f (x)+ f ( f (x)))). (2)

Trừ (2) cho (3) ta có

f (2x)− f (2 f ( f (x))) = f (2 f (x+ f (x)))− f (2 f ( f (x)+ f ( f (x)))). (3)

Từ (3), kết hợp giả thiết f giảm ta suy ra trường hợp tồn tại x mà f ( f (x)) > x hoặcf ( f (x)) < x không thể xảy ra. Do đó f ( f (x)) = x, ∀x ∈ R.

57

Page 63: Đ—I H¯C QU¨C GIA HÀ N¸I TRƯ˝NG Đ—I H¯C KHOA H¯C TÜ NHIÊN KHOA … · 2016-05-09 · Đ—I H¯C QU¨C GIA HÀ N¸I TRƯ˝NG Đ—I H¯C KHOA H¯C TÜ NHIÊN KHOA TOÁN

Bài tập 3.3.10. (IMO 2003, Shortlist) Tìm hàm f : R→ R không giảm thỏa mãn

(i). f (0) = 0, f (1) = 1 ;(ii). f (a)+ f (b) = f (a) f (b)+ f (a+b−ab), ∀a < 1 < b.

Gợi ý. Đặt g(x) = f (x + 1)− 1, khi đó g(x) không giảm và g(−1) = −1,g(0) = 0.

Chỉ ra g(x) là hàm nhân tính. Từ đó xét các trường hợp g(1) = 0 hoặc g(1) 6= 0.

Bài tập 3.3.11. (Olympic Châu Mỹ La Tinh 1991) Cho hàm f xác định và không giảm

trên đoạn [0,1] thỏa mãn

(i). f (0) = 0 ;

(ii). f (x3) =

f (x)2

, ∀x ∈ [0,1] ;(iii). f (1− x) = 1− f (x), ∀x ∈ [0,1] ;

Tìm giá trị của f (18

1991).

Bài tập 3.3.12. (Brazil 2003) Cho hàm f : R+→ R tăng thực sự trên R+ thỏa mãn

f (2xy

x+ y) =

f (x)+ f (y)2

, ∀x,y ∈ R+.

Chứng minh rằng tồn tại x > 0 sao cho f (x) < 0.

Gợi ý. Đặt xn = 1/n và f (xn) = yn, ∀n ∈ N∗. Ta có

2xn−1xn+1

xn−1 + xn+1= xn, ∀n ∈ N∗,

từ đó kết hợp với điều kiện ta suy ra

yn =yn−1 + yn+1

2⇒ yn− yn+1 = yn−1− yn, ∀n ∈ N∗.

Đặt d = y1− yy > 0 thì ta có yn+1 = y1−nd. Vậy với n đủ lớn ta có ĐPCM.

3.4 Phương pháp sử dụng tính chất của ánh xạ

Phương pháp chung: Với đơn ánh f , ta sử dụng tính đơn ánh của f một cáchtrực tiếp, nghĩa là " f (x) = f (y) ⇐⇒ x = y", để suy ra những điều kiện mới của hàm f ,thường là phương trình mới, giá trị của f tại một điểm nào đó ,...

Còn đối với toàn ánh f , ta có thể vận dụng tính toàn ánh vào việc chọn những giá trịthích hợp của biến số, hay vận dụng miền giá trị của toàn ánh để đổi biến một cách hợp

58

Page 64: Đ—I H¯C QU¨C GIA HÀ N¸I TRƯ˝NG Đ—I H¯C KHOA H¯C TÜ NHIÊN KHOA … · 2016-05-09 · Đ—I H¯C QU¨C GIA HÀ N¸I TRƯ˝NG Đ—I H¯C KHOA H¯C TÜ NHIÊN KHOA TOÁN

lý,...Nếu f là một song ánh, tức vừa là một đơn ánh vừa là một toàn ánh, thì ta có thể vận

dụng cả hai điều trên.

Phần đầu của mục này, ta chủ yếu đi xét một số bài PTH vận dụng tính đơn ánhcủa hàm số. Sau đó, ta sẽ đi tìm hiểu kĩ hơn về cách sử dụng tính toàn ánh vào việc giảivà tạo ra các bài toán PTH mới.

Bài toán 3.4.1. Tìm hàm số f (x) xác định trên R thỏa mãn

f ( f (x)+2y) = 4x+4y+3, ∀x,y ∈ R.

LỜI GIẢI. Nhận xét f (x) là đơn ánh. Thật vậy, giả sử f (x1) = f (x2) thì

f ( f (x1)+2y) = f ( f (x2)+2y)⇒ 4x1 +4y+3 = 4x2 +4y+3, ∀y ∈ R.

Suy ra x1 = x2. Vậy f là đơn ánh.Ta có f ( f (x)+2y) = 4x+4y+3 = f ( f (y)+2x), vì f đơn ánh nên f (x)+2y = f (y)+2x,

hay f (x)−2x = f (y)−2y,∀x,y∈R. Do đó f (x)−2x = c, c∈R. Thay lại f (x) = 2x+c

vào điều kiện ta có c = 1. Vậy nghiệm của bài toán là f (x) = 2x+1,∀x ∈ R.

Bài toán 3.4.2. Tìm tất cả các hàm f : R→ R thỏa mãn

f (2 f (x)− y) = 2x+ y+6, ∀x ∈ R. (1)

LỜI GIẢI. Bằng cách xét riêng biến x, dễ dàng chứng minh f là một đơn ánh. Trong (1),thay y =−2x ta có f (2 f (x)+2x) = 6,∀x ∈ R.

Suy ra f (2 f (x)+ 2x) = f (2 f (y)+ 2y),∀x,y ∈ R. Vì f đơn ánh nên ta có 2 f (x)+ 2x =2c,∀x ∈ R, với c ∈ R bất kì. Thay f (x) =−x+ c vào (1) ta suy ra c = 2.Vậy f (x) =−x+2,∀x ∈ R.

Nhận xét: Ta có bài toán tổng quát, với cách giải tương tự, như sau: Tìm hàm f thỏamãn f [a f (x)+by] = cx+dy+1,∀x,y ∈ R, trong đó a,b,c,d là các số thực khác 0.

Bài toán 3.4.3. (IMO 1997, Shortlist) Tồn tại hay không các hàm số f (x),g(x) xác định

trên R thỏa mãn các điều kiện f (g(x)) = x2, g( f (x)) = x3, ∀x ∈ R.

LỜI GIẢI. Giả sử tồn tại các hàm f (x),g(x) thỏa mãn bài toán. Từ g( f (x)) = x3 ta suyra f (x) đơn ánh, vì nếu f (x1) = f (x2)⇒ x3

1 = x32⇒ x1 = x2. Do đó f (1), f (−1), f (0) là

ba giá trị phân biệt. Nhưng ta lại có [ f (x)]2 = f (g( f (x))) = f (x3) nên f (0), f (1), f (−1)là 3 nghiệm của phương trình t2 = t, mâu thuẫn với f (0), f (1), f (−1) là 3 giá trị phânbiệt. Vậy không tồn tại các hàm thỏa mãn bài toán.

59

Page 65: Đ—I H¯C QU¨C GIA HÀ N¸I TRƯ˝NG Đ—I H¯C KHOA H¯C TÜ NHIÊN KHOA … · 2016-05-09 · Đ—I H¯C QU¨C GIA HÀ N¸I TRƯ˝NG Đ—I H¯C KHOA H¯C TÜ NHIÊN KHOA TOÁN

Bài toán 3.4.4. (IMO 1988, Shortlist) Tìm tất cả các hàm số f : N→ N thỏa mãn điều

kiện f ( f (n)+ f (m)) = n+m, ∀m,n ∈ N.

LỜI GIẢI. Nhận xét f là đơn ánh. Thật vậy, giả sử f (m) = f (n) khi đó

n+m = f ( f (n)+ f (m)) = f ( f (n)+ f (n)) = n+n = 2n→ m = n.

Để tận dụng tính chất đơn ánh ta có nhận xét sau đây

f ( f (n)+ f (n)) = 2n = (n+1)+(n−1) = f ( f (n+1)+ f (n−1)).

Từ đó, theo tính chất của đơn ánh ta suy ra rằng

f (n)+ f (n) = f (n+1)+ f (n−1)

⇐⇒ f (n+1)− f (n) = f (n)− f (n−1) ∀n ∈ N.

Nên ta có f (n+1)− f (n) = c, trong đó c là hằng số tự nhiên.Bằng quy nạp ta chứng minh được rằng f (n) = nc+ f (0), ∀n ∈ N.

Bây giờ nếu đặt f (0) = b, với b là hằng số tự nhiên, thì ta có f (n) = nc+b.Thay lại vào phương trình hàm ban đầu ta có

n+m = f ( f (n)+ f (m)) = f ((nc+b)+(mc+b)) = (n+m)c2 +2bc+b,

điều này đúng với mọi n,m ∈ N. Từ đó ta có c = 1 và b = 0.Như vậy, f (n) = n,∀n ∈ N. Hàm số này thỏa mãn bài toán.Vậy nghiệm của bài toán là f (n) = n, ∀n ∈ N.

Nhận xét: Ta còn có kết quả mạnh hơn f (m)+ f (n) = f (p)+ f (q), ∀ m+n = p+q. Từđó rút ra được phương trình f (n+1)− f (n) = f (n)− f (n−1). Đến đây ngoài cách tiếptục làm như trên, ta còn có thể tìm ra công thức tổng quát của f (n) bằng cách sử dụngphương trình sai phân.Ngoài cách giải trên, ta còn có thể tham khảo cách giải dưới đây.Lời giải 2.Rõ ràng, phương trình đồng nhất f (x) = x là nghiệm của bài toán. Ta đi chỉ ra điều này,bằng cách chỉ ra f (1) = 1 sau đó dùng quy nạp.Giả sử f (1) = t > 1. Đặt s = f (t−1) > 0.Ta thấy rằng nếu f (m) = n thì f (2n) = f ( f (m) + f (m)) = 2m. Vì thế, f (2t) = 2 vàf (2s) = 2t−2. Do đó 2s+2t = f ( f (2s)+ f (2t)) = f (2t) = 2 suy ra t < 1, mâu thuẫn.Điều này chứng tỏ f (1) = 0 hoặc f (1) = 1.Nếu f (1) = 0 thì f (0) = f ( f (1)+ f (1)) = 2, f (4) = f ( f (0)+ f (0)) = 0 = f (1), mâuthuẫn do f đơn ánh. Nên f (1) = 1.

60

Page 66: Đ—I H¯C QU¨C GIA HÀ N¸I TRƯ˝NG Đ—I H¯C KHOA H¯C TÜ NHIÊN KHOA … · 2016-05-09 · Đ—I H¯C QU¨C GIA HÀ N¸I TRƯ˝NG Đ—I H¯C KHOA H¯C TÜ NHIÊN KHOA TOÁN

Bây giờ, bằng quy nạp, nếu f (n) = n thì f (n + 1) = f ( f (n)+ f (1)) = n + 1. Dẫn đếnkết luận f (n) = n, ∀n ∈ N.

Một cách tổng quát: Với f : N→ N và k ∈ N∗, ta có

f (k

∑i=1

f (ni)) =k

∑i=1

ni, ∀ni ∈ N ⇐⇒ f (n) = n, ∀n ∈ N

Không khó để ta có thể nhận thấy cách giải bài toán tổng quát này dựa vào bài toán banđầu, tức với k = 2. Thật vậy, ta giữ nguyên mọi biến nk với k ≥ 3, chỉ để hai biến n1,n2

thay đổi sao cho tổng không đổi, sử dụng tính đơn ánh ta sẽ suy ra kết quả quen thuộcf (m)+ f (n) = f (p)+ f (q), ∀ m +n = p +q. Từ đó giải quyết được bài toán tổng quáttrên. Dưới đây là một số bài toán tương tự.

Bài toán 3.4.5. Chứng minh rằng không tồn tại hàm f : Z→ Z thỏa mãn

f (m+ f (n)) = f (m)−n, ∀m,n ∈ Z.

LỜI GIẢI. Dễ thấy f là đơn ánh. Với m = n = 0 ta có f ( f (0)) = f (0) suy ra f (0) = 0,do tính đơn ánh của f . Chỉ lấy m = 0 ta được f ( f (n)) =−n. Suy ra

f ( f (m)+ f (n)) = f ( f (m))−n =−m−n, ∀m,n ∈ Z.

Nên với mọi số nguyên m,n, p,q thỏa mãn m+n = p+q ta có

f ( f (m)+ f (n)) =−m−n =−p−q = f ( f (p)+ f (q)),

suy ra f (m)+ f (n) = f (p)+ f (q), ∀m + n = p + q. Đến đây, một cách tương tự suy raf (n) = n, ∀n ∈ Z. Thử lại dễ thấy hàm này không thỏa mãn bài toán. Vậy ta có ĐPCM.

Bài toán 3.4.6. Cho k ∈ N. Tìm hàm f : N∗→ N∗ thỏa mãn

f ( f (n)+m) = n+ f (m+ k). (1)

LỜI GIẢI. Trước hết, dễ thấy rằng f là đơn ánh. Mặt khác, từ (1) ta có

f ( f (n)+ f (1)) = n+ f ( f (1)+ k) = n+1+ f (k + k) = f ( f (n+1)+ k).

Vì f là đơn ánh nên ta có f (n)+ f (1) = f (n+1)+ k suy ra

f (n+1)− f (n) = f (1)− k, ∀n ∈ N∗.

61

Page 67: Đ—I H¯C QU¨C GIA HÀ N¸I TRƯ˝NG Đ—I H¯C KHOA H¯C TÜ NHIÊN KHOA … · 2016-05-09 · Đ—I H¯C QU¨C GIA HÀ N¸I TRƯ˝NG Đ—I H¯C KHOA H¯C TÜ NHIÊN KHOA TOÁN

Đặt f (1)− k = a. Khi đó, bằng quy nạp ta dễ dàng suy ra

f (n) = na+ k, ∀n ∈ N.

Thay lại vào (1) ta được a2n = n, ∀n ∈ N∗.Từ đó a2 = 1, ta thấy rằng a 6=−1 vì nếu không f (n) =−n+ k sẽ âm với n đủ lớn, mâuthuẫn. Do đó a = 1 và f (n) = n + k, hàm này thỏa mãn bài toán. Vậy nghiệm của bàitoán là f (n) = n+ k.

Bài toán 3.4.7. (IMO 1987) Chứng minh rằng không tồn tại hàm số f : N→N thỏa mãn

f ( f (n)) = n+1987.

LỜI GIẢI. Giả sử tồn tại hàm f thỏa mãn bài toán. Dễ thấy f đơn ánh.Giả sử f (n) chỉ không nhận k giá trị phân biệt c1,c2, ...,ck trong N.Khi đó, f ( f (n)) không nhận 2k giá trị tự nhiên phân biệt c1,c2, ...,ck vàf (c1), f (c2), ..., f (ck), vì f là đơn ánh nên các giá trị f (ci) là khác nhau.Bây giờ, nếu w 6= c1, ...,ck, f (c2), ..., f (ck) thì tồn tại m ∈ N sao cho f (m) = w.Vì w 6= f (ci), m 6= ci, nên tồn tại n ∈ N sao cho f (n) = m. Khi đó, f ( f (n)) = w, điềunày chỉ ra rằng f ( f (n)) chỉ không nhận 2k giá trị c1, ...,ck, f (c1), ..., f (ck) và nhận mọigiá trị khác.Nhưng khi n thay đổi trên N thì f ( f (n)) = n + 1987 chỉ không thể nhận 1987 giá trị là0,1,2, ...,1986 và 2k 6= 1987, dẫn đến mâu thuẫn.Vậy không tồn tại hàm f thỏa mãn bài toán (ĐPCM).

Nhận xét: Ta có thể thay 1987 bởi hằng số tự nhiên lẻ bất kì mà không làm bài toánthay đổi. Mở rộng ra, nếu f m(n) = n + k, ∀n ∈ N, trong đó m ≥ 2,k là các hằng số tựnhiên, để có bài PTH với kết quả và cách giải tương tự ta phải có điều kiện giữa k và m

là: ”k không biểu diễn được dưới dạng 2m−1q, với q ∈ N nào đó".Với việc chứng minh bằng quy nạp khẳng định " f p(n),2 ≤ p ≤ m, chỉ không nhận

2p−1q giá trị", ta có khẳng định điều kiện trên là đúng đắn.Một hướng để thay đổi bài toán là thay n trong vế phải bởi hàm g(n) cho trước. Chẳng

hạn: Tìm điều kiện của k ∈N để tồn tại hàm f : N→N thỏa mãn f ( f (n)) = kn9,∀n∈N,hay: Tồn tại hay không hàm f : N→N thỏa mãn f ( f (n)) = n2. Vấn đề này xin dành chobạn đọc quan tâm.

Bài toán 3.4.8. (IMO 1992) Tìm tất cả các hàm f : R→ R thỏa mãn

f (x2 + f (y)) = y+[ f (x)]2, ∀x,y ∈ R. (*)

62

Page 68: Đ—I H¯C QU¨C GIA HÀ N¸I TRƯ˝NG Đ—I H¯C KHOA H¯C TÜ NHIÊN KHOA … · 2016-05-09 · Đ—I H¯C QU¨C GIA HÀ N¸I TRƯ˝NG Đ—I H¯C KHOA H¯C TÜ NHIÊN KHOA TOÁN

LỜI GIẢI. Ta sẽ sử dụng tính song ánh của f để chỉ ra f (0) = 0.Đây là mấu chốt của bài toán. Vì nếu có f (0) = 0, lấy x = 0 ta có f ( f (y)) = y, ∀y ∈ R.Đồng thời hàm f không giảm. Thật vậy, với mọi t ∈ R lấy y = f (t) suy ra f (y) =f ( f (t)) = t. Khi đó thay vào (*) ta có

f (x2 + t) = f (x2 + f (y)) = y+[ f (x)]2

= f (t)+ [ f (x)]2 ≥ f (t), ∀x ∈ R, x 6= 0.

Do đó f không giảm, và thỏa mãn f ( f (x)) = x, ∀x ∈ R. Đây là bài toán quen thuộc, cónghiệm là f (x) = x,∀x ∈ R, hàm này cũng thỏa mãn (*).Bây giờ, ta chứng minh f (0) = 0.

Trước tiên khi cố định x, xét riêng biến y ta dễ dàng suy ra f là song ánh.Do đó, tồn tại a sao cho f (a) = 0, đặt f (0) = b.Trong (*) lấy x = 0 ta có f ( f (y)) = y+[ f (0)]2 = y+b2, ở đây lấy y = a ta có f ( f (a)) =a+b2 hay b = a+b2 (1). Trong (*) lấy x = a,y = 0 ta có f (a2 +b) = 0 mà f (a) = 0, f

đơn ánh nên a2 +b = a (2). Từ (1) và (2) ta suy ra a = b = 0. Do đó f (0) = 0.Vậy f (x) = x, ∀x ∈ R là hàm cần tìm.Nhận xét. Ta thấy rằng việc tình được f (0) = 0 là rất quan trọng. Vận dụng tính songánh của f cho ta một cách tính giá trị của f (0) khá đơn giản. Ngoài ra, còn có nhiềucách khác nhau để tính f (0). Ta lấy ví dụ một cách sau đây:Đặt f (0) = t, cho x = y = 0 ta có f (t) = t2. Ta cũng dễ dàng có

f (x2 + t) = [ f (x)]2 và f ( f (x)) = x+ t2.

Bây giờ ta đi tính f (t2 + f (1)2) theo hai cách. Đầu tiên, ta có

f (t2 + f (1)2) = f ( f (1)2 + f (t)) = t + f ( f (1))2

= t +(1+ t2)2 = 1+ t +2t2 + t4.

Cách thứ hai, ta có

f (t2 + f (1)2) = f (t2 + f (1+ t)) = 1+ t + f (t)2 = 1+ t + t4.

Từ đó, ta có t = 0 hay f (0) = 0.

Mở rộng bài toán này, ta có bài toán dưới đây:

Bài toán 3.4.9. Cho ∈ N∗. Tìm tất cả các hàm f : R→ R thỏa mãn

f (x2n + f (y)) = y+[ f (x)]2n, ∀x,y ∈ R. (*)

63

Page 69: Đ—I H¯C QU¨C GIA HÀ N¸I TRƯ˝NG Đ—I H¯C KHOA H¯C TÜ NHIÊN KHOA … · 2016-05-09 · Đ—I H¯C QU¨C GIA HÀ N¸I TRƯ˝NG Đ—I H¯C KHOA H¯C TÜ NHIÊN KHOA TOÁN

LỜI GIẢI. Cách giải bài toán này tương tự như bài toán trên. Ta cũng dễ dàng tính đượcf (0) nhờ sử dụng tính song ánh của hàm f , nhưng ở đây nếu dùng cách khác thì sẽ khókhăn hơn nhiều.

XÉT BÀI TOÁN :

Bài toán 3.4.10. Tìm tất cả các hàm f : R→ R thỏa mãn

f (x+ f (y)) = y+ f (x), ∀x,y ∈ R.

Thoạt nhìn, ta có thấy nó đơn giản hơn nhiều so với bài toán trên, nhưng để giảinó triệt để trên R thì cần thêm điều kiện nữa (nếu trên Q thì không cần thiết, vì khi đóta không khó để quy về phương trình hàm Cauchy trên Q). Chẳng hạn, thêm điều kiệnf (x)≥ 0 với mọi x≥ 0, hay f đơn điệu, hoặc mạnh hơn là f liên tục trên R,...

Nếu có thêm giả thiết f đơn điệu thì nó là một ý của bài toán Italy 1999. Ta xét 2 mởrộng bài toán như sau:

Bài toán 3.4.11. Tìm hàm f : R→ R đơn điệu trên R, thỏa mãn

f (x2n+1 + f (y)) = y+[ f (x)]2n+1, ∀x,y ∈ R,

ở đây, n là số tự nhiên bất kì.

Bài toán 3.4.12. Với n ∈ N∗. Tìm hàm f : R→ R đơn điệu thỏa mãn

f (x+[ f (y)]2n+1) = y2n+1 + f (x), ∀x,y ∈ R.

Gợi ý. Hai bài toán này cũng vận dụng tính song ánh của f để tính f (0) = 0. Sau đókhéo léo chỉ ra f cộng tính. Như vậy bài toán đưa về PTH Cauchy trong lớp hàm đơnđiệu. Bạn đọc có thể xem lại trong phần PTH Cauchy.

Bài toán 3.4.13. (Balkan 2000) Tìm hàm f : R→ R thỏa mãn

f (x f (x)+ f (y)) = [ f (x)]2 + y, ∀x,y ∈ R. (2)

LỜI GIẢI. Ta chứng minh f là song ánh. Thật vậy: trước tiên bằng việc xét riêng biến y,dễ thấy rằng f là đơn ánh. Ngoài ra, cũng cố định biến x khi cho biến y chạy trên toàn Rthì ta có f cũng nhận giá trị trên toàn R, hay f là toàn ánh. Như vậy, f là song ánh.Khi đó, tồn tại duy nhất a sao cho f (a) = 0. Trong (2) lấy x = 0 ta đượcf ( f (y)) = [ f (0)]2 +y. Cũng trong (2), khi ta thay x = a ta được f ( f (y)) = y, do f (a) = 0.

64

Page 70: Đ—I H¯C QU¨C GIA HÀ N¸I TRƯ˝NG Đ—I H¯C KHOA H¯C TÜ NHIÊN KHOA … · 2016-05-09 · Đ—I H¯C QU¨C GIA HÀ N¸I TRƯ˝NG Đ—I H¯C KHOA H¯C TÜ NHIÊN KHOA TOÁN

Từ 2 khẳng định trên ta suy ra f (0) = 0, nhưng theo tính đơn ánh của f thì ta suy raf (a) = 0 ⇐⇒ a = 0. Từ đó f (0) = 0 và f ( f (y)) = y,∀y ∈ R.Trong (2) cho y = 0 ta được f (x f (x)) = [ f (x)]2⇒ f ( f (x) f ( f (x))) = [ f ( f (x))]2.Kết hợp f ( f (x)) = x suy ra f (x f (x)) = x2, do đó [ f (x)]2 = x2. Suy ra với mọi x ∈ R thìhoặc f (x) = x hoặc f (x) =−x.Ta chứng minh không đồng thời tồn tại a,b 6= 0 sao cho f (a) = a và f (b) =−b. Giả sửngược lại, tồn tại a,b như thế. Thay x = a,y = b vào (2) ta có f (a2−b) = a2 +b, nhưng[ f (x)]2 = x2, ∀x ∈ R nên ta phải có (a2 +b)2 = (a2−b)2 hay 4a2b = 0, điều này khôngthể xảy ra vì a,b 6= 0. Vì vậy f (x) = x, ∀x ∈ R hoặc f (x) =−x, ∀x ∈ R. Thử lại ta thấyhai hàm này thỏa mãn bài toán.Vậy bài toán có hai nghiệm là f (x) = x, ∀x ∈ R hoặc f (x) =−x, ∀x ∈ R.Nhận xét: Khi đã có [ f (x)]2 = x2, ∀x∈R, ta còn có thể giải tiếp bài toán bằng cách xét 2trường hợp hoặc f (1) = 1 hoặc f (1) =−1, tương ứng sẽ là hai nghiệm f (x) = x, ∀x∈Rhoặc f (x) =−x, ∀x ∈ R.

Việc mở rộng với số mũ cao hơn đối với bài toán "Balkan 2000" xin dành cho bạnđọc. Thay đổi một chút trong bài toán trên ta có bài toán tương tự sau.

Bài toán 3.4.14. Tìm f : R→ R thỏa mãn một trong hai điều kiện

(i). f (x2 + f (y)) = y+ x f (x) ∀x,y ∈ R,

(ii). f ([ f (x)]2 + f (y)) = y+ x f (x) ∀x,y ∈ R.

LỜI GIẢI. Ta đi tìm f thỏa mãn (ii). Đối với (i) tương tự, ngoài ra có thể thấy hai điềukiện này có thể biến đổi về nhau.Ta cũng dễ thấy: f đơn ánh và f (0) = 0, f ( f (y)) = y ∀y ∈ R.Trong (ii), thay x bởi f (x) ta có f ([ f ( f (x))]2 + f (y)) = y+ f (x) f ( f (x)).Mặt khác, f ( f (y)) = y, ∀y ∈R nên f (x2 + f (y)) = y+x f (x), ∀x,y ∈R. Kết hợp (ii) thìf (x2 + f (y)) = f ([ f (x)]2 + f (y)) mà f đơn ánh nên x2 + f (y) = [ f (x)]2 + f (y). Suy ra[ f (x)]2 = x2, ∀x ∈ R.Tương tự trên ta chỉ ra rằng không đồng thời tồn tại a 6= 0, b 6= 0 mà f (a) = a, f (b) =−b.Thật vậy, giả sử tồn tại a,b như trên. Trong (ii) lấy x = a,y = b ta có f (a2−b) = a2 +b.Do [ f (x)]2 = x2, ∀x ∈ R nên (a2− b)2 = (a2 + b)2 suy ra a2b = 0, mâu thuẫn. Vậyf (x) = x ∀x ∈ R hoặc f (x) =−x ∀x ∈ R. Thử lại, hai hàm này thỏa mãn bài toán.

Bài toán 3.4.15. Các số dương p,q phải thỏa mãn điều kiện gì để tồn tại hàm số

f : R+→ R+ với tính chất f (x f (y)) = xpyq, ∀x,y ∈ R+.

LỜI GIẢI. Ta dễ thấy f toàn ánh trên R+, nên tồn tại a > 0 sao cho f (a) = 1. Thay y = a

ta có f (x) = aqxp, ∀x > 0. Suy ra x f (y) = aqxyp, f (x f (y)) = apq+qxpyp2.

65

Page 71: Đ—I H¯C QU¨C GIA HÀ N¸I TRƯ˝NG Đ—I H¯C KHOA H¯C TÜ NHIÊN KHOA … · 2016-05-09 · Đ—I H¯C QU¨C GIA HÀ N¸I TRƯ˝NG Đ—I H¯C KHOA H¯C TÜ NHIÊN KHOA TOÁN

Do đó, xpyq = apq+qxpyp2, ∀x,y ∈ R+. Điều này chỉ xảy ra với q = p2,a = 1.

Ngược lại, nếu q = p2 thì hàm f (x) = xp có tính chất

f (x f (y)) = xp[ f (y)]p = xp(yp)p = xpyp2= xpyq, ∀x,y ∈ R+.

Vậy p,q phải thỏa mãn hệ thức q = p2.

Ở trên, ta chỉ sử dụng tính toàn ánh trong việc chọn giá trị thích hợp của biến số.Tiếp theo, ta xét một số bài toán vận dụng miền giá trị của toàn ánh để giải PTH.

Bài toán 3.4.16. Tìm hàm f : R→ R là toàn ánh thỏa mãn với mọi x,y ∈ R thì

f ( f (x− y)) = f (x)− f (y).

LỜI GIẢI. Lấy y = 0, ta có f ( f (x)) = f (x)− f (0). Do f toàn ánh nên với mọi t ∈ R tồntại x sao cho f (x) = t. Suy ra f (t) = t− f (0), ∀t ∈ R. Thay lại vào PTH ban đầu ta dễsuy ra f (0) = 0, nên f (t) = t, ∀t ∈ R. Đây là nghiệm của bài toán.

Bài toán 3.4.17. Tìm hàm f : R→ R thỏa mãn điều kiện

f (x+ f (y)) = x+ f (y)+ x f (y), ∀x,y ∈ R.

Gợi ý. Ta thấy rằng f (x) ≡ −1 là nghiệm. Ta đi tìm nghiệm khác. Tồn tại a sao chof (a) 6= −1. Lấy y = a ta có f (x + f (a)) = x(1 + f (a))+ f (a), khi đó vế phải nhận giátrị trên toàn R khi x chạy trên toàn R, nên ta có f toàn ánh. Nếu chỉ lấy x = 0 ta đượcf ( f (y)) = f (y), ∀y ∈ R, kết hợp f toàn ánh ta suy ra f (x) = x, ∀x ∈ R. Nhưng thử lạithì hàm này không thỏa mãn bài toán. Vậy nghiệm của bài toán là f (x)≡−1.

Bài toán 3.4.18. (IMO 2002, Shortlist) Tìm hàm số f : R→ R thỏa mãn

f ( f (x)+ y) = 2x+ f ( f (y)− x), ∀x,y ∈ R.

LỜI GIẢI. Ta chỉ ra rằng f là toàn ánh. Thật vậy, lấy y = − f (x) ta có f (0) = 2x +f ( f (− f (x)−x)), hay f ( f (− f (x)−x)) = f (0)−2x. Do mọi số thực đều biểu diễn đượcdưới dạng f (0)−2x nên miền giá trị của f là R, hay f là toàn ánh. Suy ra với mỗi y sẽtồn tại z sao cho f (z) = y. Đặc biệt, tồn tại a thỏa mãn f (a) = 0.Lấy x = a, theo phương trình đầu, ta có

f (y)−a = f ( f (y)−a)+a ⇐⇒ f ( f (y)−a) = ( f (y)−a)+a.

Vì f toàn ánh nên với mỗi số thực x tồn tại số thực y sao cho x = f (y)− a. Vì vậyf (x) = x+a, ∀x ∈ R. Thử lại, hàm f (x) = x+a, với a là hằng số, thỏa mãn bài toán.

66

Page 72: Đ—I H¯C QU¨C GIA HÀ N¸I TRƯ˝NG Đ—I H¯C KHOA H¯C TÜ NHIÊN KHOA … · 2016-05-09 · Đ—I H¯C QU¨C GIA HÀ N¸I TRƯ˝NG Đ—I H¯C KHOA H¯C TÜ NHIÊN KHOA TOÁN

Để kết thúc mục này, ta đi xét một dạng PTH sử dụng miền giá trị của toàn ánhmột cách khá phức tạp, khó nhận thấy hơn, nhưng đặc sắc hơn rất nhiều.

Bài toán 3.4.19. Tìm hàm f : R→ R thỏa mãn

f (2x− f (y)) =f (x)+3x−2 f (y)

2, ∀x,y ∈ R. (*)

LỜI GIẢI. Ta để ý rằng 3x−2 f (y) = 2 f (2x− f (y))− f (x), do đó nếu cố định y thì hàmsố g(x) = 2 f (2x− f (y))− f (x) là hàm số bậc nhất theo biến x, nên g(x) là toàn ánh trênR. Do đó, hàm 2 f (u)− f (v) với hai biến u,v ∈ R sẽ nhận giá trị trên toàn R.Do đó, để xác định f (x) trên R có một cách "khá đặc biệt":Ta đi biểu diễn f (2 f (u)− f (v)) theo 2 f (u)− f (v), với u,v ∈ R. Ta làm như sau:Trong (*) thay x bởi f (y) ta có 2 f ( f (y)) = f ( f (y))+3 f (y)−2 f (y) = f ( f (y))+ f (y),suy ra f ( f (y)) = f (y), ∀y ∈ R. Trong (*), thay x bởi f (x) ta có

2 f (2 f (x)− f (y)) = f ( f (x))+3 f (x)−2 f (y). (2)

Từ (1) và (2) ta suy ra f (2 f (x)− f (y)) = 2 f (x)− f (y), ∀x,y ∈ R. Do miền giá trị củahàm 2 f (x)− f (y) với x,y ∈ R là toàn R nên f (x) = x, ∀x ∈ R. Thử lại, ta thấy hàmf (x) = x thỏa mãn bài toán.Vậy hàm số cần tìm là f (x) = x, ∀x ∈ R.Nhận xét. Ý tưởng của bài toán là không khó, nhưng được giấu kín: Với g là toàn ánhtrên R, h là hàm xác định trên R đã biết, khi đó nếu f (g(x)) = h(g(x)), ∀x ∈R ta suy raf ≡ h trên R.

Bài toán 3.4.20. (IMO 1999) Xác định tất cả các hàm f : R→ R thỏa mãn

f (x− f (y)) = f ( f (y))+ x f (y)+ f (x)−1, ∀x ∈ R. (**)

LỜI GIẢI. Ta thấy rằng f (x)≡ 0 không thỏa mãn bài toán. Nên tồn tại a sao cho f (a) 6= 0.Trong (**) lấy y = a ta có f (x− f (a)) = f ( f (a))+ x f (a)+ f (x)−1.

Đặt g(x) = f (x− f (a))− f (x), khi đó dễ thấy g là toàn ánh, do

g(x) = f (x− f (a))− f (x) = x f (a)−1+ f ( f (a)), f (a) 6= 0,

nhận giá trị trên toàn R khi x chạy trên toàn R.Ta sẽ tìm cách biểu diễn f ( f (x)− f (y)) theo f (x)− f (y).Đặt f (0) = c. Trong (**), thay x = f (y) ta có f (0) = 2 f ( f (y))+ [ f (y)]2−1. Suy ra

f ( f (y)) =c+1

2− [ f (y)]2

2, ∀y ∈ R. (i)

67

Page 73: Đ—I H¯C QU¨C GIA HÀ N¸I TRƯ˝NG Đ—I H¯C KHOA H¯C TÜ NHIÊN KHOA … · 2016-05-09 · Đ—I H¯C QU¨C GIA HÀ N¸I TRƯ˝NG Đ—I H¯C KHOA H¯C TÜ NHIÊN KHOA TOÁN

Thay x bởi f (x) vào (**) ta có

f ( f (x)− f (y)) = f ( f (y))+ f (x) f (y)+ f ( f (x))−1. (ii)

Thay (i) vào (ii) ta có

f ( f (x)− f (y)) =c+1

2− [ f (y)]2

2+ f (x) f (y)+

c+12− [ f (x)]2

2−1.

Suy ra f ( f (x)− f (y)) = c+[ f (x)− f (y)]2

2, ∀x,y ∈ R.

Đến đây, sử dụng tính toàn ánh của g ta suy ra f (x) = c− x2

2, ∀x ∈ R.

Do đó f ( f (x)) = c− [ f (x)]2

2, kết hợp với (i) ta có a =

a+12⇒ a = 1.

Nên f (x) =−x2

2−1, ∀x ∈ R. Thử lại ta thấy hàm này thỏa mãn (**).

Vậy nghiệm của bài toán là f (x) =−x2

2−1, ∀x ∈ R.

Nhận xét: Việc xét hàm g(x) toàn ánh như trên là "tương đối kĩ thuật", và việc có thểbiểu diễn hàm f (g(x)) theo g(x) đòi hỏi người ra đề phải tính toán hợp lý. Dưới đây làmột số bài toán tương tự.

Bài tập 3.4.1. Xác định tất cả các hàm f : R→ R thỏa mãn

f (x− f (y)) = f ( f (y))−2x f (y)+ f (x), ∀x,y ∈ R,

Đáp số. Hàm cần tìm là f (x) = x2.

Bài tập 3.4.2. Xác định tất cả các hàm f : R→ R thỏa mãn

2 f (3x−2 f (y)) = 4 f ( f (y))+3 f (x)−3x−1, ∀x,y ∈ R.

Đáp số. Hàm cần tìm là f (x) =−x+1.

Bài toán 3.4.21. (Tổng quát IMO 1999) Với a 6= 0,b,c là các số thực. Tìm hàm f : R→Rthỏa mãn f (x− f (y)) = f (x)+ f ( f (y))−ax f (y)−b f (y)− c, ∀x,y ∈ R. (E(a,b,c))

LỜI GIẢI. Ta thấy rằng việc tìm hàm f thỏa mãn E(−1,0,1) chính là bài toán IMO 1999phía trên.Ta có thể giải bài toán này bằng cách hoàn toàn tương tự. Với những gợi ý như sau:Trước tiên, hàm f (x) ≡ 0 là nghiệm của E(a,b,c) khi và chỉ khi c = 0. Ta sẽ đi tìmnghiệm f không tầm thường.

68

Page 74: Đ—I H¯C QU¨C GIA HÀ N¸I TRƯ˝NG Đ—I H¯C KHOA H¯C TÜ NHIÊN KHOA … · 2016-05-09 · Đ—I H¯C QU¨C GIA HÀ N¸I TRƯ˝NG Đ—I H¯C KHOA H¯C TÜ NHIÊN KHOA TOÁN

Khi đó, tồn tại u thực sao cho f (u) 6= 0. Trong E(a,b,c) lấy y = u, khi đó do a 6= 0 tadễ thấy g(x) = f (x− f (u))− f (x) sẽ nhận giá trị trên toàn R, hay g toàn ánh. Suy raf (x)− f (y) sẽ nhận giá trị trên toàn R khi x,y thay đổi trên R.Bây giờ, đặt f (0) = d. Trong E(a,b,c) thay x bởi f (y) ta có

f ( f (y)) =d + c

2+

b2

f (y)+a2[ f (y)]2. (1)

Thay x bởi f (x) trong E(a,b,c) ta có

f ( f (x)− f (y)) = f ( f (x))−a f (x) f (y)+ f ( f (y))−b f (y)− c. (2)

Từ (1) và (2) ta suy ra

f ( f (x)− f (y)) = d +b2[ f (x)− f (y)]+

a2[ f (x)− f (y)]2.

Từ đó, f (z) = d +b2

z+a2

z2,∀z ∈ R.

Thay z bởi f (y) và kết hợp với (1) ta suy ra d = c. Thử lại nghiệm đúng.

Vậy nghiệm của bài toán là f (z) = c+b2

z+a2

z2,∀z ∈ R.

MỘT SỐ BÀI TẬP VẬN DỤNG:

Bài tập 3.4.3. Cho hàm số f : R→ R thỏa mãn 4 f ( f (x)) = 2 f (x)+ x, ∀x ∈ R.

Chứng minh rằng f (0) = 0.

Bài tập 3.4.4. Xác định tất cả các hàm f : Z→ Z thỏa mãn

f (2 f (n)− f (m)) = 2n−m, ∀m,n ∈ Z.

Bài tập 3.4.5. (Latvia TST) Tìm f ,g : R→ R trong đó g là đơn ánh và

f (g(x)+ y) = g( f (y)+ x), ∀x,y ∈ Z.

Gợi ý. Dễ thấy f (g(0)+ y) = g( f (y)) và g( f (0)+ x) = f (g(x)). Từ đó suy ra

g( f (g(x))) = f (g(0)+g(x)) = g( f (g(0))+ x).

Do g đơn ánh nên f (g(x)) = f (g(0))+ x, nhưng f (g(x)) = g( f (0)+ x). Từ đó suy ranghiệm của bài toán là g(x) = x+a và f (x) = x+b, với a,b ∈ R tùy ý.

Bài tập 3.4.6. (Germany TST 2009) Tìm f : R→R biết rằng nếu x3 +x f (y)+ f (z) = 0thì [ f (x)]3 + y f (x)+ z = 0.

69

Page 75: Đ—I H¯C QU¨C GIA HÀ N¸I TRƯ˝NG Đ—I H¯C KHOA H¯C TÜ NHIÊN KHOA … · 2016-05-09 · Đ—I H¯C QU¨C GIA HÀ N¸I TRƯ˝NG Đ—I H¯C KHOA H¯C TÜ NHIÊN KHOA TOÁN

Gợi ý. Chứng minh f toàn ánh, đơn ánh và f (x) = 0 ⇐⇒ x = 0. Từ đó chỉ ra f (x) =x,∀x ∈ R.

Bài tập 3.4.7. (IMO 2009, Shortlist) Tìm các hàm f xác định trên R thỏa mãn

f (x f (x+ y)) = f (y f (x))+ x2, ∀x,y ∈ R.

Gợi ý. Chứng minh f toàn ánh, đơn ánh. Từ đó suy ra hàm cần tìm là f (x) = x, ∀x∈Rhoặc f (x) =−x, ∀x ∈ R.

Bài tập 3.4.8. Tìm hàm f : R→ R thỏa mãn

f ( f (x)+ y) =−2y+ f ( f (y)− x), ∀x,y ∈ R.

Bài tập 3.4.9. Tìm hàm f : R→ R thỏa mãn

f (x− f (y)) = 2 f (x)+ x+ f (y), ∀x,y ∈ R.

Gợi ý. Lấy x = f (y) ta có f ( f (y)) =− f (y)+f (0)

2.

Biểu diễn f ( f (x)− f (y)) theo f (x)− f (y) như sau f ( f (x)− f (y)) = −[ f (x)− f (y)]+f (0). Sau đó thay x bởi f (x)− f (y) ta được f ( f (x)−2 f (y)) =−[ f (x)−2 f (y)]+2 f (0).Từ đó suy ra f (0) = 0.Chỉ ra miền giá trị của f (x)−2 f (y) là R. Nên f (x) =−x, ∀x ∈ R.

Bài toán 3.4.22. (Balkan 2007) Tìm hàm f : R→ R thỏa mãn

f ( f (x)+ y) = f ( f (x)− y)+4 f (x)y, ∀x,y ∈ R.

Gợi ý. Ta tìm nghiệm khác nghiệm tầm thường f (x)≡ 0. Cách giải dùng miền giá trịtương tự bài toán IMO 1999.

Bài tập 3.4.10. Cho n ∈ N∗, giả sử f : R→ R là hàm liên tục thỏa mãn

f (0) = 0, f (1) = 1, và f n(x) = x. ∀x ∈ [0,1].Chứng minh rằng f (x) = x. ∀x ∈ [0,1].

Gợi ý. Sử dụng tính chất nếu f liên tục và đơn ánh thì f đơn điệu. Mà f (0) = 0 vàf (1) = 1 nên f đồng biến. Từ đây suy ra f (x) = x.

Bài tập 3.4.11. Tìm tất cả các hàm f : N→ N đơn ánh thỏa mãn

f ( f (n))≤ n+ f (n)2

, ∀n ∈ N.

70

Page 76: Đ—I H¯C QU¨C GIA HÀ N¸I TRƯ˝NG Đ—I H¯C KHOA H¯C TÜ NHIÊN KHOA … · 2016-05-09 · Đ—I H¯C QU¨C GIA HÀ N¸I TRƯ˝NG Đ—I H¯C KHOA H¯C TÜ NHIÊN KHOA TOÁN

Gợi ý. f (n) = n là hàm cần tìm.

Bài tập 3.4.12. (Romania 1986) Giả sử f ,g : N→ N thỏa mãn f là toàn ánh, g là đơn

ánh và thỏa mãn f (n)≥ g(n), ∀n ∈ N. Chứng minh rằng f ≡ g.

Gợi ý. Giả sử f (n) 6= g(n) với n ∈ N nào đó.Đặt A = {g(n) : f (n) 6= g(n)}. Bằng cách xét phần tử nhỏ nhất g(p) của A, đồng thời sửdụng tính song ánh của f , đơn ánh của g, ta suy ra tồn tại g(q)∈ A mà g(q) < g(p), mâuthuẫn. Suy ra ĐPCM .

Bài tập 3.4.13. (VMO 1993 - Bảng A) Tìm tất cả các hàm f : N∗→ N∗ thỏa mãn

f ( f (n)) = 1993.n1945, ∀n ∈ N∗.

Bài tập 3.4.14. Cho c ∈ R, 1 ≥ c > 0. Chứng minh rằng không tồn tại f : (0,+∞)→(0,+∞) thỏa mãn

f ( f (x)+1

f (x)) = x+ c, ∀x > 0. (*)

Gợi ý. Giả sử tồn tại f . Khi đó, f đơn ánh và với mọi x > c thì tồn tại y ≥ 2 sao chof (y) = x. Do f đơn ánh nên tồn tại a ∈ (0,2) để f (a) 6= c.- Nếu f (a) > c, theo nhận xét ban đầu tồn tại x ≥ 2 sao cho f (x) = f (a) suy ra x = a,mâu thuẫn.- Nếu f (a) < c thì

1f (a)

>1c≥ c, do đó tồn tại x để f (x) =

1f (a)

. Khi đó theo (*) ta sẽ có

x = a suy ra1

f (a)= f (a), hay f (a) = 1 nhưng f (a) < c, mâu thuẫn. Vậy ta có ĐPCM !

Bài tập 3.4.15. (Iran TST 2011) Tìm hàm f : R→ R toàn ánh thỏa mãn

f (x+ f (x)+2 f (y)) = f (2x)+ f (2y), ∀x,y ∈ R.

3.5 Phương pháp điểm bất động

Định nghĩa 3.1. Ta gọi x là điểm bất động của hàm số f nếu f (x) = x.

Phương pháp sử dụng điểm bất động trong việc giải các bài toán PTH là mộtphương pháp khó, khó để nhận biết và khó để vận dụng so với các phương pháp khác.Nhưng ngược lại, nếu áp dụng hợp lý thì nó lại mang lại lời giải hay và ấn tượng.

Đối với phương pháp này, khi sử dụng người ta thường có những lập luận sau đây:- Khi biết một điểm bất động thì ta đi xác định tập hợp các điểm bất động của hàm

đang xét. Để làm điều đó, ta thường kiểm tra tính duy nhất của điểm bất động, tích của

71

Page 77: Đ—I H¯C QU¨C GIA HÀ N¸I TRƯ˝NG Đ—I H¯C KHOA H¯C TÜ NHIÊN KHOA … · 2016-05-09 · Đ—I H¯C QU¨C GIA HÀ N¸I TRƯ˝NG Đ—I H¯C KHOA H¯C TÜ NHIÊN KHOA TOÁN

hai điểm bất động có là điểm bất động hay không? nghịch đảo của điểm bất động có làđiểm bất động không?...

- Các bài PTH giải bằng phương pháp điểm bất động thường là các bài PTH có tínhđối xứng. Vì thế ta có thể dễ dàng nhận ra các điểm bất động. Tập các điểm bất độngthường là tập hữu hạn. Thông thường, một bài toán PTH sử dụng tốt phương pháp nàythường là bài PTH có điểm bất động duy nhất.

Ta đi xem xét một bài toán sau đây để làm quen và hiểu rõ hơn về phương pháphay này.

Bài toán 3.5.1. (IMO 1994) Gọi S là tập các số thực lớn hơn −1.

Hãy tìm tất cả các hàm f : S→ S thỏa mãn các điều kiện:

(i) f (x+ f (y)+ x f (y)) = y+ f (x)+ y f (x), ∀x,y ∈ S

(ii) Hàm sốf (x)

xtăng thực sự trên các khoảng −1 < x < 0, 0 < x < +∞.

LỜI GIẢI. Trong (i) cho x = y, ta có f (x+ f (x)+x f (x)) = x+ f (x)+x f (x), ∀x ∈ S. Dođó với mọi x ∈ S thì x+ f (x)+ x f (x) là điểm bất động của hàm f .Ta đi xác định tập hợp các điểm bất động của hàm f .Từ (ii) cho ta phương trình điểm bất động f (x) = x có nhiều nhất 3 nghiệm, một nghiệm(nếu có) nằm trong (−1,0), một nghiệm bằng 0 và một nghiệm nằm trong (0,∞).Giả sử u ∈ (−1,0) là một điểm bất động của f .Trong phương trình hàm (i), cho x = y = u ta được f (2u + u2) = 2u + u2. Hơn nữa,2u + u2 ∈ (−1,0) khi u ∈ (−1,0), do tính duy nhất của điểm bất động trong khoảng(−1,0) suy ra 2u+u2 = u, nhưng khi ấy u =−1 hoặc u = 0, mâu thuẫn với u ∈ (−1,0).Do vậy, hàm f không có điểm bất động nằm trong khoảng (−1,0).Hoàn toàn tương tự, hàm f không có điểm bất động thuộc (0,∞).Như thế 0 là điểm bất động duy nhất có thể có của hàm f .Do ∀x∈ S, x+ f (x)+x f (x) là điểm bất động nên x+ f (x)+x f (x) = 0, hay f (x) =

−x1+ x

với mọi x ∈ S.

Kiểm tra lại, với f (x) =−x

1+ xthì x+ f (y)+ x f (y) = x− (1+ x)y

1+ y=

x− y1+ y

, suy ra

f (x+ f (y)+ x f (y)) = f (x− y1+ y

) =− x−y

1+y

1+ x−y1+y

=y− x1+ x

= f (y+ f (x)+ y f (x)),

vàf (x)

x=−1

1+ xtăng thực sự trên các khoảng −1 < x < 0, 0 < x.

Như vậy, hàm f (x) =−x

1+ xthỏa mãn các điều kiện (i) và (ii) của bài toán.

Vậy hàm số cần tìm là f (x) =−x

1+ xvới mọi x 6= 0,x >−1.

72

Page 78: Đ—I H¯C QU¨C GIA HÀ N¸I TRƯ˝NG Đ—I H¯C KHOA H¯C TÜ NHIÊN KHOA … · 2016-05-09 · Đ—I H¯C QU¨C GIA HÀ N¸I TRƯ˝NG Đ—I H¯C KHOA H¯C TÜ NHIÊN KHOA TOÁN

Nhận xét: Việc nhận ra x+ f (x)+x f (x) là điểm bất động tương đối dễ. Điểm mấu chốtcủa bài toán là tận dụng (ii) để thấy tính duy nhất của điểm bất động trong mỗi khoảng−1 < x < 0 và x > 0, và lập luận nếu đã có 1 điểm bất động u thì ta suy ra điểm bất động2u+u2 6= u cùng nằm trong mỗi khoảng trên, để từ đó ta có kết luận quan trọng hàm f

có điểm bất động duy nhất x = 0, giúp giải quyết bài toán dễ dàng. Một cách tương tựbài PTH này, ta có một số bài toán như sau:

Bài toán 3.5.2. Chứng minh rằng không tồn tại hàm f : R→ R thỏa mãn các điều kiện

(i) −x+ y = f (x+ f (y)+ y2 f (y)) ∀x,y ∈ R

(ii)f (x)

xgiảm thực sự trên (−∞,0) và tăng thực sự trên (0,+∞).

LỜI GIẢI. Giả sử f là hàm thỏa mãn bài toán.Trước tiên ta đi chứng minh nếu f có điểm bất động a thì a = 0.Từ (ii) ta thấy rằng hàm f chỉ có tối đa 3 điểm bất động, một điểm bất động (nếu có)nằm trong khoảng x < 0, một điểm bằng 0 và một điểm nằm trong khoảng x > 0.Giả sử f có điểm bất động a thỏa mãn a > 0.

Khi đó, trong (i) ta cho y = a, ta được a−x = f (x+a+a3), ∀x∈R. Ở đây, lấy x =−a3

2,

ta được a +a3

2= f (a +

a3

2). Suy ra a +

a3

2cũng là điểm bất động của hàm f , ngoài ra

khi a > 0 ta có a +a3

2> a > 0. Do đó f có hai điểm bất động phân biệt trên khoảng

x > 0, mâu thuẫn. Do vậy f không có điểm bất động trên khoảng x > 0.Tương tự như thế, ta cũng dễ dàng chứng minh được rằng f không có điểm bất độngthỏa mãn x < 0.Do vậy nếu có điểm bất động a, thì ta suy ra a = 0.

Trong (i) ta lấy y = 0, ta được−x = f (x+ f (0)), ∀x ∈R. Ở đây, lấy x =− f (0)2

, ta được

f (0)2

= f (f (0)

2), như vậy hàm f có điểm bất động

f (0)2

, suy raf (0)

2= 0 hay f (0) = 0.

Bây giờ ta đi chứng minh không tồn tại t 6= 0 sao cho f (t) = 0.Thật vậy, giả sử tồn tại t 6= 0 mà f (t) = 0. Khi đó, trong (i) ta cho y = t, ta đượct−x = f (x), ∀x ∈R. Tiếp theo, ta cho x =

t2

ta đượct2

= f (t2), hay

t2

là điểm bất động

khác 0 của hàm f , do t 6= 0. Điều này mâu thuẫn với khẳng định ở trên.Do vậy, không tồn tại t 6= 0 mà f (t) = 0, hay f (x) = 0 ⇐⇒ x = 0.Trong (i) lấy y = x, ta có 0 = f (x+ f (x)+ x2 f (x)), ∀x ∈ R.

Từ đây ta có x+ f (x)+ x2 f (x) = 0 hay f (x) =− x1+ x2 , ∀x ∈ R.

Thử lại, ta thấy tuy f (x) =− x1+ x2 thỏa mãn (ii), nhưng lại không thỏa mãn (i).

Do vậy không tồn tại hàm f thỏa mãn bài toán. (ĐPCM) !

Nhận xét: Ý tưởng của bài toán dựa vào hai kết quả mấu chốt: f có điểm bất động duy

73

Page 79: Đ—I H¯C QU¨C GIA HÀ N¸I TRƯ˝NG Đ—I H¯C KHOA H¯C TÜ NHIÊN KHOA … · 2016-05-09 · Đ—I H¯C QU¨C GIA HÀ N¸I TRƯ˝NG Đ—I H¯C KHOA H¯C TÜ NHIÊN KHOA TOÁN

nhất x = 0 và f (x) = 0 ⇐⇒ x = 0, từ đó giải tiếp bài toán.

Bài toán 3.5.3. Tìm hàm số f : R→ R thỏa mãn các điều kiện sau

(1) f (x3y+2xy− y f (x)) = xy3 +2xy− x f (y), ∀x ∈ R,

(2) Hàmf (x)

xnghịch biến trên x < 0 và đồng biến trên x > 0.

LỜI GIẢI. Trong (1) lấy y = x ta có f (x4 +2x2− x f (x)) = x4 +2x2− x f (x), ∀x ∈ R.Như vậy, với mọi x ∈ R thì x4 +2x2− x f (x) là điểm bất động của f .Ta đi tìm tập hợp các điểm bất động của f .Từ (2) ta thấy rằng f có tối đa 3 điểm bất động: một điểm bất động (nếu có) nằm trongkhoảng x > 0, một điểm nằm trong khoảng x < 0 và một điểm bất động x = 0.Ta chứng minh f không có điểm bất động a > 0. Thật vậy, giả sử tồn tại a > 0 màf (a) = a, khi đó a4 +2a2−a f (a) = a4 +a2 > 0 cũng là điểm bất động dương của f . Dof có một điểm bất động dương duy nhất nên a = a4 +a2 ⇐⇒ a3 +a = 1, phương trìnhnày có nghiệm dương duy nhất.Trong (1) lấy x = a kết hợp a3 +a = 1, sau khi rút gọn ta có

f (y) = a(y3 +2y− f (y)) ⇐⇒ f (y) =a

1+a(y3 +2y), ∀y ∈ R.

Nhưng với hàm f (y) =a

1+a(y3 +2y) vừa tìm được, không khó để ta thấy rằng nó không

thỏa mãn (1).Điều này chứng tỏ giả sử trên là sai, hay f không có điểm bất động dương.Tiếp theo, nếu f có điểm bất động x < 0, thì x4 + 2x2− x f (x) = x4 + x2 sẽ là điểm bấtđộng dương của f , mâu thuẫn với khẳng định vừa chứng minh ở trên. Do đó, f cũngkhông có điểm bất động âm.Mặt khác, với mọi x∈R thì x4 +2x2−x f (x) là điểm bất động của f , do đó f có duy nhấtđiểm bất động x = 0 và x4 + 2x2− x f (x) = 0, ∀x ∈ R. Suy ra f (x) = x3 + 2x, ∀x 6= 0,đẳng thức này cũng đúng với x = 0, do x = 0 là điểm bất động duy nhất của f . Nênf (x) = x3 +2x, ∀x ∈ R. Thử lại, ta dễ thấy hàm này thỏa mãn bài toán.Kết luận: hàm số cần tìm là f (x) = x3 +2x, ∀x ∈ R.

Bài toán 3.5.4. (IMO 1983) Tìm f : (0,+∞)→ (0,+∞) thỏa mãn

(i) f (x f (y)) = y f (x)(ii) f (x)→ 0 khi x→+∞.

LỜI GIẢI. Trong (i) cho x = y ta có f (x f (x)) = x f (x). Suy ra với mọi x > 0 thì x f (x) làđiểm bất động của f .Ta đi xác định tập hợp các điểm bất động của f .Ta thấy rằng f là đơn ánh, vì từ f (y1) = f (y2) lần lượt thay y bởi y1,y2 trong (i) ta suy

74

Page 80: Đ—I H¯C QU¨C GIA HÀ N¸I TRƯ˝NG Đ—I H¯C KHOA H¯C TÜ NHIÊN KHOA … · 2016-05-09 · Đ—I H¯C QU¨C GIA HÀ N¸I TRƯ˝NG Đ—I H¯C KHOA H¯C TÜ NHIÊN KHOA TOÁN

ra y1 f (x) = f (x f (y1)) = f (x f (y2)) = y2 f (x)⇒ y1 = y2.

Trong (i) cho x = y = 1 ta có f ( f (1)) = f (1)⇒ f (1) = 1. Do đó f có điểm bất động làx = 1. Bây giờ, ta thấy rằng:- Nếu x,y là 2 điểm bất định của f thì f (xy) = f (x f (y)) = y f (x) = yx, suy ra xy cũng làđiểm bất động của f .- Nếu x > 0 là điểm bất động của f , ta có

1 = f (1) = f (1x

x) = f (1x

f (x)) = x f (1x),

suy ra f (1x) =

1x

, hay1x

cũng là điểm bất động của f .

Ta chỉ ra rằng f chỉ có điểm bất động duy nhất x = 1.Thật vậy: nếu f có điểm bất động x > 1 thì x2 = x.x cũng là điểm bất động của f ,và ta dẫn đến xn là điểm bất động của f với n ∈ N∗. Do đó f (xn) = xn, điều này mâuthuẫn với điều kiện (ii), do khi x→ +∞ thì xn = f (xn)→ +∞. Còn nếu f có điểm bất

động 0 < x < 1 thì1x

> 1 cũng là điểm bất động của f . Và tương tự ta cũng dẫn đến

mâu thuẫn. Vậy x = 1 là điểm bất động duy nhất của f . Do đó x f (x) = 1, ∀x > 0 hay

f (x) =1x, ∀x > 0. Thử lại, hàm này thỏa mãn các điều kiện (i), (ii).

Vậy hàm số cần tìm là f (x) =1x, ∀x ∈ R.

Bài toán 3.5.5. Tìm tất cả các hàm f : R→ R thỏa mãn điều kiện

f (x+ f (y)) = x+ f (y)+ x f (y) ∀x,y ∈ R. (1)

LỜI GIẢI. Giả sử f là hàm thỏa mãn bài toán.Chọn y = 0, x = − f (0) từ (1) ta có f (0) = − f 2(0) suy ra f (0) = 0 hoặc f (0) = −1.Nếu f (0) = 0, trong (1) cho y = 0 ta suy ra f (x) = x, ∀x ∈ R, thay vào (1) ta cóx + y = x + y + xy, ∀x,y ∈ R. Suy ra xy = 0,∀x,y ∈ R, vô lí. Vậy f (0) = −1. Khi đó,trong (1) cho x = 0 ta có f ( f (y)) = f (y),∀y ∈ R. Vậy ∀y ∈ R thì f (y) là điểm bất độngcủa f .Ta đi xác định tập các điểm bất động của hàm f .Giả sử a là điểm bất động của hàm f , tức f (a) = a. Trong (1) cho x = 1, y = a ta có

f (1+a) = f (1+ f (a)) = 1+ f (a)+ f (a) = 1+2 f (a) = 1+2a,

do f ( f (y)) = f (y),∀y ∈ R nên ta có f (1+2a) = f ( f (1+a)) = f (1+a) = 1+2a.Mặt khác f (1 + 2a) = f (1 + a + f (a)) = (1 + a)+ f (a)+ (1 + a) f (a) = 1 + 3a + a2,

nên 1+2a = 1+3a+a2 suy ra a = 0 hoặc a =−1.Ta thấy rằng a = 0 là không thể vì f (0) =−1, do đó a =−1. Vậy f chỉ có điểm bất độngduy nhất a =−1 suy ra f (y) =−1, ∀y ∈R. Thử lại thấy rằng f (x) =−1 là nghiệm của

75

Page 81: Đ—I H¯C QU¨C GIA HÀ N¸I TRƯ˝NG Đ—I H¯C KHOA H¯C TÜ NHIÊN KHOA … · 2016-05-09 · Đ—I H¯C QU¨C GIA HÀ N¸I TRƯ˝NG Đ—I H¯C KHOA H¯C TÜ NHIÊN KHOA TOÁN

bài toán.

Tiếp theo, ta đi xem xét một vài tính chất của hàm f đối với tập hợp điểm bất động

Sn của hàm hợp f n, và tính chất giữa các tập Sn. Nó có thể giúp ích đối với phương pháp

này. Ta bắt đầu với định nghĩa sau:

Định nghĩa 3.2. Với hàm f bất kì, kí hiệu f 1 = f và f n+1 = f ( f n) với n ∈ N∗.Ta định nghĩa Sn là tập hợp các điểm bất động của f n.

Ta dễ dàng thấy rằng nếu x là điểm bất động của f thì x cũng là điểm bất động của f n,tức S1 ⊆ Sn.Nếu x là điểm bất động của fn thì f (x) cũng là điểm bất động của f n vìf n( f (x)) = f n+1(x) = f ( f n(x)) = f (x), nên tập f (Sn) nhận giá trị trong chính Sn.Ngoài ra, f là đơn ánh trên Sn vì nếu f (a) = f (b) với a,b ∈ Sn thì

a = f n(a) = f n−1( f (a)) = f n−1( f (b)) = f n(b) = b.

Điều này dẫn đến: nếu Sn là tập hữu hạn thì f là song ánh trên Sn.Vì từ g(x) = x suy ra g2(x) = g(g(x)) = g(x) = x nên các điểm bất động của g cũng làcác điểm bất động của g2. Lấy g = f , f 2, f 4, f 8, ... ta thu được S1 ⊆ S2 ⊆ S4 ⊆ S8 ⊆ . . .

Đây là tính chất hữu ích giữa các tập Sn.

Bài toán 3.5.6. (VietNam TST 1990) Tìm tất cả các hàm số f : R → R thỏa mãn

f ( f (x)) = x2−2 với mọi x ∈ R.

LỜI GIẢI. Giả sử rằng hàm f thỏa mãn bài toán tồn tại.Việc xem xét đến các tập S2 và S4 giúp ta giải quyết bài toán này.Tập hợp các điểm bất động của f 2 là tập hợp nghiệm của phương trình x = x2−2, tức làS2 = {−1,2}.Tập các điểm bất động của f 4 là nghiệm của phương trình x = x4−2x2 +2, giải phương

trình này ta suy ra S4 = {−1,2,−1±

√5

2}. Đặt c =

−1+√

52

và d =−1−

√5

2.

Vì f là song ánh trên S2, S4 và c,d ∈ S4 \S2 nên ta suy ra f (c) = c hoặc f (c) = d.Nếu f (c) = c thì f 2(c) = c suy ra c là điểm bất động của f 2, điều này không đúng.Vì vậy f (c) = d, tương tự f (d) = c. Khi đó c = f (d) = f ( f (c)) = f 2(c), mâu thuẫn.Vậy không tồn tại hàm f thỏa mãn bài toán.

Ta có một cách mở rộng bài toán như dưới đây, cách giải hoàn toàn tương tự, nhưngta cũng có cách khác không dùng đến tính chất của S2,S4.

76

Page 82: Đ—I H¯C QU¨C GIA HÀ N¸I TRƯ˝NG Đ—I H¯C KHOA H¯C TÜ NHIÊN KHOA … · 2016-05-09 · Đ—I H¯C QU¨C GIA HÀ N¸I TRƯ˝NG Đ—I H¯C KHOA H¯C TÜ NHIÊN KHOA TOÁN

Bài toán 3.5.7. Chứng minh rằng không tồn tại hàm f : R→ R thỏa mãn

f ( f (x)) = x2−q, q >34.

LỜI GIẢI. Đặt g(x) = x2−q. Hàm g(x) có 2 điểm bất động a,b là 2 nghiệm của phươngtrình g(x) = x2−q = x hay x2− x−q = 0. Ta có

g(g(x)) = x ⇐⇒ (x2−q)2−q = x

⇐⇒ (x2− x−q)(x2− x−q+1) = 0. (1)

Gọi u,v là 2 nghiệm của nhân tử thứ 2 trong vế trái của (1). Khi đó hàm g(g(x)) có 4điểm bất động là a,b,u,v.Ta có g(u) = v, g(v) = u. Thật vậy: vì u,v là 2 nghiệm của phương trình x2−x−q+1 = 0nên sử dụng hệ thức Viet và g(x) = x2−q ta sẽ có g(u) = v, g(v) = u.Giả sử tồn tại f thỏa mãn bài toán, tức f ( f (x)) = g(x). Ta có v = g(u) = f ( f (u)) suy raf (v) = g( f (u)). Tương tự, ta cũng có f (u) = g( f (v)). Thế thì nếu đặt f (u) = z ta có

g(g(z)) = g(g( f (u))) = g( f (v)) = f (u) = z.

Ta được z là điểm bất động của g(g(x)).Bây giờ, ta đi chứng minh z khác với 4 điểm bất động a,b,u,v.- Nếu z = a khí đó v = g(u) = f (z) = f (a) ta có f (v) = f ( f (a)) = a suy ra f (u) = z =a = f (v), khi đó g(u) = f ( f (u)) = f ( f (v)) = g(v) ⇐⇒ v = u, điều này là không thể.Như vậy z 6= a. Tương tự, z 6= b.- Nếu z = u ta có f (u) = u suy ra v = g(u) = f ( f (u)) = f (u) = u, mâu thuẫn. Nếu z = v

thì f (v) = f (z) = f ( f (u)) = g(u) = v và ta cũng dẫn đến u = g(v) = f ( f (v)) = f (v) = v,mâu thuẫn.Như vậy, z là điểm bất động thứ 5 của g(g(x)). Vô lý, vì g(g(x)) là đa thức bậc 4. Mâuthuẫn này chứng tỏ giả sử f tồn tại là sai.Vậy không tồn tại hàm f thỏa mãn bài toán.

Nhận xét. Ở trên, việc mở rộng với g(x) = x2− q thì điều kiện q >34

để đảm bảo

phương trình g(g(x)) = x ⇐⇒ (x2− x−q)(x2− x−q+1) = 0 có 4 nghiệm thực phânbiệt. Vì thế nếu ta lấy g(x) là hàm bậc hai tùy ý mà vẫn đảm bảo g(g(x)) = x có 4 nghiệmthực phân biệt, chẳng hạn g(x) = x2− x−3, thì ta cũng có thể giải tương tự. Nhưng nếuthay g(x) mà không đảm bảo điều trên thì bài toán sẽ khác, bạn đọc có thể suy nghĩ thêm.Lấy ví dụ, với g(x) = x2, bài toán mới là: Tìm hàm f : R→ R thỏa mãn f ( f (x)) = x2,có một nghiệm dễ nhận thấy là f (x) = |x|

√2, như vậy chí ít khác với bài toán ban đầu là

đã có nghiệm. Việc thay f 2 bởi f n, n ∈ N∗ đồng thời thay đổi g(x) có thể rất phức tạp.

77

Page 83: Đ—I H¯C QU¨C GIA HÀ N¸I TRƯ˝NG Đ—I H¯C KHOA H¯C TÜ NHIÊN KHOA … · 2016-05-09 · Đ—I H¯C QU¨C GIA HÀ N¸I TRƯ˝NG Đ—I H¯C KHOA H¯C TÜ NHIÊN KHOA TOÁN

Bài toán 3.5.8. (IMO 1996) Tìm tất cả các hàm f : N→ N thỏa mãn

f (m+ f (n)) = f ( f (m))+ f (n), ∀m,n ∈ N. (1)

LỜI GIẢI. Trong m = n = 0 ta có f (0+ f (0)) = f ( f (0))+ f (0). Suy ra f (0) = 0, từ đókhi cho m = 0 ta có f ( f (n)) = f (n), trường hợp riêng f ( f (0)) = f (0) = 0. Từ đẳng thứctrên ta có thể viêt lại (i) như sau f (m+ f (n)) = f (m)+ f (n).Theo trên, với mọi n ∈N thì f (n) là điểm bất động của f . Gọi k là điểm bất động khác 0bé nhất của hàm f . Nếu như không tồn tại k thì f (n) = 0 với mọi n∈N. Khi đó, f (n)≡ 0là một nghiệm của bài toán.Xét trường hợp tồn tại k như trên. Bằng quy nạp ta dễ dàng chứng minh được rằngf (qk) = qk với q ∈ N tùy ý.Bây giờ, ta lại đi xác định tập các điểm bất động của f .Giả sử f có điểm bất động n > k. Ta viết n = kq+ r, với q,r ∈N;0≤ r < k. Khi đó, ta cón = f (n) = f (r +kq) = f (r + f (kq)) = f (r)+ f (kq) = kq+ f (r). Suy ra f (r) = r, điềunày dẫn đến r = 0, do k > r≥ 0 là điểm bất động nhỏ nhất của f . Như thế, mọi điểm bấtđộng của f là bội của k.Tuy nhiên, với mọi n ∈ N thì f (n) là điểm bất động của f . Do đó, f (n) là bội của k vớimọi n.Ta giả sử các số nguyên không âm c1,c2, ...,ck−1 thỏa mãn f (r) = crk với 0≤ r < k, vàchọn c0 = 0.Thế thì hàm f tổng quát thỏa mãn bài toán là f (qk + r) = qk + crk, với 0 ≤ r < k. Dễdàng kiểm tra các hàm này thỏa mãn điều kiện bài toán.Thật vậy, cho m = ak + r, n = bk + s với 0≤ r,s < k thì

f ( f (m)) = f (m) = ak + crk, f (n) = bk + csk,

do đó f (m+ f (n)) = f (ak + crk +bk + csk) = ak +bk + crk + csk

và f ( f (m))+ f (n) = ak +bk + crk + csk, kéo theo (1) được thỏa mãn.Tóm lại, các hàm xác định như trên là nghiệm tổng quát của bài toán.

MỘT SỐ BÀI TẬP VẬN DỤNG:

Bài tập 3.5.1. Tìm tất cả các hàm f : R→ R thỏa mãn

f (x+2 f (y)) = 2x+ f (y)+ x f (y), ∀x,y ∈ R.

Bài tập 3.5.2. (Tournoi des villes 1996) Tìm tất cả các f : R→ R thỏa mãn f ( f (x)) =x2−1996 với mọi x ∈ R.

78

Page 84: Đ—I H¯C QU¨C GIA HÀ N¸I TRƯ˝NG Đ—I H¯C KHOA H¯C TÜ NHIÊN KHOA … · 2016-05-09 · Đ—I H¯C QU¨C GIA HÀ N¸I TRƯ˝NG Đ—I H¯C KHOA H¯C TÜ NHIÊN KHOA TOÁN

Bài tập 3.5.3. Chứng minh rằng không tồn tại hàm f : R→ R thỏa mãn các điều kiện

(i) f (−x+ y) = x+ f (y)+ y2 f (y) ∀x,y ∈ R

(ii)f (x)

xgiảm thực sự trên (−∞,0) và tăng thực sự trên (0,+∞).

Bài tập 3.5.4. Chứng minh rằng không tồn tại hàm f : R→ R thỏa mãn các điều kiện

(i) 2x+ y = f (x+2 f (y)+ y2 f (y)) ∀x,y ∈ R

(ii)f (x)

xgiảm thực sự trên (−∞,0) và tăng thực sự trên (0,+∞).

Bài tập 3.5.5. Tìm tất cả các hàm số f : R→ R thỏa mãn

f ( f (x)) = x2− x−3, ∀x ∈ R.

Bài tập 3.5.6. (Italy TST 2005) Cho hàm số f : {1,2, ...,1600} → {1,2, ...,1600} thỏa

mãn f (1) = 1 và f 2005 = x, ∀x ∈ {1,2, ...,1600}.a) Chứng minh rằng f có một điểm bất động khác 1.

b) Tìm tất cả n > 1600 sao cho với mọi hàm f : {1,2, ...,n}→ {1,2, ...,n} thỏa mãn các

điều kiện trên có ít nhất hai điểm bất động.

Gợi ý. Ta có các nhận xét sau:- NX 1: hàm f đơn ánh.- NX 2: Nếu f n1(x0) = x0, f n2(x0) = x0 thì f d(x0) = x0 với d = (n1,n2).- NX 3: Nếu n nhỏ nhất thỏa mãn f n(x0)= x0 thì n|2005 và tập A = {x0, f (x0), ..., f n−1(x0)}có tính chất f n(x) = x, ∀x ∈ A. Khi đó ta gọi tập A có tính chất Tn.

Giả sử f (x) 6= x, ∀x > 1. Khi đó {2,3, ...,1600} có thể phân hoạch thành p tập có tínhchất T5 và q tập có tính chất T401, và r tập có tính chất T2005, với p,q,r ∈ N. Suy ra5p+401q+2005r = 1599, phương trình này vô nghiệm dẫn đến giả sử trên là sai. Và tacó ĐPCM.

3.6 Phương pháp đưa về dãy số

3.6.1 Cơ sở lý thuyết phương trình sai phân

Trong phần này, ta xét nhiều đến phương trình sai phân (PTSP) tuyến tính thuần nhất.Bạn đọc quan tâm có thể tự tìm hiểu thêm về phương trình sai phân không thuần nhất, sẽkhông quá khó để bạn đọc có thể làm điều đó. Chúng ta bắt đầu với định nghĩa và cáchxác định nghiệm của phương trình sai phân trong các trường hợp đơn giản.

Xét dãy số thực xn, n ∈ N. Ta có định nghĩa sau:

79

Page 85: Đ—I H¯C QU¨C GIA HÀ N¸I TRƯ˝NG Đ—I H¯C KHOA H¯C TÜ NHIÊN KHOA … · 2016-05-09 · Đ—I H¯C QU¨C GIA HÀ N¸I TRƯ˝NG Đ—I H¯C KHOA H¯C TÜ NHIÊN KHOA TOÁN

Định nghĩa 3.3. PTSP tuyến tính của dãy xn là một biểu thức tuyến tính giữa các giá

trị của dãy xn tại các điểm khác nhau:

a0xn+k +a1xn+k−1 + ...+akxn = fn, ∀n ∈ N, (1)

trong đó k ∈N∗; ai; i = 0,1, ...,k với a0,ak 6= 0 là các hằng số thực, và fn là hàm số của

n; các giá trị xn cần tìm được gọi là ẩn.

Phương trình (1) được gọi là PTSP bậc k. Để tính được các giá trị xn ta phải cho trước

k giá trị liên tiếp của xn.

Định nghĩa 3.4. Nếu fn ≡ 0, ∀n ∈ N thì (1) trở thành

a0xn+k +a1xn+k−1 + ...+akxn = 0, ∀n ∈ N, (2)

phương trình này được gọi là PTSP tuyến tính thuần nhất bậc k.

Định nghĩa 3.5. Xét dãy xn, n ∈ N cùng với PTSP (2). Khi đó phương trình

a0λk +a1λ

k−1 + ...+ak−1λ +ak = 0 (3)

được gọi là phương trình đặc trưng của PTSP tuyến tính thuần nhất (2).

Bây giờ, ta trình bày (không chứng minh) cách xác định nghiệm tổng quát, theocông thức truy hồi, của PTSP tuyến tính thuần nhất trong một số trường hợp đơn giản,được thể hiện qua các định lý dưới đây.

Định lý 3.1. Nếu (3) có k nghiệm thực phân biệt khác nhau là λ1,λ2, ...,λk thì nghiệm

tổng quát xn của (2) có dạng

xn =k

∑i=1

ciλni ,

trong đó ci; i = 1,2, ...,k là các hằng số thực tùy ý.

Định lý 3.2. Nếu phương trình đặc trưng (3) có nghiệm thực λ j, bội s, thì ngoài nghiệm

λ nj , ta bổ xung thêm các nghiệm nλ n

j , n2λ nj , ..., ns−1λ n

j . Khi đó, nghiệm tổng quát của

(2) là

xn =s−1

∑i=0

cijn

nj +

k

∑j 6=i≥1

c1λni ,

trong đó cij, ci là các hằng số thực tùy ý; và tổng ∑ thứ hai chỉ có (k− s) nghiệm λi.

Định lý 3.3. Nếu phương trình (3) có nghiệm phức (đơn) λ j = a+bi = r(cosϕ + sinϕ),

trong đó r = |λ j|=√

a2 +b2, ϕ = argumen λ j, có nghĩa là tanϕ =ba, thì (3) cũng có

nghiệm phức liên hợp λ j. Khi đó nghiệm tổng quát của (2) có dạng

xn =k

∑j 6=i=1

ciλni + rn(c1

jcos(nϕ)+ c2j sin(nϕ)),

trong đó ci, c1j , c2

i là các hằng số tùy ý.

80

Page 86: Đ—I H¯C QU¨C GIA HÀ N¸I TRƯ˝NG Đ—I H¯C KHOA H¯C TÜ NHIÊN KHOA … · 2016-05-09 · Đ—I H¯C QU¨C GIA HÀ N¸I TRƯ˝NG Đ—I H¯C KHOA H¯C TÜ NHIÊN KHOA TOÁN

Trong 2 định lý trên, các nghiệm còn lại đều là nghiệm thực đơn. Các trường hợpkhác; cũng như đối với PTSP tuyến tính không thuần nhất, cách xây dựng nghiệm riêngvà nghiệm tổng quát trong trường hợp này, bạn đọc có thể tự tìm hiểu thêm.

Ta áp dụng lý thuyết phương trình sai phân đối với hàm số khi tập xác định của hàmđó là N hoặc Z. Một trường hợp thường gặp khác khi ta làm việc với biểu thức dạng

∑i∈N

ai f i(x). Khi đó, với mỗi x cố định, ta có thể xác định dãy số như sau

x0 = x, x1 = f (x), x2 = f 2(x), ..., xk = f k(x) = f ( f k−1)(x), ...

3.6.2 Một số bài toán vận dụng

Bài toán 3.6.1. (Dãy Fibonacci) Tìm hàm f : N→ R thỏa mãn f (0) = 0, f (1) = 1 và

f (n) = f (n−1)+ f (n−2), ∀2≤ n ∈ N.

LỜI GIẢI. Đặt xn = f (n), ∀n ∈ N. Khi đó ta có x0 = 0, x1 = 1 và

xn = xn−1 + xn−2, ∀2≤ n ∈ N.

Phương trình đặc trưng với dãy xn là λ 2−λ − 1 = 0, phương trình này có 2 nghiệm

phân biệt λ =1±√

52

. Do đó công thức tổng quát của dãy số là

xn = c1(1+√

52

)n + c2(1−√

52

)n, ∀n ∈ N.

Do x0 = 0, x1 = 1 nên ta có c1 + c2 = 0 và c11+√

52

+ c21−√

52

= 1. Từ đó ta có

c1 =1√5, c =

−1√5.

Vậy f (n) = xn =1√5(1+√

52

)n− 1√5(1−√

52

)n, ∀n ∈ N.

Bài toán 3.6.2. Cho f : N∗→ R thỏa mãn điều kiện f (1) = 1, f (2) = 0 và

f (n+2) = f (n+1)− f (n), ∀n ∈ N∗.

Chứng minh rằng | f (n)| ≤√

72

, ∀n ∈ N∗.

81

Page 87: Đ—I H¯C QU¨C GIA HÀ N¸I TRƯ˝NG Đ—I H¯C KHOA H¯C TÜ NHIÊN KHOA … · 2016-05-09 · Đ—I H¯C QU¨C GIA HÀ N¸I TRƯ˝NG Đ—I H¯C KHOA H¯C TÜ NHIÊN KHOA TOÁN

LỜI GIẢI. Phương trình đặc trưng λ 2−λ +1 = 0 có nghiệm λ =1± i√

32

. Ta có

r = |λ |=√

1/4+3/4 = 1; tanϕ =√

3/21/2

=√

3⇒ ϕ =π

3.

Do đó λ = cos(π/3)± isin(π/3) và

f (n) = c1 cos(nπ/3)+ c2 sin(nπ/3), ∀n ∈ N∗.

Kết hợp f (1) = 1, f (2) = 0 ta suy ra c1 = 1 và c2 =√

3/2. Từ đó

f (n) = cosnπ

3+

32

sinnπ

3, ∀n ∈ N∗.

Từ đó suy ra

| f (n)| ≤

√12 +(

√3

2)2 =

√7

2, ∀n ∈ N∗ (ĐPCM) .

Bài toán 3.6.3. Tìm hàm f : N→ N thỏa mãn

f (2 f (n)+3 f (m)) = 2n+3m, ∀m,n ∈ N. (*)

LỜI GIẢI. Dễ thấy f đơn ánh. Với mọi m,n, p,q ∈ N mà 2n+3m = 2p+3q ta có

f (2 f (n)+3 f (m)) = 2n+3m = 2p+3q = f (2 f (p)+3 f (q)),

do f đơn ánh nên 2 f (n)+3 f (m) = 2 f (p)+3 f (q), ∀ 2n+3m = 2p+3q.Từ 2n +3m = 2p +3q suy ra 2(n− p) = 3(q−m), nên một cách chọn đơn giản nhất làn− p = 3, q−m = 2. Khi đó ta có 2 f (p+3)+3 f (m) = 2 f (p)+3 f (m+2), ∀m, p ∈N.Tiếp tục, ta chọn p = m thì 2 f (m+3)−3 f (m+2)+ f (m) = 0, ∀m ∈ N, (1).Phương trình đặc trưng của (1) là 2x3− 3x2 + 1 = 0, có nghiệm x1 = 1 bội 2, và

x2 = −1/2. Suy ra công thức tổng quát của f (m) là f (m) = c1 + c2m + c3(−12)m,

với ci, i = 1,2,3, là các hằng số tự nhiên, nhưng do f (m) ∈ N nên c3 = 0. Do đó,f (m) = c1 + c2m, ∀m ∈ N.Thay lại vào (*) ta sẽ suy ra c1 = 0, c2 = 1. Vậy f (m) = m, ∀m ∈ N.

Nhận xét: Có thể mở rộng bài toán này bằng cách biến đổi phương trình (*) thànhf (a f (n)+b f (m)) = an+bm, với a,b∈N. Tuy nhiên, chúng ta có thể nhận thấy nếu giảibằng phương pháp qui về dãy số sẽ có khó khăn vì PTSP bậc cao. Có một cách giải đặcsắc tương tự với bài toán Canada 2008. Xin dành cho bạn đọc quan tâm tìm hiểu thêm.

82

Page 88: Đ—I H¯C QU¨C GIA HÀ N¸I TRƯ˝NG Đ—I H¯C KHOA H¯C TÜ NHIÊN KHOA … · 2016-05-09 · Đ—I H¯C QU¨C GIA HÀ N¸I TRƯ˝NG Đ—I H¯C KHOA H¯C TÜ NHIÊN KHOA TOÁN

Bài toán 3.6.4. (IMO 1992, Shortlist) Cho a,b là hai số thực dương. Xác định tất cả

các hàm f : [0,+∞)→ [0,+∞) thỏa mãn

f ( f (x))+a f (x) = b(a+b)x, ∀x≥ 0.

LỜI GIẢI. Thay x bởi f (x), f 2(x) = f ( f (x)), ..., f n(x), ... ta suy ra

f n+2(x)+a f n+1(x) = b(a+b) f n(x), ∀n ∈ N.

Với mỗi x bất kì, cố định nó và đặt x0 = x, xn+1 = f (xn) = f n+1(x), n ∈N ta thu đượcdãy số thực xn (phụ thuộc vào x trên) xác định bởi x0 = x, x1 = f (x) và

xn+2 +axn+1 = b(a+b)xn, ∀x ∈ N.

Phương trình đặc trưng λ 2 + aλ − b(a + b) = 0 có nghiệm λ = b hoặc λ = −a− b.

Do đó ta có công thức tổng quát của dãy như sau

xn = c1bn + c2(−a−b)n, ∀n ∈ N.

Nếu c2 6= 0 thì do a,b > 0 nên limn→+∞

|(−a−b)n||b|n

= +∞. Do đó tồn tại n ∈ N đủ

lớn thỏa mãn xn = c1bn + c2(−a− b)n < 0. Nhưng khi đó f n(x) < 0, mâu thuẫn vớigiả thiết f : [0,+∞)→ [0,+∞). Do vậy ta phải có c2 = 0. Khi đó với n = 0 ta cóx = x0 = c1; còn với n = 1 ta có f (x) = x1 = c1b do đó f (x) = bx, ∀x≥ 0.

Thử lại, ta đi đến kết luận nghiệm của bài toán là f (x) = bx, ∀x≥ 0.

Bài toán 3.6.5. Cho trước k ∈ N. Tìm hàm f : N→ N thỏa mãn

f ( f (x))+ f (x) = 2x+3k, ∀x ∈ N.

LỜI GIẢI. Với mỗi x đặt a0 = x và với n≥ 1 đặt an+1 = f (an). Khi đó ta có

an+2 +an+1 = 2an +3k và an+3 +an+2 = 2an+1 +3k, ∀n ∈ N.

Suy ra an+3− 3an+1 + 2an = 0, ∀n ∈ N. Phương trình đặc trưng λ 3− 3λ + 2 = 0 cónghiệm λ1 = 1 bội 2 và λ2 =−2. Nên công thức tổng quát của an là

an = c1 + c2n+ c3(−2)n, ∀n ∈ N.

Nếu c3 6= 0 thì tồn tại n mà an < 0, mâu thuẫn. Do đó c3 = 0 và an = c1 + c2n. Thayvào phương trình xác định dãy an ta có

c1 + c2(n+2)+ c1 + c2(n+1) = 2(c1 + c2n)+3k ⇒ c2 = k.

83

Page 89: Đ—I H¯C QU¨C GIA HÀ N¸I TRƯ˝NG Đ—I H¯C KHOA H¯C TÜ NHIÊN KHOA … · 2016-05-09 · Đ—I H¯C QU¨C GIA HÀ N¸I TRƯ˝NG Đ—I H¯C KHOA H¯C TÜ NHIÊN KHOA TOÁN

Do đó an = c1 + kn. Bây giờ chú ý a0 = x, a1 = f (x) ta suy ra c1 = x, f (x) = x + k.

Thử lại, ta thấy f (x) = x+ k, ∀x ∈ N là nghiệm của bài toán.Nhận xét: Tương tự bài toán này, ta có thể giải được bài toán IMO 1987 như sau:Chứng minh không tồn tại f : N→ N thỏa mãn f ( f (n)) = n+1987, ∀n ∈ N.

MỘT SỐ BÀI TẬP TRONG MỤC NÀY:

Bài tập 3.6.1. Xác định hàm f : N→ N trong các trường hợp sau

(i). f (0) = 3, f (1) = 8 và f (n) = 5 f (n−1)−6 f (n−2).(ii). f (n+2)+ f (n+1)−2 f (n) = 0, ∀n ∈ N. Chứng minh rằng f (n) = f (0), ∀n ∈ N.

(iii). Chứng minh không tồn tại f : N∗→ N∗ thỏa mãn f ( f (n)) = n+1987, ∀n ∈ N∗.(iv). f (1) = 2, f (n+1) > f (n), f ( f (n)) = f (n)+n, ∀n ∈ N∗.

Bài tập 3.6.2. (Romania 1999) Xác định hàm đơn điệu f : R→ R thỏa mãn

f ( f ( f (x)))−3 f ( f (x))+6 f (x) = 4x+3, ∀x ∈ R.

Đáp số. f (x) = x+1, ∀x ∈ R.

Bài tập 3.6.3. Chứng minh rằng không tồn tại hàm f : N∗→ N∗ thỏa mãn

f (m f (n)) = n+ f (2011m), ∀m,n ∈ N∗.

Gợi ý. Dễ thấy f đơn ánh. Ta có f ( f (m) f (n))= n+ f (2011 f (m))= n+m+ f (20112).Từ đó với mọi m+n = p+q thì ta có f (n) f (m) = f (p) f (q). Suy ra

f (n)f (p)

=f (q)f (m)

⇒ f (n)f (n−1)

=f (n−1)f (n−2)

= ... =f (2)f (1)

= k (const).

Dẫn đến f (n) = kn−1 f (1). Từ đó k = 1 và f (n) = f (1), không thỏa mãn với điều kiện.

Bài tập 3.6.4. Cho f : N∗→N∗ thỏa mãn f ( f (n)) = f (n+1)+ f (n), ∀n ∈N∗. Chứng

minh rằng f (n) là hàm đơn ánh.

Gợi ý. Giả sử tồn tại m > n mà f (m) = f (n). Từ bài toán suy ra f (m + 1) = f (n +1). Từ đó f (m + k) = f (n + k), ∀k ∈ N. Và như vậy f (n) chỉ nhận hữu hạn giá trịnguyên dương. Mặt khác f ( f (1)) = f (2)+ f (1)≥ 2 suy ra f ( f ( f (1))) = f ( f (1)+1)+f ( f (1))≥ 3. Bằng quy nạp ta có f k(1)≥ k, ∀k ∈ N∗. Điều này dẫn đến mâu thuẫn.

Bài tập 3.6.5. (Balkan 2002) Tìm tất cả các hàm số f : N→ N thỏa mãn

f ( f (n))+ f (n) = 2n+2001 hoặc 2n+2002, ∀n ∈ N.

Đáp số. f (n) = n+667.

84

Page 90: Đ—I H¯C QU¨C GIA HÀ N¸I TRƯ˝NG Đ—I H¯C KHOA H¯C TÜ NHIÊN KHOA … · 2016-05-09 · Đ—I H¯C QU¨C GIA HÀ N¸I TRƯ˝NG Đ—I H¯C KHOA H¯C TÜ NHIÊN KHOA TOÁN

Bài tập 3.6.6. (VietNam 2003) Cho F là tập hợp tất cả hàm số f : R+→ R+ thỏa mãn

f (3x)≥ f ( f (2x))+ x, ∀x > 0.

Tìm α ∈ R lớn nhất sao cho với mọi hàm f ∈ F ta đều có f (x)≥ α x, ∀x > 0.

Đáp số. α = 1/2.

85

Page 91: Đ—I H¯C QU¨C GIA HÀ N¸I TRƯ˝NG Đ—I H¯C KHOA H¯C TÜ NHIÊN KHOA … · 2016-05-09 · Đ—I H¯C QU¨C GIA HÀ N¸I TRƯ˝NG Đ—I H¯C KHOA H¯C TÜ NHIÊN KHOA TOÁN

Kết luận

Sau thời gian hai năm học tập tại Khoa Toán – Cơ – Tin, Trường Đại học Khoa học Tựnhiên Hà Nội, được sự giúp đỡ chỉ bảo tận tình của các thầy cô trong khoa, đặc biệt làTS. Nguyễn Thành Văn, tôi đã hoàn thành luận văn với tên đề tài “Một số lớp bài toánvề phương trình hàm”.

Luận văn đã đạt được một số kết quả sau :

1. Luận văn đã nêu ra được một số kiến thức cơ bản trong trong đại số và giải tích cóứng dụng nhiều trong việc giải quyết các bài toán phương trình hàm.

2. Luận văn đã hệ thống và phân loại một số dạng toán thường gặp theo phương phápgiải của các bài toán phương trình hàm với nhiều bài toán có lời giải, nhận xét vàbình luận.

3. Luận văn đã nêu ra một số hướng khai thác mở rộng, tổng quát, và các hướng tưduy tìm lời giải cùng các biến hóa trong một số dạng toán phương trình hàm.

Phân dạng các phương trình hàm sẽ giúp cho sự định hướng và giải quyết chúng. Vìvậy tôi hi vọng bản luận văn này có thể làm tài liệu tham khảo cho quá trình nghiên cứu,giảng dạy và học tập toán ở bậc phổ thông. Tôi rất mong nhận được sự góp ý của cácthầy cô và các bạn đồng nghiệp để đề tài này tiếp tục được hoàn thiện.

Xin chân thành cảm ơn!

86

Page 92: Đ—I H¯C QU¨C GIA HÀ N¸I TRƯ˝NG Đ—I H¯C KHOA H¯C TÜ NHIÊN KHOA … · 2016-05-09 · Đ—I H¯C QU¨C GIA HÀ N¸I TRƯ˝NG Đ—I H¯C KHOA H¯C TÜ NHIÊN KHOA TOÁN

Tài liệu tham khảo

[1 ] Trần Nam Dũng, Tài liệu bồi dưỡng đội tuyển Việt Nam tham dự IMO 2010.

[2 ] PGS.TS. Nguyễn Quý Dy (chủ biên), Tuyển tập 200 bài thi vô địch toán - Tập 3,

Nhà xuất bản giáo dục, 2001.

[3 ] Nguyễn Văn Mậu, Phương trình hàm, Nhà xuất bản Giáo dục, 2001.

[4 ] Lê Đình Thịnh, Lê Đình Định, Phương pháp sai phân, Nhà xuất bản Đại học Quốcgia Hà Nội.

[5 ] Tạp chí Toán học và Tuổi trẻ.

[6 ] Christopher G.Small, Functional equations and how to solve them, Springer.

[7 ] Dusan Djukic, Vladimir Jankovic, Ivan Matic, Nikola Petrovic, A Colection of

Problems Suggested for The IMO 1959-2004, Springer 2004.

[8 ] Marko Radovanovic, Functional Equations, The Authors and The IMO Com-pendium Group 2007.

[9 ] Mohamed Akkouchi, A Class of Functional Equations Characterizing Ponolymi-

als of Dgree Two, 2001.

[10 ] Pierre Bontein, Moubinool Omarjee , Cours - Equations Fonctionelles, Mardi2003.

[11 ] Website: mathlinks.ro, vnmath.com, diendantoanhoc.net...

87